You are on page 1of 29

Labor Relations Set IV * Takata v BLR to Silang v Coa * Page 1 of 29

Republic of the Philippines Accordingly, Samahang Lakas Manggagawa ng TAKATA (SALAMAT) shall remain in the roster of labor organizations. 15
SUPREME COURT
Manila
In reversing, the BLR found that petitioner failed to prove that respondent deliberately and maliciously misrepresented the number of rank-and-file
THIRD DIVISION
employees. It pointed out petitioner's basis for the alleged noncompliance with the minimum membership requirement for registration was the attendance
G.R. No. 196276 June 4, 2014
of 68 members to the May 1, 2009 organizational meeting supposedly comprising only 17% of the total 396 regular rank-and-file employees. However,
TAKATA (PHILIPPINES) CORPORATION, Petitioner,
the BLR found that the list of employees who participated in the organizational meeting was a separate and distinct requirement from the list of the names
vs.
of members comprising at least 20% of the employees in the bargaining unit; and that there was no requirement for signatures opposite the names of the
BUREAU OF LABOR RELATIONS and SAMAHANG LAKAS MANGGAGAWA NG TAKATA (SALAMAT),Respondents.
union members; and there was no evidence showing that the employees assailed their inclusion in the list of union members.
DECISION
PERALTA, J.:
Petitioner filed a motion for reconsideration, which was denied by the BLR in a Resolution16 dated January 8, 2010.
Before us is a petition for review on certiorari filed by petitioner TAKATA Philippines Corporation assailing the Decision 1 dated December 22, 2010 and
the Resolution2 dated March 28, 2011 of the Court of Appeals in CA-G.R. SP No. 112406.
Undaunted, petitioner went to the CA via a petition for certiorari under Rule 65.

On July 7, 2009, petitioner filed with the Department of Labor and Employment (DOLE) Regional Office a Petition3for Cancellation of the Certificate of
Union Registration of Respondent Samahang Lakas Manggagawa ng Takata (SALAMA1) on the ground that the latter is guilty of misrepresentation, false After the submission of the parties' respective pleadings, the case was submitted for decision.
statement and fraud with respect to the number of those who participated in the organizational meeting, the adoption and ratification of its Constitution
and By-Laws, and in the election of its officers. It contended that in the May 1, 2009 organizational meeting of respondent, only 68 attendees signed the On December 22, 2010, the CA rendered its assailed decision which denied the petition and affirmed the decision of the BLR. Petitioner's motion for
attendance sheet, and which number comprised only 17% of the total number of the 396 regular rank- and-file employees which respondent sought to reconsideration was denied in a Resolution dated March 29, 2011.
represent, and hence, respondent failed to comply with the 20% minimum membership requirement. Petitioner insisted that the document "Pangalan ng
mga Kasapi ng Unyon" bore no signatures of the alleged 119 union members; and that employees were not given sufficient information on the documents
they signed; that the document "Sama-Samang Pahayag ng Pagsapi" was not submitted at the time of the filing of respondent's application for union Hence this petition for review filed by petitioner raising the following issues, to wit:
registration; that the 119 union members were actually only 117; and, that the total number of petitioner's employees as of May 1, 2009 was 470, and not
396 as respondent claimed.4 THE HONORABLE COURT OF APPEALS COMMITTED GRAVE AND SERIOUS ERROR IN AFFIRMING THE DECISION OF PUBLIC RESPONDENT
BLR AND NOT FINDING ANY VIOLATION BY SAMAHANG LAKAS MANGGAGAWA SA TAKATA (SALAMAT) OF THE RULE ON FORUM SHOPPING
Respondent denied the charge and claimed that the 119 union members were more than the 20% requirement for union registration. The document IN THE FILING OF TWO VERIFIED APPEALS FOR AND ITS BEHALF. BOTH OF THE APPEALS SHOULD HAVE BEEN DISMISSED OUTRIGHT BY
"Sama-Samang Pahayag ng Pagsapi sa Unyon" which it presented in its petition for certification election 5 supported their claim of 119 members. PUBLIC RESPONDENT BLR, ON GROUND OF FORUM SHOPPING.
Respondent also contended that petitioner was estopped from assailing its legal personality as it agreed to a certification election and actively
participated in the pre-election conference of the certification election proceedings.6 Respondent argued that the union members were informed of the THE HONORABLE COURT OF APPEALS SERIOUSLY ERRED IN FINDING THAT THE APPLICATION FOR REGISTRATION OF SAMAHANG LAKAS
contents of the documents they signed and that the 68 attendees to the organizational meeting constituted more than 50% of the total union membership, MANGGAGAWA SA TAKATA (SALAMAT) WAS COMPLIANT WITH THE LAW. CONSIDERING THE CIRCUMSTANCES OBTAINING IN THE
hence, a quo rumexisted for the conduct of the said meeting.7 REGISTRATION OF SALAMAT, IT IS CLEAR THAT THE SAME IS TAINTED WITH FRAUD, MISREPRESENTATION AND FALSIFICATION. SALAMAT
DID NOT POSSESS THE REQUIREDNUMBER OF MEMBERS AT THE TIME OF FILING OF ITS APPLICATION FOR REGISTRATION, HENCE, IT
On August 27, 2009, DOLE Regional Director, Atty. Ricardo S. Martinez, Sr., issued a Decision 8 granting the petition for cancellation of respondent's SHOULD BE HELD GUILTY OF MISREPRESENTATION, AND FALSE STATEMENTS AND FRAUD IN CONNECTION THEREWITH.17
certificate of registration, the dispositive portion of which reads:
Anent the first issue, petitioner contends that respondent had filed two separate appeals with two different representations at two different venues, in
WHEREFORE, from the foregoing considerations, the petition is hereby GRANTED. Accordingly, the respondent Union Certificate of Registration No. violation of the rule on multiplicity of suits and forum shopping, and instead of dismissing both appeals, the appeal erroneously filed before the Labor
RO400A-2009-05-01-UR-LAG, dated May 19, 2009 is hereby REVOCKED (sic) and /or CANCELLED pursuant to paragraph (a) & (b), Section 3, Rule Secretary was the one held validly filed, entertained and even granted; that it is not within the discretion of BLR to choose which between the two appeals
XIV of Department Order No. 40-03 and the Samahang Lakas ng Manggagawa ng TAKATA (SALAMAT) is hereby delisted from the roll of legitimate should be entertained, as it is the fact of the filing of the two appeals that is being prohibited and not who among the representatives therein possessed
labor organization of this office.9 the authority.

In revoking respondent's certificate of registration, the Regional Director found that the 68 employees who attended the organizational meeting was We are not persuaded.
obviously less than 20% of the total number of 396 regular rank-and-file employees which respondent sought to represent, hence, short of the union
registration requirement; that the attendance sheet which contained the signatures and names of the union members totalling to 68 contradicted the list of We find no error committed by the CA in finding that respondent committed no forum shopping. As the CA correctly concluded, to wit:
names stated in the document denominated as "Pangalan ng mga Kasaping Unyon." The document "Sama-Samang Pahayag ng Pagsapi" was not
attached to the application for registration as it was only submitted in the petition for certification election filed by respondent at a later date. The Regional
Director also found that the proceedings in the cancellation of registration and certification elections are two different and entirely separate and It is undisputed that BMP Paralegal Officer Domingo P. Mole was no longer authorized to file an appeal on behalf of union SALAMAT and that BMP was
independent proceedings which were not dependent on each other. duly informed that its services was already terminated. SALAMAT even submitted before the BLR its "Resolusyon Blg. 01-2009" terminating the services
of BMP and revoking the representation of Mr. Domingo Mole in any of the pending cases being handled by him on behalf of the union. So, considering
that BMP Paralegal Officer Domingo P. Mole was no longer authorized to file an appeal when it filed the Notice and Memorandum of Appeal to DOLE
Dissatisfied, respondent, through Bukluran ng Manggagawang Pilipino (BMP) Paralegal Officer, Domingo P. Mole, filed a Notice and Memorandum of Regional Office No. IV-A, the same can no longer be treated as an appeal filed by union SALAMAT. Hence, there is no forum shopping to speak of in this
Appeal10 with the Bureau of Labor Relations (BLR). However, on September 28,2009, respondent, through its counsels, Attys. case as only the Appeal Memorandum with Formal Entry of Appearance filed by Atty. Napoleon C. Banzuela, Jr. and Atty. Jehn Louie W. Velandrez is
sanctioned by SALAMAT.18
Napoleon C. Banzuela, Jr. and Jehn Louie W. Velandrez, filed an Appeal Memorandum with Formal Entry of Appearance11 to the Office of the DOLE
Secretary, which the latter eventually referred to the BLR. Petitioner filed an Opposition to the Appeals 12 praying for their dismissal on the ground of forum Since Mole's appeal filed with the BLR was not specifically authorized by respondent, such appeal is considered to have not been filed at all. It has been
shopping as respondent filed two separate appeals in two separate venues; and for failing to avail of the correct remedy within the period; and that the held that "if a complaint is filed for and in behalf of the plaintiff who is not authorized to do so, the complaint is not deemed filed.
certificate of registration was tainted with fraud, misrepresentation and falsification.
An unauthorized complaint does not produce any legal effect."19
In its Answer,13 respondent claimed that there was no forum shopping as BMP's Paralegal Officer was no longer authorized to file an appeal on behalf of
respondent as the latter's link with BMP was already terminated and only the Union President was authorized to file the appeal; and that it complied with
Department Order No. 40-03. Respondent through its authorized representative filed its Appeal Memorandum with Formal Entry of Appearance before the Labor Secretary, and not
with the BLR. As the appeal emanated from the petition for cancellation of certificate of registration filed with the Regional Office, the decision canceling
the registration is appealable to the BLR, and not with the Labor Secretary. However, since the Labor Secretary motu propio referred the appeal with the
On December 9, 2009, after considering respondent's Appeal Memorandum with Formal Entry of Appearance and petitioner's Answer, the BLR rendered BLR, the latter can now act on it. Considering that Mole's appeal with the BLR was not deemed filed, respondent’s appeal, through Banzuela and
its Decision14 reversing the Order of the Regional Director, the decretal portion of which reads: Associates, which the Labor Secretary referred to the BLR was the only existing appeal with the BLR for resolution. There is, therefore, no merit to
petitioner's claim that BLR chose the appeal of Banzuela and Associates over Mole's appeal.
WHEREFORE, the appeal is hereby GRANTED. The Decision of Regional Director Ricardo S. Martinez, Sr., dated 27 August 2009, is hereby
REVERSEDand SET ASIDE.
Labor Relations Set IV * Takata v BLR to Silang v Coa * Page 2 of 29

The case of Abbott Laboratories Philippines, Inc. v. Abbott Laboratories Employees Union 20 cited by petitioner is not at all applicable in this case as the "Pangalan ng Mga Kasapi ng Unyon" showing the names of 119 employees as union members, thus respondent sufficiently complied even beyond the
issue therein is the authority of the Labor Secretary to review the decision of the Bureau of Labor Relations rendered in the exercise of its appellate 20% minimum membership requirement. Respondent also submitted the attendance sheet of the organizational meeting which contained the names and
jurisdiction over decision of the Regional Director in cases involving cancellations of certificate of registration of labor unions. We found no grave abuse of signatures of the 68 union members who attended the meeting. Considering that there are 119 union members which are more than 20% of all the
discretion committed by the Secretary of Labor in not acting on therein petitioner's appeal. The decision of the Bureau of Labor Relations on cases employees of the bargaining unit, and since the law does not provide for the required number of members to attend the organizational meeting, the 68
brought before it on appeal from the Regional Director are final and executory. Hence, the remedy of the aggrieved party is to seasonably avail of the attendees which comprised at least the majority of the 119 union members would already constitute a quorum for the meeting to proceed and to validly
special civil action of certiorari under Rule 65 and the Rules of Court. In this case, after the Labor Secretary motu propio referred respondent's appeal ratify the Constitution and By-laws of the union. There is, therefore, no basis for petitioner to contend that grounds exist for the cancellation of
filed with it to the BLR which rendered its decision reversing the Regional Director, petitioner went directly to the CA via a petition for certiorari under Rule respondent's union registration. For fraud and misrepresentation to be grounds for cancellation of union registration under Article 239 of the Labor Code,
65. the nature of the fraud and misrepresentation must be grave and compelling enough to vitiate the consent of a majority of union members.22

As to the second issue, petitioner seeks the cancellation of respondent's registration on grounds offraud and misrepresentation bearing on the minimum Petitioner's claim that the alleged union members signed documents without adequate information is not persuasive. The one who alleges a fact has the
requirement of the law as to its membership, considering the big disparity in numbers, between the organizational meeting and the list of members, and burden of proving it and a mere allegation is not evidence.23 In fact, we note that not one of those listed in the document denominated as "Pangalan ng
so misleading the BLR that it obtained the minimum required number of employees for purposes of organization and registration. Mga Kasaping Unyon" had come forward to deny their membership with respondent. Notably, it had not been rebutted that the same union members had
signed the document entitled "Sama-Samang Pahayag ng Pagsapi," thus, strengtheningtheir desire to be members of the respondent union.
We find no merit in the arguments.
Petitioner claims that in the list of members, there was an employee whose name appeared twice and another employee who was merely a project
employee. Such could not be considered a misrepresentation in the absence of showing that respondent deliberately did so for the purpose of increasing
Art. 234 of the Labor Code provides:
their union membership. In fact, even if those two names were not included in the list of union members, there would still be 117 members which was still
more than 20% of the 396 rank-and-file employees.
ART. 234. Requirements of Registration. - A federation, national union or industry or trade union center or an independent union shall acquire legal
personality and shall be entitled to the rights and privileges granted by law to legitimate labor organizations upon issuance of the certificate of registration
As to petitioner's argument that the total number of its employees as of May 1, 2009 was 470, and not396 as respondent claimed, still the 117 union
based on the following requirements:
members comprised more than the 20% membership requirement for respondent's registration.

(a) Fifty pesos (P50.00)registration fee;


In Mariwasa Siam Ceramics v. Secretary of the Department of Labor and Employment, 24 we said:
(b) The names of its officers, their addresses, the principal address of the labor organization, the minutes of the organizational meetings and
the list of the workers who participated in such meetings;
(c) In case the applicant is an independent union, the names of all its members comprising at least twenty percent (20%) of all the employees in For the purpose of de-certifying a union such as respondent, it must be shown that there was misrepresentation, false statement or fraud in connection
the bargaining unit where it seeks to operate; with the adoption or ratification of the constitution and by-laws or amendments thereto, the minutes of ratification; or, in connection with the election of
(d) If the applicant union has been in existence for one or more years, copies of its annual financial reports; and officers, the minutes of the election of officers, the list of voters, or failure to submit these documents together with the list of the newly elected-appointed
(e) Four copies of the constitution and by-laws of the applicant union, minutes of its adoption or ratification, and the list of the members who officers and their postal addresses to the BLR.
participated in it."
The bare fact that two signatures appeared twice on the list of those who participated in the organizational meeting would not, to our mind, provide a valid
And after the issuance of the certificate of registration, the labor organization's registration could be assailed directly through cancellation of registration
reason to cancel respondent’s certificate of registration. The cancellation of a union’s registration doubtless has an impairing dimension on the right of
proceedings in accordance with Articles 238 and 239 of the Labor Code. And the cancellation of union certificate of registration and the grounds thereof
labor to self-organization. For fraud and misrepresentation to be grounds for cancellation of union registration under the Labor Code, the nature of the
are as follows:
fraud and misrepresentation must be grave and compelling enough to vitiate the consent of a majority of union members.

ART. 238. Cancellation of Registration. - The certificate of registration of any legitimate labor organization, whether national or local, may be cancelled by
In this case, we agree with the BLR and the CA that respondent could not have possibly committed misrepresentation, fraud, or false statements. The
the Bureau, after due hearing, only on the grounds specified in Article 239 hereof.
alleged failure of respondent to indicate with mathematical precision the total number of employees in the bargaining unit is of no moment, especially as it
was able to comply with the 20% minimum membership requirement. Even if the total number of rank-and-file employees of petitioner is 528, while
ART. 239. Grounds for Cancellation of Union Registration. - The following may constitute grounds for cancellation of union registration: respondent declared that it should only be 455, it still cannot be denied that the latter would have more than complied with the registration
requirement.25WHEREFORE, premises considered, the petition for review is DENIED. The Decision dated December 22, 2010 and the Resolution dated
March 28, 2011 of the Court of Appeals, in CA-G.R. SP No. 112406, are AFFIRMED.SO ORDERED
(a) Misrepresentation, false statement or fraud in connection with the adoption or ratification of the constitution and by-laws or amendments
thereto, the minutes of ratification, and the list of members who took part in the ratification;
Republic of the Philippines
SUPREME COURT
(b) Misrepresentation, false statements or fraud in connection with the election of officers, minutes of the election of officers, and the list of
Manila
voters;
SECOND DIVISION
G.R. No. 163532 March 10, 2010
(c) Voluntary dissolution by the members. YOKOHAMA TIRE PHILIPPINES, INC., Petitioner,
vs.
YOKOHAMA EMPLOYEES UNION, Respondent.
Petitioner's charge that respondent committed misrepresentation and fraud in securing its certificate of registration is a serious charge and must be RESOLUTION
carefully evaluated. Allegations thereof should be compounded with supporting circumstances and evidence. 21 We find no evidence on record to support CARPIO, J.:
petitioner's accusation.
This is a petition1 for review on certiorari under Rule 45 of the Rules of Court. The petition challenges the 16 January 2004 Decision2 and 12 May 2004
Petitioner's allegation of misrepresentation and fraud is based on its claim that during the organizational meeting on May 1, 2009, only 68 employees Resolution3 of the Court of Appeals in CA-G.R. SP No. 65460. The Court of Appeals affirmed the 12 March 4 and 3 May5 2001 Resolutions of the Bureau
attended, while respondent claimed that it has 119 members as shown in the document denominated as "Pangalan ng mga Kasapi ng Unyon;" hence, of Labor Relations (BLR) in BLR-A-C-7-2-05-01, reversing the 18 December 2000 Decision6 of the Department of Labor and Employment (DOLE)
respondent misrepresented on the 20% requirement of the law as to its membership. Regional Office No. 3, San Fernando, Pampanga (Regional Office), in Case No. RO300-0001-CP-002.

We do not agree. Yokohama Employees Union (YEU) is the labor organization of the rank-and-file employees of Yokohama Tire Philippines, Inc. (YTPI). YEU was
registered as a legitimate labor labor union on 10 September 1999.
It does not appear in Article 234 (b) of the Labor Code that the attendees in the organizational meeting must comprise 20% of the employees in the
bargaining unit. In fact, even the Implementing Rules and Regulations of the Labor Code does not so provide. It is only under Article 234 (c) that requires YEU filed before the Regional Office a petition for certification election. YTPI filed before the Regional Office a petition 7 dated 24 January 2000 for the
the names of all its members comprising at least twenty percent (20%) of all the employees in the bargaining unit where it seeks to operate. Clearly, the revocation of YEU’s registration. YTPI alleged that YEU violated Article 239(a) 8 of the Labor Code: (1) YEU fraudulently included the signature of a
20% minimum requirement pertains to the employees’ membership in the union and not to the list of workers who participated in the organizational certain Ronald O. Pineda (Pineda) in the organizational documents; (2) Pineda was not aware of any election of union officers; (3) YEU fraudulently
meeting. Indeed, Article 234 (b) and (c) provide for separate requirements, which must be submitted for the union's registration, and which respondent did obtained the employees’ signatures by making them believe that they were signing a petition for a 125% increase in the minimum wage, not a petition for
submit. Here, the total number of employees in the bargaining unit was 396, and 20% of which was about 79. Respondent submitted a document entitled
Labor Relations Set IV * Takata v BLR to Silang v Coa * Page 3 of 29

registration; (4) the employees did not belong to a single bargaining unit; and (5) YEU fraudulently stated in its organizational meeting minutes that its denied any knowledge of the holding of an election. A perusal of the affirmative handwritten statement easily explains why the public respondent
second vice president was Bernard David, not Bernardo David. preferred it to the negating affidavit, to wit:

In its 18 December 2000 Decision, the Regional Office granted the 24 January 2000 petition. The Regional Office held that YEU committed Noong unang araw na pumirma ako galing ako sa graveyard. Pagkatapos yung pangalawang meetinggraveyard din ako, pinapirma ako doon sa
misrepresentation: (1) YEU failed to remove Pineda’s signature from the organizational documents despite instructions to do so; and (2) YEU declared siyam (9) na pirasong papel noong umagang pag-uwi namin. x x x
that it conducted an election of union officers when, in truth, it did not.
July 25, 99 - Unang Pirmahan
YEU appealed the 18 December 2000 Decision to the BLR. In its 12 March 2001 Resolution, the BLR reversed the 18 December 2000 Decision. The July 26, 99 - Pinirmahan ko ang siyam na piraso
BLR found that (1) Pineda did not approach any officer of YEU to have his signature removed from the organizational documents; (2) Pineda’s affidavit July 27, 99 - Pinatatanggal ko ang aking pangalan sa listahan
that no election of officers took place was unreliable and inconsistent with his earlier written statement; (3) the affidavit of a certain Rachelle Gonzales
(Gonzales) that no election of officers took place was unreliable and inconsistent with her earlier resignation letter; (4) the affidavit of a certain Arthur
The petitioner also relied on the affidavit of Ma. Rachelle Gonzales attesting that there was no election of officers, but respondent BLR Director dismissed
Calma (Calma) did not state that no election of officers took place; (5) at least 82 other members of YEU did not question the legality of YEU’s
the affidavit as nothing but the petitioner’s belated attempt to establish its claim about the election being held considering that Gonzales did not even
organization; and (6) 50 YEU members executed a Sama-Samang Pahayag9 stating that:
intimate such matter in her handwritten resignation letter to YEU.

3. Noong ika-25 ng Hulyo 1999, kami ay dumalo sa isang pulong para sa pag-oorganisa ng aming Unyon at pagraratipika ng Saligang Batas at
Another affidavit, that of Arthur Calma, stated that no election was held, but, again, respondent BLR Director gave Calma’s affidavit scant consideration
Alituntunin nito. x x x
because the affiant admittedly remained in the YEU office for only 20 minutes. In contrast, the public respondent accorded more weight to the sama-
xxxx
samang pahayag executed by 50 YEU members who averred about the holding of an organizational meeting. The public respondent justifiably favored
5. Walang katotohanan ang alegasyon ng Yokohama na walang naganap na pagpupulong kaugnay ng pag-oorganisa o pagtatayo namin ng
the latter, deeming the meeting to include the holding of an election of officers, for, after all, Art. 234, (b),Labor Code, does not itself distinguish between
Unyon. Nakakatuwa ring isipin ang alegasyon ng kompanya na hindi namin lubos na naiintindihan ang aming kapasyahang magtayo at sumapi sa aming
the two.
Unyon.

Respondent BLR Director is further assailed for not taking into consideration the affidavit asserting that no election of officers was ever conducted, which
6. Malinaw na ginagawa ng kompanya ang lahat ng paraan upang hadlangan ang aming karapatan sa pag-oorganisa at kilalanin bilang kinatawan ng
Bernardino David, YEU’s second vice president, executed. The omission is not serious enough, however, because the affidavit was submitted only when
lahat ng mga regular na manggagawa para sa sama-samang pakikipagtawaran.
the petitioner moved for the reconsideration of the questioned decision, and because the affidavit was even inconsistent with David’s earlier sinumpaang
salaysay, whereby he attested to his attendance at the organizational meeting and to his election thereat as vice president.
7. Sa kabila ng lahat ng ito, kami ay lubos pa ring naninindigan sa aming Unyon at patuloy na ipaglalaban ang aming karapatan sa pag-oorganisa at sa
sama-samang pakikipagtawaran;10
As to the inclusion of Pineda’s signature in the organizational documents, the BLR Director correctly ruled that evidence to prove the participation of YEU
in the failure to delete Pineda’s signature from the organizational documents was wanting. It is not deniable that Pineda never approached any officer of
The BLR also held that (1) YTPI was estopped from questioning the fact that the Sama-Samang Pahayag was an unsworn document since it filed the 24 YEU; and that Pineda approached a certain Tonton whom he knew to be a union organizer but who was not an officer of the union nor an employee of
January 2000 petition for the revocation of YEU’s registration based on unsworn documents; (2) the fact that there was no express mention of an election the company.
of union officers in the Sama-Samang Pahayag did not necessarily mean that no election occurred; (3) there was an organizational meeting and an
organizational meeting may include an election of union officers; (4) any infirmity in the election of union officers may be remedied under the last
If the petitioner was [sic] sincere and intent on this imputed error, its effort to show so does not [sic] appear in the record. What appears is its abject failure
paragraph11 of Article 241 of the Labor Code and under Rule XIV of DOLE Department Order No. 9; and (5) cancellation of union registration must be
to establish Tonton’s actual identity. The petitioner seemed content in making the insinuation in the petition for certiorari that Tonton was widely
done with great caution.
recognized as the organizer behind the creation of YEU. That was not enough.

YTPI filed before the BLR a motion12 for reconsideration. In its 3 May 2001 Resolution, the BLR denied the motion for lack of merit.
In sum, the BLR Director was neither capricious nor whimsical in his exercise of judgment, and, therefore, did not commit grave abuse of discretion.
For certiorari to lie, more than mere abuse of discretion is required to be established by the petitioner. Herein, no degree of abuse of discretion was
YTPI filed before the Court of Appeals a petition13 for certiorari under Rule 65 of the Rules of Court. In its 16 January 2004 Decision, the Court of Appeals attendant.15
denied the petition and held that the BLR did not commit grave abuse of discretion: (1) Pineda’s affidavit that no election of officers took place was
unreliable and inconsistent with his earlier written statement; (2) Gonzales’ affidavit that no election of officers took place was unreliable and inconsistent
YTPI claims that the Court of Appeals erred in finding that YEU did not commit fraud or misrepresentation. YTPI stated that:
with her earlier resignation letter; (3) Calma’s affidavit was unreliable because he admitted that he stayed at the organizational meeting for only 20
minutes; (4) the affidavit of a certain Bernardino David (David) that no election of officers took place was unreliable and inconsistent with his
earlier sinumpaang salaysay; (5) David’s affidavit was only filed before the BLR when YTPI filed its motion for reconsideration of the BLR’s 12 March There was evidence that respondent committed fraud and misrepresentation in its failure to omit the name of Ronald Pineda prior to the filing of the
2001 Resolution; (6) Pineda did not approach any officer of YEU to have his signature removed from the organizational documents; (7) the Sama- respondents organizational documents with the Department of Labor and Employment. On the other hand, the Regional Director held that there was
Samang Pahayag was entitled to credit even if it was an unsworn document; (8) the allegation that the signatures of a certain Denry Villanueva no election of officers that had taken place during respondent’s alleged organizational meeting as there was no proof of such
(Villanueva) and a certain Apolinar Bognot (Bognot) in the Sama-Samang Pahayag were forged was only raised for the first time before the BLR when election.16 (Emphasis in the original)
YTPI filed its motion for reconsideration of the BLR’s 12 March 2001 Resolution; (9) Villanueva and Bognot were not signatories to YEU’s organizational
documents; (10) cancellation of union registration must be done with great caution; (11) YTPI, in filing the petition for revocation of YEU’s registration, had
the burden of proving that YEU committed fraud and misrepresentation; and (12) YTPI failed to prove that YEU committed fraud and The Court is not convinced. A petition for review on certiorari under Rule 45 of the Rules of Court should include only questions of law — questions of fact
are not reviewable. A question of law exists when the doubt centers on what the law is on a certain set of facts, while a question of fact exists when the
misrepresentation.1avvphi1
doubt centers on the truth or falsity of the alleged facts. There is a question of law if the issue raised is capable of being resolved without need of
reviewing the probative value of the evidence. Once the issue invites a review of the evidence, the question is one of fact.17
YTPI filed before the Court of Appeals a motion14 for reconsideration. In its 12 May 2004 Resolution, the Court of Appeals denied the motion for lack of
merit.
Whether YEU committed fraud and misrepresentation in failing to remove Pineda’s signature from the list of employees who supported YEU’s application
for registration and whether YEU conducted an election of its officers are questions of fact. They are not reviewable.
Hence, the present petition. YTPI raises as issues that (1) the Court of Appeals erred in finding that YEU did not commit fraud or misrepresentation, and
(2) the Court of Appeals erred in holding that YTPI had the burden of proving that YEU committed fraud and misrepresentation.
Factual findings of the Court of Appeals are binding on the Court. Absent grave abuse of discretion, the Court will not disturb the Court of Appeals’ factual
findings.18 In Encarnacion v. Court of Appeals,19 the Court held that, "unless there is a clearly grave or whimsical abuse on its part, findings of fact of the
The petition is unmeritorious. appellate court will not be disturbed. The Supreme Court will only exercise its power of review in known exceptions such as gross misappreciation of
evidence or a total void of evidence." YTPI failed to show that the Court of Appeals gravely abused its discretion.
The Court of Appeals found that YEU did not commit fraud or misrepresentation:
The Court of Appeals held that YTPI had the burden of proving that YEU committed fraud and misrepresentation:
Anent whether an election of officers was conducted or not, the petitioner relied largely on the affidavit of Pineda to substantiate its claim that no election
of officers was held by the union. However, respondent BLR Director accorded greater credence to Pineda’s handwritten statement, wherein he made The cancellation of union registration at the employer’s instance, while permitted, must be approached with caution and strict scrutiny in order that the
references to at least 2 meetings he had attended during which he had signed the organizational documents, than to Pineda’s later affidavit, whereby he right to belong to a legitimate labor organization and to enjoy the privileges appurtenant to such membership will not be denied to the employees. As the
Labor Relations Set IV * Takata v BLR to Silang v Coa * Page 4 of 29

applicant for cancellation, the petitioner naturally had the burden to present proof sufficient to warrant the cancellation. The petitioner was thus expected On October 1, 2003, the petitioner Electromat Manufacturing and Recording Corporation (company) filed a petition for cancellation of the union’s
to satisfactorily establish that YEU committed misrepresentations, false statements or fraud in connection with the election of its officers, or with the registration certificate, for the union’s failure to comply with Article 234 of the Labor Code. It argued that D.O. 40-03 is an unconstitutional diminution of
minutes of the election of officers, or in the list of votes, as expressly required in Art. 239, (c), Labor Code. But, as the respondent BLR Director has found the Labor Code’s union registration requirements under Article 234.
and determined, and We fully agree with him, the petitioner simply failed to discharge its burden. 20
On November 27, 2003, Acting Director Ciriaco A. Lagunzad of the Department of Labor and Employment (DOLE)-National Capital Region dismissed the
YTPI claims that the Court of Appeals erred in holding that YTPI had the burden of proving that YEU committed fraud and misrepresentation. YTPI stated petition.5
that:
In the appeal by the company, BLR Director Hans Leo J. Cacdac affirmed the dismissal. 6 The company thereafter sought relief from the CA through a
5.5 In the Decision dated 16 January 2004, the Honorable Court of Appeals upheld the BLR Director’s ruling that the petitioner had the burden of proving petition for certiorari, contending that the BLR committed grave abuse of discretion in affirming the union’s registration despite its non-compliance with the
that subject election of officers never took place. requirements for registration under Article 234 of the Labor Code. It assailed the validity of D.O. 40-03 which amended the rules of Book V (Labor
Relations) of the Labor Code. It posited that the BLR should have strictly adhered to the union registration requirements under the Labor Code, instead of
relying on D.O. 40-03 which it considered as an invalid amendment of the law since it reduced the requirements under Article 234 of the Labor Code. It
5.6 However, the petitioner does not have the burden of proof vis-à-vis whether or not the said elections took place. The respondent has the burden of
maintained that the BLR should not have granted the union’s registration through the issuance of a Certification of Creation of Local Chapter since the
proof in showing that an election of officers took place.21(Emphasis in the original)
union submitted only the Charter Certificate issued to it by WASTO.

The Court is not convinced. YTPI, being the one which filed the petition for the revocation of YEU’s registration, had the burden of proving that YEU
The CA Decision
committed fraud and misrepresentation. YTPI had the burden of proving the truthfulness of its accusations — that YEU fraudulently failed to remove
Pineda’s signature from the organizational documents and that YEU fraudulently misrepresented that it conducted an election of officers.
In its decision rendered on February 3, 2006,7 the CA Tenth Division dismissed the petition and affirmed the assailed BLR ruling. It brushed aside the
company’s objection to D.O. 40-03, and its submission that D.O. 40-03 removed the safety measures against the commission of fraud in the registration
In Heritage Hotel Manila v. Pinag-Isang Galing at Lakas ng mga Manggagawa sa Heritage Manila,22 the employer filed a petition to revoke the registration
of unions. It noted that "there are sufficient safeguards found in other provisions of the Labor Code to prevent the same." 8 In any event, it pointed out that
of its rank-and-file employees’ union, accusing it of committing fraud and misrepresentation. The Court held that the petition was rightfully denied because
D.O. 40-03 was issued by the DOLE pursuant to its rule-making power under the law.9
the employer failed to prove that the labor union committed fraud and misrepresentation. The Court held that:

The company moved for reconsideration, arguing that the union’s registration certificate was invalid as there was no showing that WASTO, the labor
Did respondent PIGLAS union commit fraud and misrepresentation in its application for union registration? We agree with the DOLE-NCR and the BLR
federation to which the union is affiliated, had at least ten (10) locals or chapters as required by D.O. 40-03. The CA denied the motion,10 holding that no
that it did not. Except for the evident discrepancies as to the number of union members involved as these appeared on the documents that supported the
such requirement is found under the rules. Hence, the present petition.
union’s application for registration, petitioner company has no other evidence of the alleged misrepresentation. But those discrepancies alone
cannot be taken as an indication that respondent misrepresented the information contained in these documents.
The Case for the Petitioner
The charge that a labor organization committed fraud and misrepresentation in securing its registration is a serious charge and deserves
close scrutiny. It is serious because once such charge is proved, the labor union acquires none of the rights accorded to registered The company seeks a reversal of the CA rulings, through its submissions (the petition11 and the memorandum12), on the ground that the CA seriously
organizations. Consequently, charges of this nature should be clearly established by evidence and the surrounding circumstances.23 (Emphasis erred and gravely abused its discretion in affirming the registration of the union in accordance with D.O. 40-03. Specifically, it assails as unconstitutional
supplied) Section 2(E), Rule III of D.O. 40-03 which provides:

WHEREFORE, we DENY the petition. We AFFIRM the 16 January 2004 Decision and 12 May 2004 Resolution of the Court of Appeals in CA-G.R. SP The report of creation of a chartered local shall be accompanied by a charter certificate issued by the federation or national union indicating the creation
No. 65460.SO ORDERED. or establishment of the chartered local.

Republic of the Philippines The company points out that D.O. 40-03 delisted some of the requirements under Article 234 of the Labor Code for the registration of a local chapter.
SUPREME COURT Article 234 states:
Manila
SECOND DIVISION
ART. 234. Requirements of Registration.13 Any applicant labor organization, association or group of unions or workers shall acquire legal personality and
G.R. No. 172699 July 27, 2011
shall be entitled to the rights and privileges granted by law to legitimate labor organizations upon issuance of the certificate of registration based on the
ELECTROMAT MANUFACTURING and RECORDING CORPORATION, Petitioner,
following requirements:
vs.
HON. CIRIACO LAGUNZAD, in his capacity as Regional Director, National Capital Region, Department of Labor and Employment; and HON.
HANS LEO J. CACDAC, in his capacity as Director of Bureau of Labor Relations, Department of Labor and Employment, Public Respondents. (a) Fifty pesos (P50.00) registration fee;
NAGKAKAISANG SAMAHAN NG MANGGAGAWA NG ELECTROMAT-WASTO, Private Respondent.
DECISION
BRION, J.: (b) The names of its officers, their addresses, the principal address of the labor organization, the minutes of the organizational meetings and
the list of the workers who participated in such meetings;
We resolve the present petition for review on certiorari1 assailing the decision2 and the resolution3 of the Court of Appeals (CA) dated February 3, 2006
and May 11, 2006, respectively, rendered in CA G.R. SP No. 83847. (c) The names of all its members comprising at least twenty percent (20%) of all the employees in the bargaining unit where it seeks to operate;

The Antecedents (d) If the applicant union has been in existence for one or more years, copies of its annual financial reports; and

The private respondent Nagkakaisang Samahan ng Manggagawa ng Electromat-Wasto (union), a charter affiliate of the Workers Advocates for Struggle, (e) Four (4) copies of the constitution and by-laws of the applicant union, minutes of its adoption or ratification, and the list of the members who
Transformation and Organization (WASTO), applied for registration with the Bureau of Labor Relations (BLR). Supporting the application were the participated in it.
following documents: (1) copies of its ratified constitution and by-laws (CBL); (2) minutes of the CBL’s adoption and ratification; (3) minutes of the
organizational meetings; (4) names and addresses of the union officers; (5) list of union members; (6) list of rank-and-file employees in the company; (7)
certification of non-existence of a collective bargaining agreement (CBA) in the company; (8) resolution of affiliation with WASTO, a labor federation; (9) The company contends that the enumeration of the requirements for union registration under the law is exclusive and should not be diminished, and that
WASTO’s resolution of acceptance; (10) Charter Certificate; and (11) Verification under oath. the same requirements should apply to all labor unions whether they be independent labor organizations, federations or local chapters. It adds that in
making a different rule for local chapters, D.O. 40-03 expanded or amended Article 234 of the Labor Code, resulting in an invalid exercise by the DOLE of
its delegated rule-making power. It thus posits that the union’s certificate of registration which was issued "in violation of the letters of Article 234 of the
The BLR thereafter issued the union a Certification of Creation of Local Chapter (equivalent to the certificate of registration of an independent union), Labor Code"14 is void and of no effect, and that the CA committed grave abuse of discretion when it affirmed the union’s existence.
pursuant to Department Order No. (D.O.) 40-03.4
The Case for the Union
Labor Relations Set IV * Takata v BLR to Silang v Coa * Page 5 of 29

In a Resolution dated January 16, 2008,15 the Court directed union board member Alex Espejo, in lieu of union President Roberto Beltran whose present names and addresses of the union officers; (5) the list of union members; (6) the list of rank-and-file employees in the company; (7) a certification of non-
address could not be verified, to furnish the Court a copy of the union comment/opposition to the company’s motion for reconsideration dated February existence of a CBA in the company; (8) the resolution of affiliation with WASTO and the latter’s acceptance; and (9) their Charter Certificate. These
22, 2006 in CA G.R. SP No. 83847, which the union adopted as its comment on the present petition. 16 submissions were properly verified as required by the rules. In sum, the petitioner has no factual basis for questioning the union’s registration, as even
the requirements for registration as an independent local have been substantially complied with.
Through this comment/opposition,17 the union submits that the company failed to show that the CA committed reversible error in upholding the
registration certificate issued to it by the BLR. Citing Castillo v. National Labor Relations Commission, 18 it stressed that the issuance of the certificate by We, thus, find no compelling justification to nullify D.O. 40-03. Significantly, the Court declared in another case:24
the DOLE agencies was supported by substantial evidence, which should be entitled to great respect and even finality.
Pagpalain cannot also allege that Department Order No. 9 is violative of public policy. x x x [T]he sole function of our courts is to apply or interpret the
The Court’s Ruling laws. It does not formulate public policy, which is the province of the legislative and executive branches of government. It cannot, thus, be said that the
principles laid down by the Court in Progressive and Protection Technology constitute public policy on the matter. They do, however, constitute the
Court’s interpretation of public policy, as formulated by the executive department through its promulgation of rules implementing the Labor Code.
We resolve the core issue of whether D.O. 40-03 is a valid exercise of the rule-making power of the DOLE.
However, this public policy has itself been changed by the executive department, through the amendments introduced in Book V of the Omnibus Rules by
Department Order No. 9. It is not for us to question this change in policy, it being a well-established principle beyond question that it is not within the
We rule in the affirmative. Earlier in Progressive Development Corporation v. Secretary, Department of Labor and Employment, 19 the Court encountered a province of the courts to pass judgments upon the policy of legislative or executive action.
similar question on the validity of the old Section 3, Rule II, Book V of the Rules Implementing the Labor Code 20 which stated:
This statement is as true then as it is now.
Union affiliation; direct membership with a national union. - The affiliate of a labor federation or national union may be a local or chapter thereof or an
independently registered union.
In light of the foregoing, we find no merit in the appeal.

a) The labor federation or national union concerned shall issue a charter certificate indicating the creation or establishment of a local or chapter, copy of
WHEREFORE, premises considered, we DENY the petition for lack of merit. The assailed decision and resolution of the Court of Appeals are
which shall be submitted to the Bureau of Labor Relations within thirty (30) days from issuance of such charter certificate.
AFFIRMED. Costs against the petitioner Electromat Manufacturing and Recording Corporation.SO ORDERED.
Republic of the Philippines
xxxx SUPREME COURT
Manila
SECOND DIVISION
e) The local or chapter of a labor federation or national union shall have and maintain a constitution and by-laws, set of officers and books of accounts. G.R. No. 178296 January 12, 2011
For reporting purposes, the procedure governing the reporting of independently registered unions, federations or national unions shall be observed. THE HERITAGE HOTEL MANILA, acting through its owner, GRAND PLAZA HOTEL CORPORATION,Petitioner,
vs.
Interpreting these provisions of the old rules, the Court said that by force of law, 21 the local or chapter of a labor federation or national union becomes a NATIONAL UNION OF WORKERS IN THE HOTEL, RESTAURANT AND ALLIED INDUSTRIES-HERITAGE HOTEL MANILA SUPERVISORS
legitimate labor organization upon compliance with Section 3, Rule II, Book V of the Rules Implementing the Labor Code, the only requirement being the CHAPTER (NUWHRAIN-HHMSC), Respondent.
submission of the charter certificate to the BLR. Further, the Court noted that Section 3 omitted several requirements which are otherwise required for DECISION
union registration, as follows: NACHURA, J.:

Before the Court is a petition for review on certiorari of the Decision1 of the Court of Appeals (CA) dated May 30, 2005 and Resolution dated June 4,
1) The requirement that the application for registration must be signed by at least 20% of the employees in the appropriate bargaining unit; 2007. The assailed Decision affirmed the dismissal of a petition for cancellation of union registration filed by petitioner, Grand Plaza Hotel Corporation,
owner of Heritage Hotel Manila, against respondent, National Union of Workers in the Hotel, Restaurant and Allied Industries-Heritage Hotel Manila
2) The submission of officers’ addresses, principal address of the labor organization, the minutes of organization meetings and the list of the Supervisors Chapter (NUWHRAIN-HHMSC), a labor organization of the supervisory employees of Heritage Hotel Manila.
workers who participated in such meetings;
The case stemmed from the following antecedents:
3) The submission of the minutes of the adoption or ratification of the constitution and by-laws and the list of the members who participated in
it.22 On October 11, 1995, respondent filed with the Department of Labor and Employment-National Capital Region (DOLE-NCR) a petition for certification
election.2 The Med-Arbiter granted the petition on February 14, 1996 and ordered the holding of a certification election. 3 On appeal, the DOLE Secretary,
Notwithstanding these omissions, the Court upheld the government’s implementing policy expressed in the old rules when it declared in Progressive in a Resolution dated August 15, 1996, affirmed the Med-Arbiter’s order and remanded the case to the Med-Arbiter for the holding of a preelection
Development – conference on February 26, 1997. Petitioner filed a motion for reconsideration, but it was denied on September 23, 1996.

Undoubtedly, the intent of the law in imposing lesser requirements in the case of a branch or local of a registered federation or national union is to The preelection conference was not held as initially scheduled; it was held a year later, or on February 20, 1998. Petitioner moved to archive or to dismiss
encourage the affiliation of a local union with a federation or national union in order to increase the local union’s bargaining powers respecting terms and the petition due to alleged repeated non-appearance of respondent. The latter agreed to suspend proceedings until further notice. The preelection
conditions of labor.23 conference resumed on January 29, 2000.

It was this same Section 3 of the old rules that D.O. 40-03 fine-tuned when the DOLE amended the rules on Book V of the Labor Code, thereby modifying Subsequently, petitioner discovered that respondent had failed to submit to the Bureau of Labor Relations (BLR) its annual financial report for several
the government’s implementing policy on the registration of locals or chapters of labor federations or national unions. The company now assails this years and the list of its members since it filed its registration papers in 1995. Consequently, on May 19, 2000, petitioner filed a Petition for Cancellation of
particular amendment as an invalid exercise of the DOLE’s rule-making power. Registration of respondent, on the ground of the non-submission of the said documents. Petitioner prayed that respondent’s Certificate of Creation of
Local/Chapter be cancelled and its name be deleted from the list of legitimate labor organizations. It further requested the suspension of the certification
election proceedings.4
We disagree. As in the case of D.O. 9 (which introduced the above-cited Section 3 of the old rules) in Progressive Development, D.O. 40-03 represents
an expression of the government’s implementing policy on trade unionism. It builds upon the old rules by further simplifying the requirements for the
establishment of locals or chapters. As in D.O. 9, we see nothing contrary to the law or the Constitution in the adoption by the Secretary of Labor and On June 1, 2000, petitioner reiterated its request by filing a Motion to Dismiss or Suspend the [Certification Election] Proceedings, 5 arguing that the
Employment of D.O. 40-03 as this department order is consistent with the intent of the government to encourage the affiliation of a local union with a dismissal or suspension of the proceedings is warranted, considering that the legitimacy of respondent is seriously being challenged in the petition for
federation or national union to enhance the local’s bargaining power. If changes were made at all, these were those made to recognize the distinctions cancellation of registration. Petitioner maintained that the resolution of the issue of whether respondent is a legitimate labor organization is crucial to the
made in the law itself between federations and their local chapters, and independent unions; local chapters seemingly have lesser requirements because issue of whether it may exercise rights of a legitimate labor organization, which include the right to be certified as the bargaining agent of the covered
they and their members are deemed to be direct members of the federation to which they are affiliated, which federations are the ones subject to the strict employees.
registration requirements of the law.1avvphi1
Nevertheless, the certification election pushed through on June 23, 2000. Respondent emerged as the winner. 6
In any case, the local union in the present case has more than satisfied the requirements the petitioner complains about; specifically, the union has
submitted: (1) copies of the ratified CBL; (2) the minutes of the CBL’s adoption and ratification; (3) the minutes of the organizational meetings; (4) the
Labor Relations Set IV * Takata v BLR to Silang v Coa * Page 6 of 29

On June 28, 2000, petitioner filed a Protest with Motion to Defer Certification of Election Results and Winner, 7stating that the certification election held on In its Resolution23 dated June 4, 2007, the CA denied petitioner’s motion, stating that the BLR Director’s inhibition from the case was a peculiarity not
June 23, 2000 was an exercise in futility because, once respondent’s registration is cancelled, it would no longer be entitled to be certified as the present in the Abbott case, and that such inhibition justified the assumption of jurisdiction by the DOLE Secretary.
exclusive bargaining agent of the supervisory employees. Petitioner also claimed that some of respondent’s members were not qualified to join the union
because they were either confidential employees or managerial employees. It then prayed that the certification of the election results and winner be
In this petition, petitioner argues that:
deferred until the petition for cancellation shall have been resolved, and that respondent’s members who held confidential or managerial positions be
excluded from the supervisors’ bargaining unit.
I.
8
Meanwhile, respondent filed its Answer to the petition for the cancellation of its registration. It averred that the petition was filed primarily to delay the
conduct of the certification election, the respondent’s certification as the exclusive bargaining representative of the supervisory employees, and the The Court of Appeals seriously erred in ruling that the Labor Secretary properly assumed jurisdiction over Petitioner’s appeal of the Regional Director’s
commencement of bargaining negotiations. Respondent prayed for the dismissal of the petition for the following reasons: (a) petitioner is estopped from Decision in the Cancellation Petition x x x.
questioning respondent’s status as a legitimate labor organization as it had already recognized respondent as such during the preelection conferences;
(b) petitioner is not the party-in-interest, as the union members are the ones who would be disadvantaged by the non-submission of financial reports; (c) it
has already complied with the reportorial requirements, having submitted its financial statements for 1996, 1997, 1998, and 1999, its updated list of A. Jurisdiction is conferred only by law. The Labor Secretary had no jurisdiction to review the decision of the Regional Director in a petition for
cancellation. Such jurisdiction is conferred by law to the BLR.
officers, and its list of members for the years 1995, 1996, 1997, 1998, and 1999; (d) the petition is already moot and academic, considering that the
certification election had already been held, and the members had manifested their will to be represented by respondent. B. The unilateral inhibition by the BLR Director cannot justify the Labor Secretary’s exercise of jurisdiction over the Appeal.
C. The Labor Secretary’s assumption of jurisdiction over the Appeal without notice violated Petitioner’s right to due process.

Citing National Union of Bank Employees v. Minister of Labor, et al. 9 and Samahan ng Manggagawa sa Pacific Plastic v. Hon. Laguesma, 10 the Med-
II.
Arbiter held that the pendency of a petition for cancellation of registration is not a bar to the holding of a certification election. Thus, in an Order11 dated
January 26, 2001, the Med-Arbiter dismissed petitioner’s protest, and certified respondent as the sole and exclusive bargaining agent of all supervisory
employees. The Court of Appeals gravely erred in affirming the dismissal of the Cancellation Petition despite the mandatory and unequivocal provisions of the Labor
Code and its Implementing Rules.24
Petitioner subsequently appealed the said Order to the DOLE Secretary.12 The appeal was later dismissed by DOLE Secretary Patricia A. Sto. Tomas
(DOLE Secretary Sto. Tomas) in the Resolution of August 21, 2002.13Petitioner moved for reconsideration, but the motion was also denied. 14 The petition has no merit.

In the meantime, Regional Director Alex E. Maraan (Regional Director Maraan) of DOLE-NCR finally resolved the petition for cancellation of registration. Jurisdiction to review the decision of the Regional Director lies with the BLR. This is clearly provided in the Implementing Rules of the Labor Code and
While finding that respondent had indeed failed to file financial reports and the list of its members for several years, he, nonetheless, denied the petition, enunciated by the Court in Abbott. But as pointed out by the CA, the present case involves a peculiar circumstance that was not present or covered by
ratiocinating that freedom of association and the employees’ right to self-organization are more substantive considerations. He took into account the fact the ruling in Abbott. In this case, the BLR Director inhibited himself from the case because he was a former counsel of respondent. Who, then, shall
that respondent won the certification election and that it had already been certified as the exclusive bargaining agent of the supervisory employees. In resolve the case in his place?
view of the foregoing, Regional Director Maraan—while emphasizing that the non-compliance with the law is not viewed with favor—considered the
belated submission of the annual financial reports and the list of members as sufficient compliance thereof and considered them as having been
submitted on time. The dispositive portion of the decision15 dated December 29, 2001 reads: In Abbott, the appeal from the Regional Director’s decision was directly filed with the Office of the DOLE Secretary, and we ruled that the latter has no
appellate jurisdiction. In the instant case, the appeal was filed by petitioner with the BLR, which, undisputedly, acquired jurisdiction over the case. Once
jurisdiction is acquired by the court, it remains with it until the full termination of the case. 25
WHEREFORE, premises considered, the instant petition to delist the National Union of Workers in the Hotel, Restaurant and Allied Industries-Heritage
Hotel Manila Supervisors Chapter from the roll of legitimate labor organizations is hereby DENIED.SO ORDERED. 16
Thus, jurisdiction remained with the BLR despite the BLR Director’s inhibition. When the DOLE Secretary resolved the appeal, she merely stepped into
the shoes of the BLR Director and performed a function that the latter could not himself perform. She did so pursuant to her power of supervision and
17
Aggrieved, petitioner appealed the decision to the BLR. BLR Director Hans Leo Cacdac inhibited himself from the case because he had been a former control over the BLR.26
counsel of respondent.
Expounding on the extent of the power of control, the Court, in Araneta, et al. v. Hon. M. Gatmaitan, et al., 27pronounced that, if a certain power or
In view of Director Cacdac’s inhibition, DOLE Secretary Sto. Tomas took cognizance of the appeal. In a resolution18 dated February 21, 2003, she authority is vested by law upon the Department Secretary, then such power or authority may be exercised directly by the President, who exercises
dismissed the appeal, holding that the constitutionally guaranteed freedom of association and right of workers to self-organization outweighed supervision and control over the departments. This principle was incorporated in the Administrative Code of 1987, which defines "supervision and control"
respondent’s noncompliance with the statutory requirements to maintain its status as a legitimate labor organization. as including the authority to act directly whenever a specific function is entrusted by law or regulation to a subordinate. 28 Applying the foregoing to the
present case, it is clear that the DOLE Secretary, as the person exercising the power of supervision and control over the BLR, has the authority to directly
exercise the quasi-judicial function entrusted by law to the BLR Director.
Petitioner filed a motion for reconsideration,19 but the motion was likewise denied in a resolution20 dated May 30, 2003. DOLE Secretary Sto. Tomas
admitted that it was the BLR which had jurisdiction over the appeal, but she pointed out that the BLR Director had voluntarily inhibited himself from the
case because he used to appear as counsel for respondent. In order to maintain the integrity of the decision and of the BLR, she therefore accepted the It is true that the power of control and supervision does not give the Department Secretary unbridled authority to take over the functions of his or her
motion to inhibit and took cognizance of the appeal. subordinate. Such authority is subject to certain guidelines which are stated in Book IV, Chapter 8, Section 39(1)(a) of the Administrative Code of
1987.29 However, in the present case, the DOLE Secretary’s act of taking over the function of the BLR Director was warranted and necessitated by the
Petitioner filed a petition for certiorari with the CA, raising the issue of whether the DOLE Secretary acted with grave abuse of discretion in taking latter’s inhibition from the case and the objective to "maintain the integrity of the decision, as well as the Bureau itself."30
cognizance of the appeal and affirming the dismissal of its petition for cancellation of respondent’s registration.
Petitioner insists that the BLR Director’s subordinates should have resolved the appeal, citing the provision under the Administrative Code of 1987 which
In a Decision dated May 30, 2005, the CA denied the petition. The CA opined that the DOLE Secretary may legally assume jurisdiction over an appeal states, "in case of the absence or disability of the head of a bureau or office, his duties shall be performed by the assistant head." 31 The provision clearly
does not apply considering that the BLR Director was neither absent nor suffering from any disability; he remained as head of the BLR. Thus, to dispel
from the decision of the Regional Director in the event that the Director of the BLR inhibits himself from the case. According to the CA, in the absence of
the BLR Director, there is no person more competent to resolve the appeal than the DOLE Secretary. The CA brushed aside the allegation of bias and any suspicion of bias, the DOLE Secretary opted to resolve the appeal herself.
partiality on the part of the DOLE Secretary, considering that such allegation was not supported by any evidence.
Petitioner was not denied the right to due process when it was not notified in advance of the BLR Director’s inhibition and the DOLE Secretary’s
The CA also found that the DOLE Secretary did not commit grave abuse of discretion when she affirmed the dismissal of the petition for cancellation of assumption of the case. Well-settled is the rule that the essence of due process is simply an opportunity to be heard, or, as applied to administrative
respondent’s registration as a labor organization. Echoing the DOLE Secretary, the CA held that the requirements of registration of labor organizations proceedings, an opportunity to explain one’s side or an opportunity to seek a reconsideration of the action or ruling complained of. 32 Petitioner had the
opportunity to question the BLR Director’s inhibition and the DOLE Secretary’s taking cognizance of the case when it filed a motion for reconsideration of
are an exercise of the overriding police power of the State, designed for the protection of workers against potential abuse by the union that recruits them.
These requirements, the CA opined, should not be exploited to work against the workers’ constitutionally protected right to self-organization. the latter’s decision. It would be well to state that a critical component of due process is a hearing before an impartial and disinterested tribunal, for all the
elements of due process, like notice and hearing, would be meaningless if the ultimate decision would come from a partial and biased judge.33 It was
precisely to ensure a fair trial that moved the BLR Director to inhibit himself from the case and the DOLE Secretary to take over his function.
Petitioner filed a motion for reconsideration, invoking this Court’s ruling in Abbott Labs. Phils., Inc. v. Abbott Labs. Employees Union,21 which categorically
declared that the DOLE Secretary has no authority to review the decision of the Regional Director in a petition for cancellation of union registration, and
Section 4,22 Rule VIII, Book V of the Omnibus Rules Implementing the Labor Code. Petitioner also insists that respondent’s registration as a legitimate labor union should be cancelled. Petitioner posits that once it is determined that a
ground enumerated in Article 239 of the Labor Code is present, cancellation of registration should follow; it becomes the ministerial duty of the Regional
Labor Relations Set IV * Takata v BLR to Silang v Coa * Page 7 of 29

Director to cancel the registration of the labor organization, hence, the use of the word "shall." Petitioner points out that the Regional Director has (a) Its constitution and by-laws, or amendments thereto, the minutes of ratification, and the list of members who took part in the ratification of
admitted in its decision that respondent failed to submit the required documents for a number of years; therefore, cancellation of its registration should the constitution and by-laws within thirty (30) days from adoption or ratification of the constitution and by-laws or amendments thereto;
have followed as a matter of course.
(b) Its list of officers, minutes of the election of officers, and list of voters within thirty (30) days from election;
We are not persuaded.
(c) Its annual financial report within thirty (30) days after the close of every fiscal year; and
Articles 238 and 239 of the Labor Code read:
(d) Its list of members at least once a year or whenever required by the Bureau.
ART. 238. CANCELLATION OF REGISTRATION; APPEAL
Failure to comply with the above requirements shall not be a ground for cancellation of union registration but shall subject the erring officers or members
The certificate of registration of any legitimate labor organization, whether national or local, shall be canceled by the Bureau if it has reason to believe, to suspension, expulsion from membership, or any appropriate penalty.
after due hearing, that the said labor organization no longer meets one or more of the requirements herein prescribed. 34
ILO Convention No. 87, which we have ratified in 1953, provides that "workers’ and employers’ organizations shall not be liable to be dissolved or
ART. 239. GROUNDS FOR CANCELLATION OF UNION REGISTRATION. suspended by administrative authority." The ILO has expressed the opinion that the cancellation of union registration by the registrar of labor unions,
which in our case is the BLR, is tantamount to dissolution of the organization by administrative authority when such measure would give rise to the loss of
legal personality of the union or loss of advantages necessary for it to carry out its activities, which is true in our jurisdiction. Although the ILO has allowed
The following shall constitute grounds for cancellation of union registration:
such measure to be taken, provided that judicial safeguards are in place, i.e., the right to appeal to a judicial body, it has nonetheless reminded its
xxxx
members that dissolution of a union, and cancellation of registration for that matter, involve serious consequences for occupational representation. It has,
(d) Failure to submit the annual financial report to the Bureau within thirty (30) days after the closing of every fiscal year and misrepresentation, false
therefore, deemed it preferable if such actions were to be taken only as a last resort and after exhausting other possibilities with less serious effects on
entries or fraud in the preparation of the financial report itself;
the organization.40
xxxx
(i) Failure to submit list of individual members to the Bureau once a year or whenever required by the Bureau.35
The aforesaid amendments and the ILO’s opinion on this matter serve to fortify our ruling in this case. We therefore quote with approval the DOLE
Secretary’s rationale for denying the petition, thus:
These provisions give the Regional Director ample discretion in dealing with a petition for cancellation of a union’s registration, particularly, determining
whether the union still meets the requirements prescribed by law. It is sufficient to give the Regional Director license to treat the late filing of required
documents as sufficient compliance with the requirements of the law. After all, the law requires the labor organization to submit the annual financial report It is undisputed that appellee failed to submit its annual financial reports and list of individual members in accordance with Article 239 of the Labor Code.
and list of members in order to verify if it is still viable and financially sustainable as an organization so as to protect the employer and employees from However, the existence of this ground should not necessarily lead to the cancellation of union registration. Article 239 recognizes the regulatory authority
fraudulent or fly-by-night unions. With the submission of the required documents by respondent, the purpose of the law has been achieved, though of the State to exact compliance with reporting requirements. Yet there is more at stake in this case than merely monitoring union activities and requiring
belatedly. periodic documentation thereof.

We cannot ascribe abuse of discretion to the Regional Director and the DOLE Secretary in denying the petition for cancellation of respondent’s The more substantive considerations involve the constitutionally guaranteed freedom of association and right of workers to self-organization. Also
registration. The union members and, in fact, all the employees belonging to the appropriate bargaining unit should not be deprived of a bargaining agent, involved is the public policy to promote free trade unionism and collective bargaining as instruments of industrial peace and democracy.1avvphi1 An
merely because of the negligence of the union officers who were responsible for the submission of the documents to the BLR. overly stringent interpretation of the statute governing cancellation of union registration without regard to surrounding circumstances cannot be allowed.
Otherwise, it would lead to an unconstitutional application of the statute and emasculation of public policy objectives. Worse, it can render nugatory the
protection to labor and social justice clauses that pervades the Constitution and the Labor Code.
Labor authorities should, indeed, act with circumspection in treating petitions for cancellation of union registration, lest they be accused of interfering with
union activities. In resolving the petition, consideration must be taken of the fundamental rights guaranteed by Article XIII, Section 3 of the Constitution,
i.e., the rights of all workers to self-organization, collective bargaining and negotiations, and peaceful concerted activities. Labor authorities should bear in Moreover, submission of the required documents is the duty of the officers of the union. It would be unreasonable for this Office to order the cancellation
mind that registration confers upon a union the status of legitimacy and the concomitant right and privileges granted by law to a legitimate labor of the union and penalize the entire union membership on the basis of the negligence of its officers. In National Union of Bank Employees vs. Minister of
organization, particularly the right to participate in or ask for certification election in a bargaining unit.36 Thus, the cancellation of a certificate of registration Labor, L-53406, 14 December 1981, 110 SCRA 296, the Supreme Court ruled:
is the equivalent of snuffing out the life of a labor organization. For without such registration, it loses - as a rule - its rights under the Labor Code.37
As aptly ruled by respondent Bureau of Labor Relations Director Noriel: "The rights of workers to self-organization finds general and specific constitutional
It is worth mentioning that the Labor Code’s provisions on cancellation of union registration and on reportorial requirements have been recently amended guarantees. x x x Such constitutional guarantees should not be lightly taken much less nullified. A healthy respect for the freedom of association
by Republic Act (R.A.) No. 9481, An Act Strengthening the Workers’ Constitutional Right to Self-Organization, Amending for the Purpose Presidential demands that acts imputable to officers or members be not easily visited with capital punishments against the association itself."
Decree No. 442, As Amended, Otherwise Known as the Labor Code of the Philippines, which lapsed into law on May 25, 2007 and became effective on
June 14, 2007. The amendment sought to strengthen the workers’ right to self-organization and enhance the Philippines’ compliance with its international
At any rate, we note that on 19 May 2000, appellee had submitted its financial statement for the years 1996-1999. With this submission, appellee has
obligations as embodied in the International Labour Organization (ILO) Convention No. 87, 38 pertaining to the non-dissolution of workers’ organizations by
substantially complied with its duty to submit its financial report for the said period. To rule differently would be to preclude the union, after having failed to
administrative authority.39 Thus, R.A. No. 9481 amended Article 239 to read:
meet its periodic obligations promptly, from taking appropriate measures to correct its omissions. For the record, we do not view with favor appellee’s late
submission. Punctuality on the part of the union and its officers could have prevented this petition. 41
ART. 239. Grounds for Cancellation of Union Registration.—The following may constitute grounds for cancellation of union registration:
WHEREFORE, premises considered, the Court of Appeals Decision dated May 30, 2005 and Resolution dated June 4, 2007 are AFFIRMED.SO
(a) Misrepresentation, false statement or fraud in connection with the adoption or ratification of the constitution and by-laws or amendments ORDERED.
thereto, the minutes of ratification, and the list of members who took part in the ratification;
Republic of the Philippines
(b) Misrepresentation, false statements or fraud in connection with the election of officers, minutes of the election of officers, and the list of SUPREME COURT
voters; Manila
THIRD DIVISION
G.R. No. 190515 November 15, 2010
(c) Voluntary dissolution by the members.
CIRTEK EMPLOYEES LABOR UNION-FEDERATION OF FREE WORKERS, Petitioner,
vs.
R.A. No. 9481 also inserted in the Labor Code Article 242-A, which provides: CIRTEK ELECTRONICS, INC., Respondent.
DECISION
CARPIO MORALES, J.:
ART. 242-A. Reportorial Requirements.—The following are documents required to be submitted to the Bureau by the legitimate labor organization
concerned:
Labor Relations Set IV * Takata v BLR to Silang v Coa * Page 8 of 29

Cirtek Electronics, Inc. (respondent), an electronics and semi-conductor firm situated inside the Laguna Technopark, had an existing Collective The appellate court’s brushing aside of the "Paliwanag" and the minutes of the meeting that resulted in the conclusion of the MOA because they were not
Bargaining Agreement (CBA) with Cirtek Employees Labor Union-Federation of Free Workers (petitioner) for the period January 1, 2001 up to December verified and notarized, thus violating, so the appellate court reasoned, the rules on parol evidence, does not lie. Like any other rule on evidence, parol
31, 2005. Prior to the 3rd year of the CBA, the parties renegotiated its economic provisions but failed to reach a settlement, particularly on the issue of evidence should not be strictly applied in labor cases.
wage increases. Petitioner thereupon declared a bargaining deadlock and filed a Notice of Strike with the National Conciliation and Mediation Board-
Regional Office No. IV (NCMB-RO IV) on April 26, 2004. Respondent, upon the other hand, filed a Notice of Lockout on June 16, 2004.
The reliance on the parol evidence rule is misplaced. In labor cases pending before the Commission or the Labor Arbiter, the rules of evidence prevailing
in courts of law or equity are not controlling. Rules of procedure and evidence are not applied in a very rigid and technical sense in labor cases. Hence,
While the conciliation proceedings were ongoing, respondent placed seven union officers including the President, a Vice President, the Secretary and the the Labor Arbiter is not precluded from accepting and evaluating evidence other than, and even contrary to, what is stated in the CBA.16(emphasis
Chairman of the Board of Directors under preventive suspension for allegedly spearheading a boycott of overtime work. The officers were eventually supplied)
dismissed from employment, prompting petitioner to file another Notice of Strike which was, after conciliation meetings, converted to a voluntary
arbitration case. The dismissal of the officers was later found to be legal, hence, petitioner appealed.
While a contract constitutes the law between the parties, this is so in the present case with respect to the CBA, not to the MOA in which even the union’s
signatories had expressed reservations thereto. But even assuming arguendo that the MOA is treated as a new CBA, since it is imbued with public
In the meantime, as amicable settlement of the CBA was deadlocked, petitioner went on strike on June 20, 2005. By Order1 dated June 23, 2005, the interest, it must be construed liberally and yield to the common good.
Secretary of Labor assumed jurisdiction over the controversy and issued a Return to Work Order which was complied with.
While the terms and conditions of a CBA constitute the law between the parties, it is not, however, an ordinary contract to which is applied the principles
Before the Secretary of Labor could rule on the controversy, respondent created a Labor Management Council (LMC) through which it concluded with the of law governing ordinary contracts. A CBA, as a labor contract within the contemplation of Article 1700 of the Civil Code of the Philippines which governs
remaining officers of petitioner a Memorandum of Agreement (MOA)2providing for daily wage increases of P6.00 per day effective January 1, 2004 the relations between labor and capital, is not merely contractual in nature but impressed with public interest, thus, it must yield to the common good. As
and P9.00 per day effective January 1, 2005. Petitioner submitted the MOA via Motion and Manifestation 3 to the Secretary of Labor, alleging that the such, it must be construed liberally rather than narrowly and technically, and the courts must place a practical and realistic construction upon it, giving due
remaining officers signed the MOA under respondent’s assurance that should the Secretary order a higher award of wage increase, respondent would consideration to the context in which it is negotiated and purpose which it is intended to serve. 17 (emphasis and underscoring supplied)
comply.
WHEREFORE, the petition is GRANTED. The Decision dated September 24, 2009 and the Resolution dated December 2, 2009 of the Court of Appeals
By Order4 dated March 16, 2006, the Secretary of Labor resolved the CBA deadlock by awarding a wage increase of from P6.00 to P10.00 per day are REVERSED and SET ASIDE and the Order dated March 16, 2006 and Resolution dated August 12, 2008 of the Secretary of Labor are
effective January 1, 2004 and from P9.00 to P15.00 per day effective January 1, 2005, and adopting all other benefits as embodied in the MOA. REINSTATED.SO ORDERED

Respondent moved for a reconsideration of the Decision as petitioner’s vice-president submitted a "Muling Pagpapatibay ng Pagsang-ayon sa Republic of the Philippines
Kasunduan na may Petsang ika-4 ng Agosto 2005,"5 stating that the union members were waiving their rights and benefits under the Secretary’s SUPREME COURT
Decision. Reconsideration of the Decision was denied by Resolution6 of August 12, 2008, hence, respondent filed a petition for certiorari before the Court Manila
of Appeals. THIRD DIVISION
G.R. No.174287 August 12, 2013
7 NATIONAL UNION OF BANK EMPLOYEES (NUBE), PETITIONER,
By Decision of September 24, 2009, the appellate court ruled in favor of respondent and accordingly set aside the Decision of the Secretary of Labor. It
vs.
held that the Secretary of Labor gravely abused his discretion in not respecting the MOA. It did not give credence to the minutes of the meeting8 that
PHILNABANK EMPLOYEES ASSOCIATION (PEMA) AND PHILIPPINE NATIONAL BANK, RESPONDENTS.
attended the forging of the MOA as it was not verified, nor to the "Paliwanag" 9 submitted by respondent union members explaining why they signed the
DECISION
MOA as it was not notarized.
PERALTA, J.:

Petitioner’s motion for reconsideration having been denied by Resolution10 of December 2, 2009, the present petition was filed, maintaining that the Assailed in this petition for review on certiorari under Rule 45 of the 1997 Rules of Civil Procedure are the May 22, 2006 Decision1 and August 17, 2006
Secretary of Labor’s award is in order, being in accord with the parties’ CBA history ─ respondent having already granted P15.00 per day for Resolution2 of the Court of Appeals (CA) in CA-G.R. SP No. 84606, which reversed the May 27, 2004 Decision3 of the Secretary of Labor and
2001, P10.00 per day for 2002, and P10.00 per day for 2003, and that the Secretary has the power to grant awards higher than what are stated in the Employment acting as voluntary arbitrator, the dispositive portion of which states:
CBA.
WHEREFORE, in light of the foregoing findings, the Bank is hereby ORDERED to release all union dues withheld and to continue remitting to NUBE-PNB
Respecting the MOA, petitioner posits that it was "surreptitiously entered into [in] bad faith," it having been forged without the assistance of the Federation chapter the members' obligations under the CBA, LESS the amount corresponding to the number of non-union members including those who participated
of Free Workers or counsel, adding that respondent could have waited for the Secretary’s resolution of the pending CBA deadlock or that the MOA could in the unsuccessful withdrawal of membership from their mother union.
have been concluded before representatives of the Secretary of Labor.
The parties are enjoined to faithfully comply with the above- mentioned resolution.
The relevant issues for resolution are 1) whether the Secretary of Labor is authorized to give an award higher than that agreed upon in the MOA, and 2)
whether the MOA was entered into and ratified by the remaining officers of petitioner under the condition, which was not incorporated in the MOA, that
With respect to the URGENT MOTION FOR INTERVENTION filed by PEMA, the same is hereby denied without prejudice to the rights of its members to
respondent would honor the Secretary of Labor’s award in the event that it is higher.
bring an action to protect such rights if deemed necessary at the opportune time.SO ORDERED. 4

The Court resolves both issues in the affirmative.


We state the facts.

It is well-settled that the Secretary of Labor, in the exercise of his power to assume jurisdiction under Art. 263 (g) 11 of the Labor Code, may resolve all
Respondent Philippine National Bank (PNB) used to be a government-owned and controlled banking institution established under Public Act 2612, as
issues involved in the controversy including the award of wage increases and benefits. 12 While an arbitral award cannot per se be categorized as an
amended by Executive Order No. 80 dated December 3, 1986 (otherwise known as The 1986 Revised Charter of the Philippine National Bank). Its rank-
agreement voluntarily entered into by the parties because it requires the intervention and imposing power of the State thru the Secretary of Labor when
and-file employees, being government personnel, were represented for collective negotiation by the Philnabank Employees Association (PEMA), a public
he assumes jurisdiction, the arbitral award can be considered an approximation of a collective bargaining agreement which would otherwise have been
sector union.
entered into by the parties, hence, it has the force and effect of a valid contract obligation. 13

In 1996, the Securities and Exchange Commission approved PNB’s new Articles of Incorporation and By-laws and its changed status as a private
That the arbitral award was higher than that which was purportedly agreed upon in the MOA is of no moment. For the Secretary, in resolving the CBA
corporation. PEMA affiliated with petitioner National Union of Bank Employees (NUBE), which is a labor federation composed of unions in the banking
deadlock, is not limited to considering the MOA as basis in computing the wage increases. He could, as he did, consider the financial
industry, adopting the name NUBE-PNB Employees Chapter (NUBE-PEC).
documents14 submitted by respondent as well as the parties’ bargaining history and respondent’s financial outlook and improvements as stated in its
website.15
Later, NUBE-PEC was certified as the sole and exclusive bargaining agent of the PNB rank-and-file employees. A collective bargaining agreement (CBA)
was subsequently signed between NUBE-PEC and PNB covering the period of January 1, 1997 to December 31, 2001.
It bears noting that since the filing and submission of the MOA did not have the effect of divesting the Secretary of his jurisdiction, or of automatically
disposing the controversy, then neither should the provisions of the MOA restrict the Secretary’s leeway in deciding the matters before him.1avvphi1
Pursuant to Article V on Check-off and Agency Fees of the CBA, PNB shall deduct the monthly membership fee and other assessments imposed by the
union from the salary of each union member, and agency fee (equivalent to the monthly membership dues) from the salary of the rank- and-file
Labor Relations Set IV * Takata v BLR to Silang v Coa * Page 9 of 29

employees within the bargaining unit who are not union members. Moreover, during the effectivity of the CBA, NUBE, being the Federation union, agreed Meantime, the DOLE denied PEMA’s motion to change its name in the official ballots. The certification election was finally held on October 17, 2003. The
that PNB shall remit P15.00 of the P65.00 union dues per month collected by PNB from every employee, and that PNB shall directly credit the amount to election yielded the following results:
NUBE’s current account with PNB.5

Following the expiration of the CBA, the Philnabank Employees Association-FFW (PEMA-FFW) filed on January 2, 2002 a petition for certification Number of eligible voters 3,742
election among the rank-and-file employees of PNB. The petition sought the conduct of a certification election to be participated in by PEMA-FFW and
Number of valid votes cast 2,993
NUBE-PEC.
Number of spoiled ballots 72
While the petition for certification election was still pending, two significant events transpired – the independent union registration of NUBE- PEC and its
disaffiliation with NUBE. Total 3,065

Philnabank Employees Association-FFW 289


With a legal personality derived only from a charter issued by NUBE, NUBE-PEC, under the leadership of Mariano Soria, decided to apply for a separate
registration with the Department of Labor and Employment (DOLE). On March 25, 2002, it was registered as an independent labor organization under National Union of Bank Employees (NUBE)-
Registration Certificate No. NCR-UR-3-3790-2002. Philippine National Bank (PNB) Chapter 2,683

Thereafter, on June 20, 2003, the Board of Directors of NUBE-PEC adopted a Resolution6 disaffiliating itself from NUBE. Cited as reasons were as No Union 21
follows:
14
xxxx Total 2,993

WHEREAS, in the long period of time that the Union has been affiliated with NUBE, the latter has miserably failed to extend and provide satisfactory
services and support to the former in the form of legal services, training assistance, educational seminars, and the like; On April 28, 2004, PEMA filed before the voluntary arbitrator an Urgent Motion for Intervention, 15 alleging that it stands to be substantially affected by
whatever judgment that may be issued, because one of the issues for resolution is the validity of its disaffiliation from NUBE. It further claimed that its
presence is necessary so that a complete relief may be accorded to the parties. Only NUBE opposed the motion, arguing that PEMA has no legal
WHEREAS, this failure by NUBE to provide adequate essential services and support to union members have caused the latter to be resentful to NUBE personality to intervene, as it is not a party to the existing CBA; and that NUBE is the exclusive bargaining representative of the PNB rank-and-file
and to demand for the Union’s disaffiliation from the former[;] employees and, in dealing with a union other than NUBE, PNB is violating the duty to bargain collectively, which is another form of ULP.16

WHEREAS, just recently, NUBE displayed its lack of regard for the interests and aspirations of the union members by blocking the latter’s desire for the Barely a month after, DOLE Acting Secretary Manuel G. Imson denied PEMA’s motion for intervention and ordered PNB to release all union dues
early commencement of CBA negotiations with the PNB management[;] withheld and to continue remitting the same to NUBE. The May 27, 2004 Decision opined:

WHEREAS, this strained relationship between NUBE and the Union is no longer conducive to a fruitful partnership between them and could even Before we delve into the merits of the present dispute, it behooves [Us] to discuss in passing the propriety of the MOTION FOR INTERVENTION filed by
threaten industrial peace between the Union and the management of PNB. the Philnabank Employees Association (PEMA) on April 28, 2004, the alleged [break-away] group of NUBE- PNB Chapter.

WHEREAS, under the circumstances, the current officers of the Union have no choice but to listen to the clamor of the overwhelming majority of union A cursory reading of the motion reveals a denial thereof is not prejudicial to the individual rights of its members. They are protected by law.
members for the Union to disaffiliate from NUBE.7

Coming now to the main issues of the case, suffice it to say that after an evaluative review of the record of the case, taking into consideration the
The duly notarized Resolution was signed by Edgardo B. Serrana (President), Rico B. Roma (Vice-President), Rachel C. Latorre (Secretary), Valeriana S. arguments and evidence adduced by both parties, We find that indeed no effective disaffiliation took place.
Garcia (Director/Acting Treasurer), Ruben C. Medrano (Director), and Verlo C. Magtibay (Director). It is claimed that said Resolution was overwhelmingly
ratified by about eighty-one percent (81%) of the total union membership.
It is well settled that [l]abor unions may disaffiliate from their mother federations to form a local or independent union only during the 60-day freedom
8 period immediately preceding the expiration of the CBA. [Tanduay Distillery Labor Union v. National Labor Relations Commission, et al.] However, such
On June 25, 2003, NUBE-PEC filed a Manifestation and Motion before the Med-Arbitration Unit of DOLE, praying that, in view of its independent disaffiliation must be effected by a majority of the members in the bargaining unit. (Volkschel Labor Union v. Bureau of Labor Relations).
registration as a labor union and disaffiliation from NUBE, its name as appearing in the official ballots of the certification election be changed to
"Philnabank Employees Association (PEMA)" or, in the alternative, both parties be allowed to use the name "PEMA" but with PEMA-FFW and NUBE-
PEC be denominated as "PEMA-Bustria Group" and "PEMA-Serrana Group," respectively. Applying the foregoing jurisprudence to the case at bar, it is difficult to believe that a justified disaffiliation took place. While the record apparently shows
that attempts at disaffiliation occurred sometime in June of 2003 x x x the latest result of a certification election dated 17 October 2003 mooted such
disaffiliation.
On the same date, PEMA sent a letter to the PNB management informing its disaffiliation from NUBE and requesting to stop, effective immediately, the
check-off of the P15.00 due for NUBE.9
Further, even if for the sake of argument an attempt at disaffiliation occurred, the record is bereft of substantial evidence to support a finding of effective
disaffiliation. There might have been a mass withdrawal of the union members from the NUBE-PNB Chapter. The record shows, however, that only 289
Acting thereon, on July 4, 2003, PNB informed NUBE of PEMA’s letter and its decision to continue the deduction of the P15.00 fees, but stop its out of 3,742 members shifted their allegiance from the mother union. Hence, they constituted a small minority for which reason they could not have
remittance to NUBE effective July 2003. PNB also notified NUBE that the amounts collected would be held in a trust account pending the resolution of the successfully severed the local union’s affiliation with NUBE.
issue on PEMA’s disaffiliation.10

Thus, since only a minority of the members wanted disaffiliation as shown by the certification election, it can be inferred that the majority of the members
On July 11, 2003, NUBE replied that: it remains as the exclusive bargaining representative of the PNB rank-and-file employees; by signing the Resolution wanted the union to remain an affiliate of the NUBE. [Villar, et al. v. Inciong, et al.]. There being no justified disaffiliation that took place, the bargaining
(on disaffiliation), the chapter officers have abandoned NUBE-PEC and joined another union; in abandoning NUBE-PEC, the chapter officers have agent’s right under the provision of the CBA on Check-Off is unaffected and still remained with the old NUBE-PNB Chapter. x x x
abdicated their respective positions and resigned as such; in joining another union, the chapter officers committed an act of disloyalty to NUBE-PEC and
the general membership; the circumstances clearly show that there is an emergency in NUBE-PEC necessitating its placement under temporary
trusteeship; and that PNB should cease and desist from dealing with Serrana, Roma, Latorre, Garcia, Medrano, and Magtibay, who are expelled from While it is true that the obligation of an employee to pay union dues is co-terminus with his affiliation [Philippine Federation of Petroleum Workers v. CIR],
NUBE-PEC.11 With regard to the issue of non-remittance of the union dues, NUBE enjoined PNB to comply with the union check-off provision of the CBA; it is equally tenable that when it is shown, as in this case, that the withdrawal from the mother union is not supported by majority of the members, the
otherwise, it would elevate the matter to the grievance machinery in accordance with the CBA. disaffiliation is unjustified and the disaffiliated minority group has no authority to represent the employees of the bargaining unit. This is the import of the
principle laid down in [Volkschel Labor Union v. Bureau of Labor Relations supra] and the inverse application of the Supreme Court decision in [Philippine
Federation of Petroleum Workers v. CIR] regarding entitlement to the check-off provision of the CBA.
Despite NUBE’s response, PNB stood firm on its decision. Alleging unfair labor practice (ULP) for non-implementation of the grievance machinery and
procedure, NUBE brought the matter to the National Conciliation and Mediation Board (NCMB) for preventive mediation.12 In time, PNB and NUBE
agreed to refer the case to the Office of the DOLE Secretary for voluntary arbitration. They executed a Submission Agreement on October 28, 2003. 13 As a necessary consequence to our finding that no valid disaffiliation took place, the right of NUBE to represent its local chapter at the PNB, less those
employees who are no longer members of the latter, is beyond reproach.
Labor Relations Set IV * Takata v BLR to Silang v Coa * Page 10 of 29

However, the Bank cannot be faulted for not releasing union dues to NUBE at the time when representation status issue was still being threshed out by The Secretary of Labor did not commit serious error in ordering the release of the disputed union fees/dues to NUBE-PNB Chapter.
proper governmental authority. Prudence dictates the discontinuance of remittance of union dues to NUBE under such circumstances was a legitimate IV.
exercise of management discretion apparently in order to protect the Bank’s business interest. The suspension of the check-off provision of the CBA, at There is no substantial basis for the issuance of a preli minary injunction or temporary restraining order.
the instance of the latter made in good faith, under the present circumstances cannot give rise to a right of action. For having been exercised without V.
malice much less evil motive and for not causing actual loss to the National Union of Bank Employees (NUBE), the same act of management [cannot] be Under the Rules of Court, the appeal/petition of PEMA should have been dismissed.
penalized.17 VI.
PEMA and NUBE are not one and the same, and the denial by the Secretary of Labor of the motion for intervention was proper.
VII.
Aggrieved, PEMA filed before the CA a petition under Rule 43 of the Rules on Civil Procedure with prayer for the issuance of a temporary restraining
NUBE-PNB Chapter, not PEMA, has been fighting for PNB rank-and-file interests and rights since PNB’s privatization, which is further pro of that NUBE-
order (TRO) or writ of preliminary injunction (WPI). On November 2, 2004, the CA denied the application for WPI. 18 PEMA’s motion for reconsideration
PNB Chapter and PEMA are not one and the same.
was also denied on February 24, 2005, noting PNB’s manifestation that it would submit to the judgment of the CA as to which party it should remit the
VIII.
funds collected from the employees.19
The alleged disaffiliation was not valid as proper procedure was not followed.
IX.
On June 21, 2005, however, petitioner again filed an Urgent Motion for the Issuance of a TRO against the June 10, 2005 Resolution of DOLE Acting NUBE is entitled to check-off.26
Secretary Imson, which ordered PNB to properly issue a check directly payable to the order of NUBE covering the withheld funds from the trust
account.20 Considering the different factual milieu, the CA resolved to grant the motion. 21 Stripped of the non-essential, the issue ultimately boils down on whether PEMA validly disaffiliated itself from NUBE, the resolution of which, in turn,
inevitably affects the latter’s right to collect the union dues held in trust by PNB.
Subsequent to the parties’ submission of memoranda, the CA promulgated its May 22, 2006 Decision, declaring the validity of PEMA’s disaffiliation from
NUBE and directing PNB to return to the employees concerned the amounts deducted and held in trust for NUBE starting July 2003 and to stop further We deny the petition.
deductions in favor of NUBE.22
Whether there was a valid disaffiliation is a factual issue. 27 It is elementary that a question of fact is not appropriate for a petition for review on certiorari
As to the impropriety of denying PEMA’s motion for intervention, the CA noted: under Rule 45 of the Rules of Court. The parties may raise only questions of law because the Supreme Court is not a trier of facts. As a general rule, We
are not duty-bound to analyze again and weigh the evidence introduced in and considered by the tribunals below. When supported by substantial
evidence, the findings of fact of the CA are conclusive and binding on the parties and are not reviewable by this Court, except: (1) When the conclusion is
x x x Among the rights of the [PEMA] as an affiliate of a federation is to disaffiliate from it. Any case in which this is an issue is then one in which the union a finding grounded entirely on speculation, surmises and conjectures; (2) When the inference made is manifestly mistaken, absurd or impossible; (3)
has a significant legal interest and as to which it must be heard, irrespective of any residual rights of the members after a decision that might deny a
Where there is a grave abuse of discretion; (4) When the judgment is based on a misapprehension of facts; (5) When the findings of fact are conflicting;
disaffiliation. It is a non-sequitur to make the intervention of the union in this case dependent on the question of whether its members can pursue their (6) When the CA, in making its findings, went beyond the issues of the case and the same is contrary to the admissions of both parties; (7) When the
own agenda under the same constraints.23 findings are contrary to those of the trial court; (8) When the findings of fact are conclusions without citation of specific evidence on which they are based;
(9) When the facts set forth in the petition as well as in the petitioner’s main and reply briefs are not disputed by the respondents; and (10) When the
On the validity of PEMA’s disaffiliation, the CA ratiocinated: findings of fact of the CA are premised on the supposed absence of evidence and contradicted by the evidence on record. 28 The Court finds no cogent
reason to apply these recognized exceptions.
The power and freedom of a local union to disaffiliate from its mother union or federation is axiomatic. As Volkschel vs. Bureau of Labor Relations [137
SCRA 42] recognizes, a local union is, after all, a separate and voluntary association that under the constitutional guarantee of freedom of expression is Even a second look at the records reveals that the arguments raised in the petition are bereft of merit.
free to serve the interests of its members. Such right and freedom invariably include the right to disaffiliate or declare its autonomy from the federation or
mother union to which it belongs, subject to reasonable restrictions in the law or the federation’s constitution. [Malayang Samahan ng mga Manggagawa The right of the local union to exercise the right to disaffiliate from its mother union is well settled in this jurisdiction. In MSMG-UWP v. Hon. Ramos,29 We
sa M. Greenfield vs. Ramos, 326 SCRA 428] held:

Without any restrictive covenant between the parties, [Volkschel Labor Union vs. Bureau of Labor Relations, supra, at 48,] it is instructive to look into the A local union has the right to disaffiliate from its mother union or declare its autonomy. A local union, being a separate and voluntary association, is free to
state of the law on a union’s right to disaffiliate. The voluntary arbitrator alludes to a provision in PD 1391 allowing disaffiliation only within a 60-day period serve the interests of all its members including the freedom to disaffiliate or declare its autonomy from the federation which it belongs when
preceding the expiration of the CBA. In Alliance of Nationalist and Genuine Labor Organization vs. Samahan ng mga Manggagawang Nagkakaisa sa
circumstances warrant, in accordance with the constitutional guarantee of freedom of association.
Manila Bay Spinning Mills, etc. [258 SCRA 371], however, the rule was not held to be iron-clad. Volkschel was cited to support a more flexible view that
the right may be allowed as the circumstances warrant. In Associated Workers Union-PTGWO vs. National Labor Relations Commission [188 SCRA
123], the right to disaffiliate was upheld before the onset of the freedom period when it became apparent that there was a shift of allegiance on the part of The purpose of affiliation by a local union with a mother union [or] a federation
the majority of the union members.
xxxx
"x x x is to increase by collective action the bargaining power in respect of the terms and conditions of labor. Yet the locals remained the basic units of
association, free to serve their own and the common interest of all, subject to the restraints imposed by the Constitution and By-Laws of the Association,
As the records show, a majority, indeed a vast majority, of the members of the local union ratified the action of the board to disaffiliate. Our count of the
and free also to renounce the affiliation for mutual welfare upon the terms laid down in the agreement which brought it into existence."
members who approved the board action is, 2,638. If we divide this by the number of eligible voters as per the certification election which is 3,742, the
quotient is 70.5%, representing the proportion of the members in favor of disaffiliation. The [PEMA] says that the action was ratified by 81%. Either way,
the groundswell of support for the measure was overwhelming. Thus, a local union which has affiliated itself with a federation is free to sever such affiliation anytime and such disaffiliation cannot be considered
disloyalty. In the absence of specific provisions in the federation's constitution prohibiting disaffiliation or the declaration of autonomy of a local union, a
local may dissociate with its parent union.30
The respondent NUBE has developed the ingenious theory that if the disaffiliation was approved by a majority of the members, it was neutered by the
subsequent certification election in which NUBE-PNB Chapter was voted the sole and exclusive bargaining agent. It is argued that the effects of this
change must be upheld as the latest expression of the will of the employees in the bargaining unit. The truth of the matter is that the names of PEMA and Likewise, Philippine Skylanders, Inc. v. National Labor Relations Commission31 restated:
NUBE-PNB Chapter are names of only one entity, the two sides of the same coin. We have seen how NUBE-PNB Employees Chapter evolved into
PEMA and competed with Philnabank Employees Association-FFW for supremacy in the certification election. To realize that it was PEMA which entered
The right of a local union to disaffiliate from its mother federation is not a novel thesis unillumined by case law.1âwphi1In the landmark case of Liberty
into the contest, we need only to remind ourselves that PEMA was the one which filed a motion in the certification election case to have its name PEMA
Cotton Mills Workers Union vs. Liberty Cotton Mills, Inc., we upheld the right of local unions to separate from their mother federation on the ground that as
put in the official ballot. DOLE insisted, however, in putting the name NUBE-PNB Chapter in the ballots unaware of the implications of this seemingly
separate and voluntary associations, local unions do not owe their creation and existence to the national federation to which they are affiliated but,
innocuous act.24
instead, to the will of their members. The sole essence of affiliation is to increase, by collective action, the common bargaining power of local unions for
the effective enhancement and protection of their interests. Admittedly, there are times when without succor and support local unions may find it hard,
NUBE filed a motion for reconsideration, but it was denied;25 hence, this petition raising the following issues for resolution: unaided by other support groups, to secure justice for themselves.
I.
The Secretary of Labor acted without error and without grave abuse of discretion in not giving due course to the urgent motion for intervention filed by
Yet the local unions remain the basic units of association, free to serve their own interests subject to the restraints imposed by the constitution and by-
PEMA.
laws of the national federation, and free also to renounce the affiliation upon the terms laid down in the agreement which brought such affiliation into
II.
existence.
The Secretary of Labor acted without grave abuse of discretion and without serious error in ruling that PEMA’s alleged disaffiliation was invalid.
III.
Labor Relations Set IV * Takata v BLR to Silang v Coa * Page 11 of 29

Such dictum has been punctiliously followed since then.32 between an employer and the parent organization as bargaining agent for the employees is terminated bv the disaffiliation ofthe local of which the
employees are members. x x x44
And again, in Coastal Subic Bay Terminal, Inc. v. Department of Labor and Employment – Office of the Secretary,33 this Court opined:
On the other hand, it was entirely reasonable for PNB to enter into a CBA with PEMA as represented by Serrana et al. Since PEMA had validly separated
itself from NUBE, there would be no restrictions which could validly hinder it from collectively bargaining with PNB.
Under the rules implementing the Labor Code, a chartered local union acquires legal personality through the charter certificate issued by a duly registered
federation or national union, and reported to the Regional Office in accordance with the rules implementing the Labor Code. A local union does not owe
its existence to the federation with which it is affiliated. It is a separate and distinct voluntary association owing its creation to the will of its members. Mere WHEREFORE, the foregoing considered, the instant Petition is DENIED. The May 22, 2006 Decision and August 17, 2006 Resolution of the Court of
affiliation does not divest the local union of its own personality, neither does it give the mother federation the license to act independently of the local Appeals in CA-G.R. SP No. 84606, which reversed the May 27, 2004 Decision ofthe Secretary of Labor and Employment, are AFFIRMED.SO ORDERED
union. It only gives rise to a contract of agency, where the former acts in representation of the latter. Hence, local unions are considered principals while
the federation is deemed to be merely their agent. As such principals, the unions are entitled to exercise the rights and privileges of a legitimate labor
Republic of the Philippines
organization, including the right to seek certification as the sole and exclusive bargaining agent in the appropriate employer unit. 34
SUPREME COURT
Manila
Finally, the recent case of Cirtek Employees Labor Union-Federation of Free Workers v. Cirtek Electronics, Inc35ruled: THIRD DIVISION
G.R. No. 162943 December 6, 2010
EMPLOYEES UNION OF BAYER PHILS., FFW and JUANITO S. FACUNDO, in his capacity as President,Petitioners,
x x x [A] local union may disaffiliate at any time from its mother federation, absent any showing that the same is prohibited under its constitution or rule.
vs.
Such, however, does not result in it losing its legal personality altogether. Verily, Anglo-KMU v. Samahan Ng Mga Manggagawang Nagkakaisa Sa Manila
BAYER PHILIPPINES, INC., DIETER J. LONISHEN (President), ASUNCION AMISTOSO (HRD Manager), AVELINA REMIGIO AND ANASTACIA
Bar Spinning Mills At J.P. Coats enlightens:
VILLAREAL, Respondents.
DECISION
A local labor union is a separate and distinct unit primarily designed to secure and maintain an equality of bargaining power between the employer and VILLARAMA, JR., J.:
their employee-members. A local union does not owe its existence to the federation with which it is affiliated. It is a separate and distinct voluntary
association owing its creation to the will of its members. The mere act of affiliation does not divest the local union of its own personality, neither does it This petition for review on certiorari assails the Decision1 dated December 15, 2003 and Resolution2 dated March 23, 2004 of the Court of Appeals (CA)
give the mother federation the license to act independently of the local union. It only gives rise to a contract of agency where the former acts in in CA-G.R. SP No. 73813.
representation of the latter.36
Petitioner Employees Union of Bayer Philippines3 (EUBP) is the exclusive bargaining agent of all rank-and-file employees of Bayer Philippines (Bayer),
These and many more have consistently reiterated the earlier view that the right of the local members to withdraw from the federation and to form a new and is an affiliate of the Federation of Free Workers (FFW). In 1997, EUBP, headed by its president Juanito S. Facundo (Facundo), negotiated with Bayer
local union depends upon the provisions of the union's constitution, by-laws and charter and, in the absence of enforceable provisions in the federation's for the signing of a collective bargaining agreement (CBA). During the negotiations, EUBP rejected Bayer’s 9.9% wage-increase proposal resulting in a
constitution preventing disaffiliation of a local union, a local may sever its relationship with its parent. 37 In the case at bar, there is nothing shown in the bargaining deadlock. Subsequently, EUBP staged a strike, prompting the Secretary of the Department of Labor and Employment (DOLE) to assume
records nor is it claimed by NUBE that PEMA was expressly forbidden to disaffiliate from the federation nor were there any conditions imposed for a valid jurisdiction over the dispute.
breakaway. This being so, PEMA is not precluded to disaffiliate from NUBE after acquiring the status of an independent labor organization duly registered
before the DOLE.
In November 1997, pending the resolution of the dispute, respondent Avelina Remigio (Remigio) and 27 other union members, without any authority from
their union leaders, accepted Bayer’s wage-increase proposal. EUBP’s grievance committee questioned Remigio’s action and reprimanded Remigio and
Also, there is no merit on NUBE’s contention that PEMA’s disaffiliation is invalid for non-observance of the procedure that union members should make her allies. On January 7, 1998, the DOLE Secretary issued an arbitral award ordering EUBP and Bayer to execute a CBA retroactive to January 1, 1997
such determination through secret ballot and after due deliberation, conformably with Article 241 (d) of the Labor Code, as amended.38 Conspicuously, and to be made effective until December 31, 2001. The said CBA 4 was registered on July 8, 1998 with the Industrial Relations Division of the DOLE-
other than citing the opinion of a "recognized labor law authority," NUBE failed to quote a specific provision of the law or rule mandating that a local National Capital Region (NCR).5
union’s disaffiliation from a federation must comply with Article 241 (d) in order to be valid and effective.
Meanwhile, the rift between Facundo’s leadership and Remigio’s group broadened. On August 3, 1998, barely six months from the signing of the new
Granting, for argument’s sake, that Article 241 (d) is applicable, still, We uphold PEMA’s disaffiliation from NUBE. First, non-compliance with the CBA, during a company-sponsored seminar,6 Remigio solicited signatures from union members in support of a resolution containing the decision of the
procedure on disaffiliation, being premised on purely technical grounds cannot rise above the employees’ fundamental right to self-organization and to signatories to: (1) disaffiliate from FFW, (2) rename the union as Reformed Employees Union of Bayer Philippines (REUBP), (3) adopt a new constitution
form and join labor organizations of their own choosing for the purpose of collective bargaining. 39 Second, the Article nonetheless provides that when the and by-laws for the union, (4) abolish all existing officer positions in the union and elect a new set of interim officers, and (5) authorize REUBP to
nature of the organization renders such secret ballot impractical, the union officers may make the decision in behalf of the general membership. In this administer the CBA between EUBP and Bayer.7 The said resolution was signed by 147 of the 257 local union members. A subsequent resolution was
case, NUBE did not even dare to contest PEMA’s representation that "PNB employees, from where [PEMA] [derives] its membership, are scattered from also issued affirming the first resolution.8
Aparri to Jolo, manning more than 300 branches in various towns and cities of the country," hence, "[to] gather the general membership of the union in a
general membership to vote through secret balloting is virtually impossible."40 It is understandable, therefore, why PEMA’s board of directors merely opted
A tug-of-war then ensued between the two rival groups, with both seeking recognition from Bayer and demanding remittance of the union dues collected
to submit for ratification of the majority their resolution to disaffiliate from NUBE. Third, and most importantly, NUBE did not dispute the existence of the
from its rank-and-file members. On September 8, 1998, Remigio’s splinter group wrote Facundo, FFW and Bayer informing them of the decision of the
persons or their due execution of the document showing their unequivocal support for the disaffiliation of PEMA from NUBE. Note must be taken of the
majority of the union members to disaffiliate from FFW.9 This was followed by another letter informing Facundo, FFW and Bayer that an interim set of
fact that the list of PEMA members (identifying themselves as "PEMA-Serrana Group"41) who agreed with the board resolution was attached as Annex
REUBP executive officers and board of directors had been appointed, and demanding the remittance of all union dues to REUBP. Remigio also asked
"H" of PEMA’s petition before the CA and covered pages 115 to 440 of the CA rollo. While fully displaying the employees’ printed name, identification
Bayer to desist from further transacting with EUBP. Facundo, meanwhile, sent similar requests to Bayer10 requesting for the remittance of union dues in
number, branch, position, and signature, the list was left unchallenged by NUBE. No evidence was presented that the union members’ ratification was
favor of EUBP and accusing the company of interfering with purely union matters. 11 Bayer responded by deciding not to deal with either of the two
obtained by mistake or through fraud, force or intimidation. Surely, this is not a case where one or two members of the local union decided to disaffiliate
groups, and by placing the union dues collected in a trust account until the conflict between the two groups is resolved.12
from the mother federation, but one where more than a majority of the local union members decided to disaffiliate.

On September 15, 1998, EUBP filed a complaint for unfair labor practice (first ULP complaint) against Bayer for non-remittance of union dues. The case
Consequently, by PEMA's valid disaffiliation from NUBE, the vinculum that previously bound the two entities was completely severed. As NUBE was
was docketed as NLRC-NCR-Case No. 00-09-07564-98.13
divested of any and all power to act in representation of PEMA, any act performed by the former that affects the interests and affairs of the latter,
including the supposed expulsion of Serrana et al., is rendered without force and effect.
EUBP later sent a letter dated November 5, 1998 to Bayer asking for a grievance conference. 14 The meeting was conducted by the management on
November 11, 1998, with all REUBP officers including their lawyers present. Facundo did not attend the meeting, but sent two EUBP officers to inform
Also, in effect, NUBE loses it right to collect all union dues held in its trust by PNB. The moment that PEMA separated from and left NUBE and exists as
REUBP and the management that a preventive mediation conference between the two groups has been scheduled on November 12, 1998 before the
an independent labor organization with a certificate of registration, the former is no longer obliged to pay dues and assessments to the latter; naturally,
National Conciliation and Mediation Board (NCMB).15
there would be no longer any reason or occasion for PNB to continue making deductions. 42 As we said in Volkschel Labor Union v. Bureau of Labor
Relations:43
Apparently, the two groups failed to settle their issues as Facundo again sent respondent Dieter J. Lonishen two more letters, dated January 14,
199916 and September 2, 1999,17 asking for a grievance meeting with the management to discuss the failure of the latter to comply with the terms of their
x x x In other words, ALUMETAL [NUBE in this case] is entitled to receive the dues from respondent companies as long as petitioner union is affiliated
CBA. Both requests remained unheeded.
with it and respondent companies are authorized by their employees (members of petitioner union) to deduct union dues. Without said affiliation, the
employer has no link to the mother union. The obligation of an employee to pay union dues is coterminous with his affiliation or membership. "The
employees' check-off authorization, even if declared irrevocable, is good only as long as they remain members of the union concerned." A contract
Labor Relations Set IV * Takata v BLR to Silang v Coa * Page 12 of 29

On February 9, 1999, while the first ULP case was still pending and despite EUBP’s repeated request for a grievance conference, Bayer decided to turn Thus, petitioners filed a Rule 65 petition to the CA. On December 15, 2003, the CA sustained both the Labor Arbiter and the NLRC’s rulings. The
over the collected union dues amounting to P254,857.15 to respondent Anastacia Villareal, Treasurer of REUBP. appellate court explained,

Aggrieved by the said development, EUBP lodged a complaint18 on March 4, 1999 against Remigio’s group before the Industrial Relations Division of the A cursory reading of the three pleadings, to wit: the Complaint (Vol. I, Rollo, p[p]. 166-167); the Amended Complaint (Vol. I, Rollo[,] pp. 168-172) and
DOLE praying for their expulsion from EUBP for commission of "acts that threaten the life of the union." the Second Amended Complaint dated March 8, 2000 (Vol. II, Rollo, pp. 219-225) will readily show that the instant case was brought about by the action
of the Group of REM[I]GIO to disaffiliate from FFW and to organized (sic) REUBP under the tutelage of REM[I]GIO and VILLAREAL. At first glance of the
case at bar, it involves purely an (sic) inter-union and intra-union conflicts or disputes between EUBP-FFW and REUBP which issue should have been
On June 18, 1999, Labor Arbiter Jovencio Ll. Mayor, Jr. dismissed the first ULP complaint for lack of jurisdiction.19The Arbiter explained that the root
resolved by the Bureau of Labor Relations under Article 226 of the Labor Code. However, since no less than petitioners who admitted that respondents
cause for Bayer’s failure to remit the collected union dues can be traced to the intra-union conflict between EUBP and Remigio’s group20 and that the
committed gross violations of the CBA, then the BLR is divested of jurisdiction over the case and the issue should have been referred to the Grievance
charges imputed against Bayer should have been submitted instead to voluntary arbitration. 21 EUBP did not appeal the said decision.22
Machinery and Voluntary Arbitrator and not to the Labor Arbiter as what petitioners did in the case at bar. x x x
xxxx
On December 14, 1999, petitioners filed a second ULP complaint against herein respondents docketed as NLRC-RAB-IV Case No. 12-11813-99-L. Three
days later, petitioners amended the complaint charging the respondents with unfair labor practice committed by organizing a company union, gross Furthermore, the CBA entered between BAYER and EUBP-FFW [has] a life span of only five years and after the said period, the employees have all the
violation of the CBA and violation of their duty to bargain.23 Petitioners complained that Bayer refused to remit the collected union dues to EUBP despite right to change their bargaining unit who will represent them. If there exist[s] two opposing unions in the same company, the remedy is not to declare that
several demands sent to the management.24 They also alleged that notwithstanding the requests sent to Bayer for a renegotiation of the last two years of such act is considered unfair labor practice but rather they should conduct a certification election provided [that] it should be conducted within 60 days of
the 1997-2001 CBA between EUBP and Bayer, the latter opted to negotiate instead with Remigio’s group. 25 the so[-]called freedom period before the expiration of the CBA.

On even date, REUBP and Bayer agreed to sign a new CBA. Remigio immediately informed her allies of the management’s decision.26 WHEREFORE, premises considered, this Petition is DENIED and the assailed Decision dated September 27, 2001 as well as the Order dated June 21,
2002, denying the motion for reconsideration, by the National Labor Relations Commission, First Division, in NLRC Case No. RAB-IV-12-11813-99-L,
are hereby AFFIRMED in toto. Costs against petitioners.SO ORDERED.40
In response, petitioners immediately filed an urgent motion for the issuance of a restraining order/injunction27before the National Labor Relations
Commission (NLRC) and the Labor Arbiter against respondents. Petitioners asserted their authority as the exclusive bargaining representative of all rank-
and-file employees of Bayer and asked that a temporary restraining order be issued against Remigio’s group and Bayer to prevent the employees from Undaunted, petitioners filed this Rule 45 petition before this Court. Initially, the said petition was denied for having been filed out of time and for failure to
ratifying the new CBA. Later, petitioners filed a second amended complaint28 to include in its complaint the issue of gross violation of the CBA for violation comply with the requirements provided in the 1997 Rules of Civil Procedure, as amended.41 Upon petitioners’ motion, however, we decided to reinstate
of the contract bar rule following Bayer’s decision to negotiate and sign a new CBA with Remigio’s group. their appeal.

Meanwhile, on January 26, 2000, the Regional Director of the Industrial Relations Division of DOLE issued a decision dismissing the issue on expulsion The following are the issues raised by petitioners, to wit:
filed by EUBP against Remigio and her allies for failure to exhaust reliefs within the union and ordering the conduct of a referendum to determine which of
the two groups should be recognized as union officers. 29 EUBP seasonably appealed the said decision to the Bureau of Labor Relations (BLR). 30 On June
I. WHETHER OR NOT THE HONORABLE COURT OF APPEALS, IN ARRIVING AT THE DECISION PROMULGATED ON 15 DECEMBER
16, 2000, the BLR reversed the Regional Director’s ruling and ordered the management of Bayer to respect the authority of the duly-elected officers of
2003 AND RESOLUTION PROMULGATED ON 23 MARCH 2004, DECIDED THE CASE IN ACCORDANCE WITH LAW AND
EUBP in the administration of the prevailing CBA.31
JURISPRUDENCE; AND

Unfortunately, the said BLR ruling came late since Bayer had already signed a new CBA32 with REUBP on February 21, 2000. The said CBA was
II. WHETHER OR NOT THE HONORABLE COURT OF APPEALS, IN ARRIVING AT THE DECISION PROMULGATED ON 15 DECEMBER
eventually ratified by majority of the bargaining unit.33
2003 AND RESOLUTION PROMULGATED ON 23 MARCH 2004, GRAVELY ABUSE[D] ITS DISCRETION IN ITS FINDINGS AND
CONCLUSION THAT:
On June 2, 2000, Labor Arbiter Waldo Emerson R. Gan dismissed EUBP’s second ULP complaint for lack of jurisdiction. 34 The Labor Arbiter explained
the dismissal as follows:
THE ACTS OF ABETTING OR ASSISTING IN THE CREATION OF ANOTHER UNION, NEGOTIATING OR BARGAINING WITH SUCH
UNION, WHICH IS NOT THE SOLE AND EXCLUSIVE BARGAINING AGENT, VIOLATING THE DUTY TO BARGAIN COLLECTIVELY,
All told, were it not for the fact that there were two (2) [groups] of employees, the Union led by its President Juanito Facundo and the members who REFUSAL TO PROCESS GRIEVABLE ISSUES IN THE GRIEVANCE MACHINERY AND/OR REFUSAL TO DEAL WITH THE SOLE AND
decided to disaffiliate led by Ms. Avelina Remigio, claiming to be the rightful representative of the rank and file employees, the Company would not have EXCLUSIVE BARGAINING AGENT ARE ACTS CONSTITUTING OR TANTAMOUNT TO UNFAIR LABOR PRACTICE.42
acted the way it did and the Union would not have filed the instant case.
Respondents Bayer, Lonishen and Amistoso, meanwhile, identify the issues as follows:
Clearly then, as the case involves intra-union disputes, this Office is bereft of any jurisdiction pursuant to Article 226 of the Labor Code, as amended,
which provides pertinently in part, thus:
I. WHETHER OR NOT THE UNIFORM FINDINGS OF THE COURT OF APPEALS, THE NLRC AND THE LABOR ARBITER ARE BINDING
ON THIS HONORABLE COURT;
"Bureau of Labor Relations – The Bureau of Labor Relations and the Labor Relations Divisions in the regional offices of the Department of Labor and
Employment shall have original and exclusive authority to act, at their own initiative or upon request of either or both parties, on all inter-union and intra-
II. WHETHER OR NOT THE LABOR ARBITER AND THE NLRC HAVE JURISDICTION OVER THE INSTANT CASE;
union conflicts, and all disputes, grievances or problems arising from or affecting labor-management relations in all workplaces whether agricultural or
non-agricultural, except those arising from the implementation or interpretation of collective bargaining agreements which shall be the subject of
grievance procedure and/or voluntary arbitration." III. WHETHER OR NOT THE INSTANT CASE INVOLVES AN INTRA-UNION DISPUTE;

Specifically, with respect to the union dues, the authority is the case of Cebu Seamen’s Association[,] Inc. vs. Ferrer-Calleja, (212 SCRA 51), where the IV. WHETHER OR NOT RESPONDENTS COMPANY, LONISHEN AND AMISTOSO COMMITTED AN ACT OF UNFAIR LABOR PRACTICE;
Supreme Court held that when the issue calls for the determination of which between the two groups within a union is entitled to the union dues, the same AND
cannot be taken cognizance of by the NLRC.x x x x
V. WHETHER OR NOT THE INSTANT CASE HAS BECOME MOOT AND ACADEMIC.43
WHEREFORE, premises considered, the instant complaint is hereby DISMISSED on the ground of lack of jurisdiction.SO ORDERED.35
Essentially, the issue in this petition is whether the act of the management of Bayer in dealing and negotiating with Remigio’s splinter group despite its
On June 28, 2000, the NLRC resolved to dismiss36 petitioners’ motion for a restraining order and/or injunction stating that the subject matter involved an validly existing CBA with EUBP can be considered unfair labor practice and, if so, whether EUBP is entitled to any relief.
intra-union dispute, over which the said Commission has no jurisdiction.37
Petitioners argue that the subject matter of their complaint, as well as the subsequent amendments thereto, pertain to the unfair labor practice act of
Aggrieved by the Labor Arbiter’s decision to dismiss the second ULP complaint, petitioners appealed the said decision, but the NLRC denied the respondents Bayer, Lonishen and Amistoso in dealing with Remigio’s splinter union. They contend that (1) the acts of abetting or assisting in the creation
appeal.38 EUBP’s motion for reconsideration was likewise denied.39 of another union is among those considered by the Labor Code, as amended, specifically under Article 248 (d)44 thereof, as unfair labor practice; (2) the
act of negotiating with such union constitutes a violation of Bayer’s duty to bargain collectively; and (3) Bayer’s unjustified refusal to process EUBP’s
grievances and to recognize the said union as the sole and exclusive bargaining agent are tantamount to unfair labor practice. 45
Labor Relations Set IV * Takata v BLR to Silang v Coa * Page 13 of 29

Respondents Bayer, Lonishen and Amistoso, on the other hand, contend that there can be no unfair labor practice on their part since the requisites for CBA with EUBP. Thus as to Bayer, Lonishen and Amistoso the question was whether they were liable for unfair labor practice, which issue was within the
unfair labor practice – i.e., that the violation of the CBA should be gross, and that it should involve violation in the economic provisions of the CBA – were jurisdiction of the NLRC. The dismissal of the second ULP complaint was therefore erroneous.
not satisfied. Moreover, they cite the ruling of the Labor Arbiter that the issues raised in the complaint should have been ventilated and threshed out
before the voluntary arbitrators as provided in Article 261 of the Labor Code, as amended.46 Respondents Remigio and Villareal, meanwhile, point out
However, as to respondents Remigio and Villareal, we find that petitioners’ complaint was validly dismissed.
that the case should be dismissed as against them since they are not real parties in interest in the ULP complaint against Bayer,47 and since there are no
specific or material acts imputed against them in the complaint.48
Petitioners’ ULP complaint cannot prosper as against respondents Remigio and Villareal because the issue, as against them, essentially involves an
intra-union dispute based on Section 1 (n) of DOLE Department Order No. 40-03. To rule on the validity or illegality of their acts, the Labor Arbiter and the
The petition is partly meritorious.
NLRC will necessarily touch on the issues respecting the propriety of their disaffiliation and the legality of the establishment of REUBP – issues that are
outside the scope of their jurisdiction. Accordingly, the dismissal of the complaint was validly made, but only with respect to these two respondents.
An intra-union dispute refers to any conflict between and among union members, including grievances arising from any violation of the rights and
conditions of membership, violation of or disagreement over any provision of the union’s constitution and by-laws, or disputes arising from chartering or
But are Bayer, Lonishen and Amistoso liable for unfair labor practice? On this score, we find that the evidence supports an answer in the affirmative.
disaffiliation of the union.49 Sections 1 and 2, Rule XI of Department Order No. 40-03, Series of 2003 of the DOLE enumerate the following circumstances
as inter/intra-union disputes, viz:
RULE XI It must be remembered that a CBA is entered into in order to foster stability and mutual cooperation between labor and capital. An employer should not
INTER/INTRA-UNION DISPUTES AND be allowed to rescind unilaterally its CBA with the duly certified bargaining agent it had previously contracted with, and decide to bargain anew with a
OTHER RELATED LABOR RELATIONS DISPUTES different group if there is no legitimate reason for doing so and without first following the proper procedure. If such behavior would be tolerated, bargaining
and negotiations between the employer and the union will never be truthful and meaningful, and no CBA forged after arduous negotiations will ever be
honored or be relied upon. Article 253 of the Labor Code, as amended, plainly provides:
Section 1. Coverage. - Inter/intra-union disputes shall include:

ART. 253. Duty to bargain collectively when there exists a collective bargaining agreement. – Where there is a collective bargaining agreement, the duty
(a) cancellation of registration of a labor organization filed by its members or by another labor organization;
to bargain collectively shall also mean that neither party shall terminate or modify such agreement during its lifetime. However, either party can serve a
written notice to terminate or modify the agreement at least sixty (60) days prior to its expiration date. It shall be the duty of both parties to keep the status
(b) conduct of election of union and workers’ association officers/nullification of election of union and workers’ association officers; quo and to continue in full force and effect the terms and conditions of the existing agreement during the 60-day period and/or until a new agreement is
reached by the parties. (Emphasis supplied.)1avvphi1
(c) audit/accounts examination of union or workers’ association funds;
This is the reason why it is axiomatic in labor relations that a CBA entered into by a legitimate labor organization that has been duly certified as the
exclusive bargaining representative and the employer becomes the law between them. Additionally, in the Certificate of Registration50 issued by the
(d) deregistration of collective bargaining agreements;
DOLE, it is specified that the registered CBA serves as the covenant between the parties and has the force and effect of law between them during the
period of its duration. Compliance with the terms and conditions of the CBA is mandated by express policy of the law primarily to afford protection to
(e) validity/invalidity of union affiliation or disaffiliation; labor51 and to promote industrial peace. Thus, when a valid and binding CBA had been entered into by the workers and the employer, the latter is
behooved to observe the terms and conditions thereof bearing on union dues and representation. 52 If the employer grossly violates its CBA with the duly
recognized union, the former may be held administratively and criminally liable for unfair labor practice. 53
(f) validity/invalidity of acceptance/non-acceptance for union membership;

Respondents Bayer, Lonishen and Amistoso, contend that their acts cannot constitute unfair labor practice as the same did not involve gross violations in
(g) validity/invalidity of impeachment/expulsion of union and workers’ association officers and members; the economic provisions of the CBA, citing the provisions of Articles 248 (1) and 26154 of the Labor Code, as amended.55 Their argument is, however,
misplaced.
(h) validity/invalidity of voluntary recognition;
Indeed, in Silva v. National Labor Relations Commission,56 we explained the correlations of Article 248 (1) and Article 261 of the Labor Code to mean that
(i) opposition to application for union and CBA registration; for a ULP case to be cognizable by the Labor Arbiter, and for the NLRC to exercise appellate jurisdiction thereon, the allegations in the complaint must
show prima facie the concurrence of two things, namely: (1) gross violation of the CBA; and (2) the violation pertains to the economic provisions of the
CBA.57
(j) violations of or disagreements over any provision in a union or workers’ association constitution and by-laws;

This pronouncement in Silva, however, should not be construed to apply to violations of the CBA which can be considered as gross violations per se,
(k) disagreements over chartering or registration of labor organizations and collective bargaining agreements; such as utter disregard of the very existence of the CBA itself, similar to what happened in this case. When an employer proceeds to negotiate with a
splinter union despite the existence of its valid CBA with the duly certified and exclusive bargaining agent, the former indubitably abandons its recognition
(l) violations of the rights and conditions of union or workers’ association membership; of the latter and terminates the entire CBA.

(m) violations of the rights of legitimate labor organizations, except interpretation of collective bargaining agreements; Respondents cannot claim good faith to justify their acts. They knew that Facundo’s group represented the duly-elected officers of EUBP. Moreover, they
were cognizant of the fact that even the DOLE Secretary himself had recognized the legitimacy of EUBP’s mandate by rendering an arbitral award
ordering the signing of the 1997-2001 CBA between Bayer and EUBP. Respondents were likewise well-aware of the pendency of the intra-union dispute
(n) such other disputes or conflicts involving the rights to self-organization, union membership and collective bargaining – case, yet they still proceeded to turn over the collected union dues to REUBP and to effusively deal with Remigio. The totality of respondents’ conduct,
therefore, reeks with anti-EUBP animus.
(1) between and among legitimate labor organizations;
Bayer, Lonishen and Amistoso argue that the case is already moot and academic following the lapse of the 1997-2001 CBA and their renegotiation with
(2) between and among members of a union or workers’ association. EUBP for the 2006-2007 CBA. They also reason that the act of the company in negotiating with EUBP for the 2006-2007 CBA is an obvious recognition
on their part that EUBP is now the certified collective bargaining agent of its rank-and-file employees.58
Section 2. Coverage. – Other related labor relations disputes shall include any conflict between a labor union and the employer or any individual, entity or
group that is not a labor organization or workers’ association. This includes: (1) cancellation of registration of unions and workers’ associations; and (2) a We do not agree. First, a legitimate labor organization cannot be construed to have abandoned its pending claim against the management/employer by
petition for interpleader. returning to the negotiating table to fulfill its duty to represent the interest of its members, except when the pending claim has been expressly waived or
compromised in its subsequent negotiations with the management. To hold otherwise would be tantamount to subjecting industrial peace to the
precondition that previous claims that labor may have against capital must first be waived or abandoned before negotiations between them may resume.
It is clear from the foregoing that the issues raised by petitioners do not fall under any of the aforementioned circumstances constituting an intra-union Undoubtedly, this would be against public policy of affording protection to labor and will encourage scheming employers to commit unlawful acts without
dispute. More importantly, the petitioners do not seek a determination of whether it is the Facundo group (EUBP) or the Remigio group (REUBP) which is fear of being sanctioned in the future.1avvphi1
the true set of union officers. Instead, the issue raised pertained only to the validity of the acts of management in light of the fact that it still has an existing
Labor Relations Set IV * Takata v BLR to Silang v Coa * Page 14 of 29

Second, that the management of Bayer decided to recognize EUBP as the certified collective bargaining agent of its rank-and-file employees for During the pendency of the controversy, Digitel Service, Inc. (Digiserv), a non-profit enterprise engaged in call center servicing, filed with the Department
purposes of its 2006-2007 CBA negotiations is of no moment. It did not obliterate the fact that the management of Bayer had withdrawn its recognition of of Labor and Employment (DOLE) an Establishment Termination Report stating that it will cease its business operation. The closure affected at least 100
EUBP and supported REUBP during the tumultuous implementation of the 1997-2001 CBA. Such act of interference which is violative of the existing CBA employees, 42 of whom are members of the herein respondent Union.
with EUBP led to the filing of the subject complaint.
Alleging that the affected employees are its members and in reaction to Digiserv’s action, Esplana and his group filed another Notice of Strike for union
On the matter of damages prayed for by the petitioners, we have held that as a general rule, a corporation cannot suffer nor be entitled to moral busting, illegal lock-out, and violation of the assumption order.
damages. A corporation, and by analogy a labor organization, being an artificial person and having existence only in legal contemplation, has no feelings,
no emotions, no senses; therefore, it cannot experience physical suffering and mental anguish. Mental suffering can be experienced only by one having a
On 23 May 2005, the Secretary of Labor ordered the second notice of strike subsumed by the previous Assumption Order. 5
nervous system and it flows from real ills, sorrows, and griefs of life – all of which cannot be suffered by an artificial, juridical person. 59 A fortiori, the prayer
for exemplary damages must also be denied.60 Nevertheless, we find it in order to award (1) nominal damages in the amount of P250,000.00 on the basis
of our ruling in De La Salle University v. De La Salle University Employees Association (DLSUEA-NAFTEU)61 and Article 2221,62 and (2) attorney’s fees Meanwhile, on 14 March 2005, Digitel filed a petition with the Bureau of Labor Relations (BLR) seeking cancellation of the Union’s registration on the
equivalent to 10% of the monetary award. The remittance to petitioners of the collected union dues previously turned over to Remigio and Villareal is following grounds: 1) failure to file the required reports from 1994-2004; 2) misrepresentation of its alleged officers; 3) membership of the Union is
likewise in order. composed of rank and file, supervisory and managerial employees; and 4) substantial number of union members are not Digitel employees.6

WHEREFORE, the petition for review on certiorari is PARTLY GRANTED. The Decision dated December 15, 2003 and the Resolution dated March 23, In a Decision dated 11 May 2005, the Regional Director of the DOLE dismissed the petition for cancellation of union registration for lack of merit. The
2004 of the Court of Appeals in CA-G.R. SP No. 73813 are MODIFIED as follows: Regional Director ruled that it does not have jurisdiction over the issue of non-compliance with the reportorial requirements. He also held that Digitel failed
to adduce substantial evidence to prove misrepresentation and the mixing of non-Digitel employees with the Union. Finally, he declared that the inclusion
of supervisory and managerial employees with the rank and file employees is no longer a ground for cancellation of the Union’s certificate of registration.7
1) Respondents Bayer Phils., Dieter J. Lonishen and Asuncion Amistoso are found LIABLE for Unfair Labor Practice, and are hereby ORDERED to remit
to petitioners the amount of P254,857.15 representing the collected union dues previously turned over to Avelina Remigio and Anastacia Villareal. They
are likewise ORDERED to pay petitioners nominal damages in the amount of P250,000.00 and attorney’s fees equivalent to 10% of the monetary award; The appeal filed by Digitel with the BLR was eventually dismissed for lack of merit in a Resolution dated 9 March 2007, thereby affirming the 11 May 2005
and Decision of the Regional Director.

2) The complaint, as against respondents Remigio and Villareal. is DISMISSED due to the lack of jurisdiction of the Labor Arbiter and the NLRC, the CA-G.R. SP No. 91719
complaint being in the nature of an intra-union dispute.No pronouncement as to costs.SO ORDERED
Republic of the Philippines
SUPREME COURT In an Order dated 13 July 2005, the Secretary of Labor directed Digitel to commence the CBA negotiation with the Union. Thus:
Manila
SECOND DIVISION WHEREFORE, all the foregoing premises considered, this Office hereby orders:
G.R. Nos. 184903 October 10, 2012
DIGITAL TELECOMMUNICATIONS PHILIPPINES, INC., Petitioner,
vs. 1. DIGITEL to commence collective bargaining negotiation with DEU without further delay; and,
DIGITEL EMPLOYEES UNION (DEU), ARCELO RAFAEL A. ESPLANA, ALAN D. LICANDO, FELICITO C. ROMERO, JR., ARNOLD D. GONZALES,
REYNEL FRANCISCO B. GARCIA, ZOSIMO B. PERALTA, REGINO T. UNIDAD and JIM L. JAVIER, Respondents. 2. The issue of unfair labor practice, consisting of union-busting, illegal termination/lockout and violation of the assumption of jurisdiction, specifically the
DECISION return-to-work aspect of the 10 March 2005 and 03 June 2005 orders, be CERTIFIED for compulsory arbitration to the NLRC. 8
PEREZ, J.:

This treats of the petition for review filed by Digital Telecommunications Philippines, Inc. (Digitel) assailing the 18 June 2008 Decision 1 and 9 October Digitel moved for reconsideration on the contention that the pendency of the petition for cancellation of the Union’s certificate of registration is a
2008 Resolution of the Court of Appeals 10th Division in CA-G.R. SP No. 91719, which affirms the Order of the Secretary of Labor and Employment prejudicial question that should first be settled before the DOLE could order the parties to bargain collectively. On 19 August 2005, then Acting Secretary
directing Digitel to commence Collective Bargaining Agreement (CBA) negotiations and in CA-G.R. SP No. 94825, which declares the dismissal of Manuel G. Imson of DOLE denied the motion for reconsideration, affirmed the 13 July 2005 Order and reiterated the order directing parties to commence
affected Digitel employees as illegal. collective bargaining negotiations.9

The facts, as borne by the records, follow. On 14 October 2005, Digitel filed a petition, docketed as CA-G.R. SP No. 91719, before the Court of Appeals assailing the 13 July and 19 August 2005
Orders of the DOLE Secretary and attributing grave abuse of discretion on the part of the DOLE Secretary for ordering Digitel to commence bargaining
negotiations with the Union despite the pendency of the issue of union legitimacy.
By virtue of a certification election, Digitel Employees Union (Union) became the exclusive bargaining agent of all rank and file employees of Digitel in
1994. The Union and Digitel then commenced collective bargaining negotiations which resulted in a bargaining deadlock. The Union threatened to go on
strike, but then Acting Labor Secretary Bienvenido E. Laguesma assumed jurisdiction over the dispute and eventually directed the parties to execute a CA-G.R. SP No. 94825
CBA.2
In accordance with the 13 July 2005 Order of the Secretary of Labor, the unfair labor practice issue was certified for compulsory arbitration before the
However, no CBA was forged between Digitel and the Union. Some Union members abandoned their employment with Digitel. The Union later became NLRC, which, on 31 January 2006, rendered a Decision dismissing the unfair labor practice charge against Digitel but declaring the dismissal of the 13
dormant. employees of Digiserv as illegal and ordering their reinstatement. The Union manifested that out of 42 employees, only 13 remained, as most had already
accepted separation pay. The dispositive portion of the Decision reads:

Ten (10) years thereafter or on 28 September 2004, Digitel received from Arceo Rafael A. Esplana (Esplana), who identified himself as President of the
Union, a letter containing the list of officers, CBA proposals and ground rules.3 The officers were respondents Esplana, Alan D. Licando (Vice-President), WHEREFORE, premises considered, the charge of unfair labor practice is hereby DISMISSED for lack of merit. However, the dismissal of the remaining
Felicito C. Romero, Jr. (Secretary), Arnold D. Gonzales (Treasurer), Reynel Francisco B. Garcia (Auditor), Zosimo B. Peralta (PRO), Regino T. Unidad thirteen (13) affected employees is hereby declared illegal and DIGITEL is hereby ORDERED to reinstate them to their former position with full
(Sgt. at Arms), and Jim L. Javier (Sgt. at Arms). backwages up to the time they are reinstated, computed as follows:

Digitel was reluctant to negotiate with the Union and demanded that the latter show compliance with the provisions of the Union’s Constitution and By- x x x x.10
laws on union membership and election of officers.
Upon motion for reconsideration filed by Digitel, four (4) affected employees, namely Ma. Loreta Eser, Marites Jereza, Leonore Tuliao and Aline G.
On 4 November 2004, Esplana and his group filed a case for Preventive Mediation before the National Conciliation and Mediation Board based on Quillopras, were removed from entitlement to the awards pursuant to the deed of quitclaim and release which they all signed. 11
Digitel’s violation of the duty to bargain. On 25 November 2004, Esplana filed a notice of strike.
In view of this unfavorable decision, Digitel filed another petition on 9 June 2006 in CA-G.R. SP No. 94825 before the Court of Appeals, challenging the
On 10 March 2005, then Labor Secretary Patricia A. Sto. Tomas issued an Order4 assuming jurisdiction over the labor dispute. above NLRC Decision and Resolution and arguing mainly that Digiserv employees are not employees of Digitel.
Labor Relations Set IV * Takata v BLR to Silang v Coa * Page 15 of 29

Ruling of the Court of Appeals Manggagawa sa Pacific Plastic v. Hon. Laguesma, 17 it was pointed out at the time of the filing of the petition for certification election – or a CBA process
as in the instant case – the union still had the personality to file a petition for certification − or to ask for a CBA negotiation – as in the present case.
On 18 June 2008, the Tenth Division of the Court of Appeals consolidated the two petitions in CA-G.R. SP No. 91719 and CA-G.R. SP No. 94825, and
disposed as follows: Digiserv is a labor-only contractor.

WHEREFORE, the petition in CA-G.R. SP No. 91719 is DISMISSED. The July 13, 2005 Order and the August 19, 2005 Resolution of the DOLE Labor-only contracting is expressly prohibited by our labor laws. Article 106 of the Labor Code defines labor-only contracting as "supplying workers to an
Secretary are AFFIRMED in toto. With costs. employer [who] does not have substantial capital or investment in the form of tools, equipment, machineries, work premises, among others, and the
workers recruited and placed by such person are performing activities which are directly related to the principal business of such employer."
The petition in CA-G.R. SP No. 94825 is partially GRANTED, with the effect that the assailed dispositions must be MODIFIED, as follows:
Section 5, Rule VIII-A, Book III of the Omnibus Rules Implementing the Labor Code (Implementing Rules), as amended by Department Order No. 18-02,
expounds on the prohibition against labor-only contracting, thus:
1) In addition to the order directing reinstatement and payment of full backwages to the nine (9) affected employees, Digital Telecommunications
Philippines, Inc. is furthered ORDERED, should reinstatement is no longer feasible, to pay separation pay equivalent to one (1) month pay, or one-half
(1/2) month pay for every year of service, whichever is higher. Section 5. Prohibition against labor-only contracting. − Labor-only contracting is hereby declared prohibited. For this purpose, labor-only contracting shall
refer to an arrangement where the contractor or subcontractor merely recruits, supplies or places workers to perform a job, work or service for a principal,
and any of the following elements are present:
2) The one hundred thousand (PhP 100,000.00) peso-fine imposed on Digital Telecommunications Philippines, Inc. is DELETED. No costs.12

i) The contractor or subcontractor does not have substantial capital or investment which relates to the job, work or service to be performed and the
The Court of Appeals upheld the Secretary of Labor’s Order for Digitel to commence CBA negotiations with the Union and emphasized that the pendency
employees recruited, supplied or placed by such contractor or subcontractor are performing activities which are directly related to the main business of
of a petition for the cancellation of a union’s registration does not bar the holding of negotiations for a CBA. The Court of Appeals sustained the finding
the principal; or
that Digiserv is engaged in labor-only contracting and that its employees are actually employees of Digitel.

ii) The contractor does not exercise the right to control over the performance of the work of the contractual employee.
Digitel filed a motion for reconsideration but was denied in a Resolution dated 9 October 2008.

The foregoing provisions shall be without prejudice to the application of Article 248 (c) of the Labor Code, as amended.
Hence, this petition for review on certiorari.

xxxx
Digitel argues that the Court of Appeals seriously erred when it condoned the act of the Secretary of Labor in issuing an assumption order despite the
pendency of an appeal on the issue of union registration. Digitel maintains that it cannot be compelled to negotiate with a union for purposes of collective
bargaining when the very status of the same as the exclusive bargaining agent is in question. The "right to control" shall refer to the right reserved to the person for whom, the services of the contractual workers are performed, to determine not only
the end to be achieved, but also the manner and means to be used in reaching that end.
Digitel insists that had the Court of Appeals considered the nature of the activities performed by Digiserv, it would reach the conclusion that Digiserv is a
legitimate contractor. To bolster its claim, Digitel asserts that the affected employees are registered with the Social Security System, Pag-ibig, Bureau of The law and its implementing rules allow contracting arrangements for the performance of specific jobs, works or services. Indeed, it is management
Internal Revenue and Philhealth with Digiserv as their employer. Digitel further contends that assuming that the affected Digiserv employees are prerogative to farm out any of its activities, regardless of whether such activity is peripheral or core in nature. However, in order for such outsourcing to be
employees of Digitel, they were nevertheless validly dismissed on the ground of closure of a department or a part of Digitel’s business operation. valid, it must be made to an independent contractor because the current labor rules expressly prohibit labor-only contracting.18

The three issues raised in this petition are: 1) whether the Secretary of Labor erred in issuing the assumption order despite the pendency of the petition After an exhaustive review of the records, there is no showing that first, Digiserv has substantial investment in the form of capital, equipment or tools.
for cancellation of union registration; 2) whether Digiserv is a legitimate contractor; and 3) whether there was a valid dismissal. Under the Implementing Rules, substantial capital or investment refers to "capital stocks and subscribed capitalization in the case of corporations, tools,
equipment, implements, machineries and work premises, actually and directly used by the contractor or subcontractor in the performance or completion
of the job, work or service contracted out." The NLRC, as echoed by the Court of Appeals, did not find substantial Digiserv’s authorized capital stock of
The pendency of a petition
One Million Pesos (P 1,000,000.00). It pointed out that only Two Hundred Fifty Thousand Pesos (P 250,000.00) of the authorized capital stock had been
for cancellation of union
subscribed and only Sixty-Two Thousand Five Hundred Pesos (P 62,500.00) had been paid up. There was no increase in capitalization for the last ten
registration does not preclude
(10) years.19
collective bargaining.

Moreover, in the Amended Articles of Incorporation, as well as in the General Information Sheets for the years 1994, 2001 and 2005, the primary purpose
The first issue raised by Digitel is not novel. It is well-settled that the pendency of a petition for cancellation of union registration does not preclude
of Digiserv is to provide manpower services. In PCI Automation Center, Inc. v. National Labor Relations Commission,20 the Court made the following
collective bargaining.
distinction: "the legitimate job contractor provides services while the labor-only contractor provides only manpower. The legitimate job contractor
undertakes to perform a specific job for the principal employer while the labor-only contractor merely provides the personnel to work for the principal
The 2005 case of Capitol Medical Center, Inc. v. Hon. Trajano13 is apropos. The respondent union therein sent a letter to petitioner requesting a employer." The services provided by employees of Digiserv are directly related to the business of Digitel, as rationalized by the NLRC in this wise:
negotiation of their CBA. Petitioner refused to bargain and instead filed a petition for cancellation of the union’s certificate of registration. Petitioner’s
refusal to bargain forced the union to file a notice of strike. They eventually staged a strike. The Secretary of Labor assumed jurisdiction over the labor
It is undisputed that as early as March 1994, the affected employees, except for two, were already performing their job as Traffic Operator which was later
dispute and ordered all striking workers to return to work. Petitioner challenged said order by contending that its petition for cancellation of union’s
renamed as Customer Service Representative (CSR). It is equally undisputed that all throughout their employment, their function as CSR remains the
certificate of registration involves a prejudicial question that should first be settled before the Secretary of Labor could order the parties to bargain
same until they were terminated effective May 30, 2005. Their long period of employment as such is an indication that their job is directly related to the
collectively. When the case eventually reached this Court, we agreed with the Secretary of Labor that the pendency of a petition for cancellation of union
main business of DIGITEL which is telecommunications. Because, if it was not, DIGITEL would not have allowed them to render services as Customer
registration does not preclude collective bargaining, thus:
Service Representative for such a long period of time.21

That there is a pending cancellation proceeding against the respondent Union is not a bar to set in motion the mechanics of collective bargaining. If a
Furthermore, Digiserv does not exercise control over the affected employees. The NLRC highlighted the fact that Digiserv shared the same Human
certification election may still be ordered despite the pendency of a petition to cancel the union’s registration certificate (National Union of Bank
Resources, Accounting, Audit and Legal Departments with Digitel which manifested that it was Digitel who exercised control over the performance of the
Employees vs. Minister of Labor, 110 SCRA 274), more so should the collective bargaining process continue despite its pendency. We must emphasize
affected employees. The NLRC also relied on the letters of commendation, plaques of appreciation and certification issued by Digitel to the Customer
that the majority status of the respondent Union is not affected by the pendency of the Petition for Cancellation pending against it. Unless its certificate of
Service Representatives as evidence of control.
registration and its status as the certified bargaining agent are revoked, the Hospital is, by express provision of the law, duty bound to collectively bargain
with the Union.14
Considering that Digiserv has been found to be engaged in labor-only contracting, the dismissed employees are deemed employees of Digitel.
Trajano was reiterated in Legend International Resorts Limited v. Kilusang Manggagawa ng Legenda (KML-Independent).15 Legend International Resorts
reiterated the rationale for allowing the continuation of either a CBA process or a certification election even during the pendency of proceedings for the
cancellation of the union’s certificate of registration. Citing the cases of Association of Court of Appeals Employees v. Ferrer- Calleja16 and Samahan ng
Labor Relations Set IV * Takata v BLR to Silang v Coa * Page 16 of 29

Section 7 of the Implementing Rules holds that labor-only contracting would give rise to: (1) the creation of an employer-employee relationship between affected employees were employed. Digitel cited the decline in the volume of transaction of operator-assisted call services as supported by Financial
the principal and the employees of the contractor or sub-contractor; and (2) the solidary liability of the principal and the contractor to the employees in the Statements for the years 2003 and 2004, during which Digiserv incurred a deficit of P 163,624.00 and P164,055.00, respectively.25 All affected employees
event of any violation of the Labor Code. working under Digiserv were served with individual notices of termination. DOLE was likewise served with the corresponding notice. All affected
employees were offered separation pay. Only 9 out of the 45 employees refused to accept the separation pay and chose to contest their dismissal before
this Court.
Accordingly, Digitel is considered the principal employer of respondent employees.

The fifth element regarding the criteria to be observed by Digitel clearly does not apply because all employees under Digiserv were dismissed. The
The affected employees were
instant case is all about the fourth element, that is, whether or not the affected employees were dismissed in good faith. We find that there was no good
illegally dismissed.
faith in the retrenchment.

In addition to finding that Digiserv is a labor-only contractor, records teem with proof that its dismissed employees are in fact employees of Digitel. The
Prior to the cessation of Digiserv’s operations, the Secretary of Labor had issued the first assumption order to enjoin an impending strike. When Digiserv
NLRC enumerated these evidences, thus:
effected the dismissal of the affected employees, the Union filed another notice of strike. Significantly, the Secretary of Labor ordered that the second
notice of strike be subsumed by the previous assumption order. Article 263(g) of the Labor Code provides:
That the remaining thirteen (13) affected employees are indeed employees of DIGITEL is sufficiently established by the facts and evidence on record.
When, in his opinion, there exists a labor dispute causing or likely to cause a strike or lockout in an industry indispensable to the national interest, the
It is undisputed that the remaining affected employees, except for two (2), were already hired by DIGITEL even before the existence of DIGISERV. (The Secretary of Labor and Employment may assume jurisdiction over the dispute and decide it or certify the same to the Commission for compulsory
other two (2) were hired after the existence of DIGISERV). The UNION submitted a sample copy of their appointment paper (Annex "A" of UNION’s arbitration. Such assumption or certification shall have the effect of automatically enjoining the intended or impending strike or lockout as specified in the
Position Paper, Records, Vol. 1, p. 100) showing that they were appointed on March 1, 1994, almost three (3) months before DIGISERV came into assumption or certification order. If one has already taken place at the time of assumption or certification, all striking or locked out employees shall
existence on May 30, 1994 (Annex "B", Ibid, Records, Vol. 1, p. 101). On the other hand, not a single appointment paper was submitted by DIGITEL immediately return to work and the employer shall immediately resume operations and readmit all workers under the same terms and conditions
showing that these remaining affected employees were hired by DIGISERV. prevailing before the strike or lockout. The Secretary of Labor and Employment or the Commission may seek the assistance of law enforcement agencies
to ensure the compliance with this provision as well as with such orders as he may issue to enforce the same.
It is equally undisputed that the remaining, affected employees continuously held the position of Customer Service Representative, which was earlier
known as Traffic Operator, from the time they were appointed on March 1, 1994 until they were terminated on May 30, 2005. The UNION alleges that The effects of the assumption order issued by the Secretary of Labor are two-fold. It enjoins an impending strike on the part of the employees and orders
these Customer Service Representatives were under the Customer Service Division of DIGITEL. The UNION’s allegation is correct. Sample of letter of the employer to maintain the status quo.
commendations issued to Customer Service Representatives (Annexes "C" and "C-1" of UNION’s Position Paper, Records, p. 100 and 111) indeed show
that DIGITEL has a Customer Service Division which handles its Call Center operations.
There is no doubt that Digitel defied the assumption order by abruptly closing down Digiserv. The closure of a department is not illegal per se. What
makes it unlawful is when the closure is undertaken in bad faith. In St. John Colleges, Inc. v. St. John Academy Faculty and Employees Union,26 bad faith
Further, the Certificates issued to Customer Service Representative likewise show that they are employees of DIGITEL (Annexes "C-5", "C-6" - "C-7" of was evidenced by the timing of and reasons for the closure and the timing of and reasons for the subsequent opening. There, the collective bargaining
UNION’s Position Paper, Records, Vol. 1, pp. 115 to 117), Take for example the "Service Award" issued to Ma. Loretta C. Esen, one of the remaining negotiations between St. John and the Union resulted in a bargaining deadlock that led to the filing of a notice of strike. The labor dispute was referred to
affected employees (Annex "C-5", Supra). The "Service Award" was signed by the officers of DIGITEL – the VP-Customer Services Division, the VP- the Secretary of Labor who assumed jurisdiction.
Human Resources Division and the Group Head-Human Resources Division. It was issued by DIGITEL to Esen thru the above named officers "In
recognition of her seven (7) years continuous and valuable contributions to the achievement of Digitel’s organization objectives". It cannot be gainsaid
Pending resolution of the dispute, St. John closed the school prompting the Union to file a complaint for illegal dismissal and unfair labor practice. The
that it is only the employer that issues service award to its employees. 22 (Emphasis not supplied)
Union members alleged that the closure of the high school was done in bad faith in order to get rid of the Union and render useless any decision of the
SOLE on the CBA deadlocked issues. We held that closure was done to defeat the affected employees’ security of tenure, thus:
As a matter of fact, even before the incorporation of Digiserv, the affected employees were already employed by Digitel as Traffic Operators, later
renamed as Customer Service Representatives.
The determination of whether SJCI acted in bad faith depends on the particular facts as established by the evidence on record. Bad faith is, after all, an
inference which must be drawn from the peculiar circumstances of a case. The two decisive factors in determining whether SJCI acted in bad faith are (1)
As an alternative argument, Digitel maintains that the affected employees were validly dismissed on the grounds of closure of Digiserv, a department the timing of, and reasons for the closure of the high school, and (2) the timing of, and the reasons for the subsequent opening of a college and
within Digitel. elementary department, and, ultimately, the reopening of the high school department by SJCI after only one year from its closure.

In the recent case of Waterfront Cebu City Hotel v. Jimenez,23 we referred to the closure of a department or division of a company as retrenchment. The Prior to the closure of the high school by SJCI, the parties agreed to refer the 1997 CBA deadlock to the SOLE for assumption of jurisdiction under Article
dismissed employees were undoubtedly retrenched with the closure of Digiserv. 263 of the Labor Code. As a result, the strike ended and classes resumed. After the SOLE assumed jurisdiction, it required the parties to submit their
respective position papers. However, instead of filing its position paper, SJCI closed its high school, allegedly because of the "irreconcilable differences
between the school management and the Academy’s Union particularly the safety of our students and the financial aspect of the ongoing CBA
For a valid retrenchment, the following elements must be present: negotiations." Thereafter, SJCI moved to dismiss the pending labor dispute with the SOLE contending that it had become moot because of the closure.
Nevertheless, a year after said closure, SJCI reopened its high school and did not rehire the previously terminated employees.
(1) That retrenchment is reasonably necessary and likely to prevent business losses which, if already incurred, are not merely de minimis, but substantial,
serious, actual and real, or if only expected, are reasonably imminent as perceived objectively and in good faith by the employer; Under these circumstances, it is not difficult to discern that the closure was done to defeat the parties’ agreement to refer the labor dispute to the SOLE;
to unilaterally end the bargaining deadlock; to render nugatory any decision of the SOLE; and to circumvent the Union’s right to collective bargaining and
(2) That the employer served written notice both to the employees and to the Department of Labor and Employment at least one month prior to the its members’ right to security of tenure. By admitting that the closure was due to irreconcilable differences between the Union and school management,
intended date of retrenchment; specifically, the financial aspect of the ongoing CBA negotiations, SJCI in effect admitted that it wanted to end the bargaining deadlock and eliminate the
problem of dealing with the demands of the Union. This is precisely what the Labor Code abhors and punishes as unfair labor practice since the net effect
is to defeat the Union’s right to collective bargaining.27 (Emphasis not supplied)
(3) That the employer pays the retrenched employees separation pay equivalent to one (1) month pay or at least ½ month pay for every year of service,
whichever is higher;
As in St. John, bad faith was manifested by the timing of the closure of Digiserv and the rehiring of some employees to Interactive Technology Solutions,
Inc. (I-tech), a corporate arm of Digitel. The assumption order directs employees to return to work, and the employer to reinstate the employees. The
(4) That the employer exercises its prerogative to retrench employees in good faith for the advancement of its interest and not to defeat or circumvent the existence of the assumption order should have prompted Digitel to observe the status quo. Instead, Digitel proceeded to close down Digiserv. The
employees’ right to security of tenure; and Secretary of Labor had to subsume the second notice of strike in the assumption order. This order notwithstanding, Digitel proceeded to dismiss the
employees.
(5) That the employer used fair and reasonable criteria in ascertaining who would be dismissed and who would be retained among the employees, such
as status, efficiency, seniority, physical fitness, age, and financial hardship for certain workers.24 The timing of the creation of I-tech is dubious. It was incorporated on 18 January 2005 while the labor dispute within Digitel was pending. I-tech’s primary
purpose was to provide call center/customer contact service, the same service provided by Digiserv. It conducts its business inside the Digitel office at
Only the first 3 elements of a valid retrenchment had been here satisfied. Indeed, it is management prerogative to close a department of the company. 110 E. Rodriguez Jr. Avenue, Bagumbayan, Quezon City. The former head of Digiserv, Ms. Teresa Taniega, is also an officer of I-tech. Thus, when
Digitel’s decision to outsource the call center operation of the company is a valid reason to close down the operations of a department under which the Digiserv was closed down, some of the employees presumably non-union members were rehired by I-tech.
Labor Relations Set IV * Takata v BLR to Silang v Coa * Page 17 of 29

Thus, the closure of Digiserv pending the existence of an assumption order coupled with the creation of a new corporation performing similar functions as WHEREFORE, the Petition is DENIED. The Decision of the Court of Appeals in CA-G.R. SP No. 91719 isAFFIRMED, while the Decision in CA-G.R. SP
Digiserv leaves no iota of doubt that the target of the closure are the union member-employees. These factual circumstances prove that Digitel terminated No. 94825 declaring the dismissal of affected union member-employees as illegal is MODIFIED to include the payment of moral and exemplary damages
the services of the affected employees to defeat their security of tenure. The termination of service was not a valid retrenchment; it was an illegal in amount of P10,000.00 and P 5,000.00, respectively, to each of the thirteen (13) illegally dismissed union-member employees.
dismissal of employees.
Petitioner Digital Telecommunications Philippines, Inc. is ORDERED to pay the affected employees backwages and separation pay equivalent to one (1)
It needs to be mentioned too that the dismissal constitutes an unfair labor practice under Article 248(c) of the Labor Code which refers to contracting out month salary, or one-half (1/2) month pay for every year of service, whichever is higher.Let this case be REMANDED to the Labor Arbiter for the
services or functions being performed by union members when such will interfere with, restrain or coerce employees in the exercise of their rights to self- computation of monetary claims due to the affected employees.SO ORDERED.
organization. At the height of the labor dispute, occasioned by Digitel’s reluctance to negotiate with the Union, I-tech was formed to provide, as it did Republic of the Philippines
provide, the same services performed by Digiserv, the Union members’ nominal employer. SUPREME COURT
Manila
THIRD DIVISION
Under Article 279 of the Labor Code, an illegally dismissed employee is entitled to backwages and reinstatement. Where reinstatement is no longer viable
G.R. No. 160352 July 23, 2008
as an option, as in this case where Digiserv no longer exists, separation pay equivalent to one (1) month salary, or one-half (1/2) month pay for every
REPUBLIC OF THE PHILIPPINES, represented by Department of Labor and Employment (DOLE), Petitioner,
year of service, whichever is higher, should be awarded as an alternative.28 The payment of separation pay is in addition to payment of backwages. 29
vs.
KAWASHIMA TEXTILE MFG., PHILIPPINES, INC., Respondent.
Indeed, while we have found that the closure of Digiserv was undertaken in bad faith, badges thereof evident in the timing of Digiserv’s closure, hand in DECISION
hand, with I-tech’s creation, the closure remains a foregone conclusion. There is no finding, and the Union makes no such assertion, that Digiserv and I- AUSTRIA-MARTINEZ, J.:
tech are one and the same corporation. The timing of Digiserv’s closure and I-tech’s ensuing creation is doubted, not the legitimacy of I-tech as a
business process outsourcing corporation providing both inbound and outbound services to an expanded local and international clientele. 30 The Republic of the Philippines assails by way of Petition for Review on Certiorari under Rule 45 of the Rules of Court, the December 13, 2002
Decision1 of the Court of Appeals (CA), which reversed the August 18, 2000 Decision2 of the Department of Labor and Employment (DOLE), and
reinstated the May 17, 2000 Order3 of Med-Arbiter Anastacio L. Bactin, dismissing the petition of Kawashima Free Workers Union-PTGWO Local Chapter
The finding of unfair labor practice hinges on Digitel’s contracting-out certain services performed by union member-employees to interfere with, restrain or No. 803 (KFWU) for the conduct of a certification election in Kawashima Textile Mfg. Phils., Inc. (respondent); and the October 7, 2003 CA
coerce them in the exercise of their right to self-organization.
Resolution4 which denied the motion for reconsideration.

We have no basis to direct reinstatement of the affected employees to an ostensibly different corporation. The surrounding circumstance of the creation The relevant facts are of record.
of I-tech point to bad faith on the part of Digitel, as well as constitutive of unfair labor practice in targeting the dismissal of the union member-employees.
However, this bad faith does not contradict, much less negate, the impossibility of the employees’ reinstatement because Digiserv has been closed and
no longer exists. On January 24, 2000, KFWU filed with DOLE Regional Office No. IV, a Petition for Certification Election to be conducted in the bargaining unit composed
of 145 rank-and-file employees of respondent.5 Attached to its petition are a Certificate of Creation of Local/Chapter6 issued on January 19, 2000 by
DOLE Regional Office No. IV, stating that it [KFWU] submitted to said office a Charter Certificate issued to it by the national federation Phil. Transport &
Even if it is a possibility that I-tech, as though Digitel, can absorb the dismissed union member-employees as I-tech was incorporated during the time of General Workers Organization (PTGWO), and a Report of Creation of Local/Chapter. 7
the controversy with the same primary purpose as Digiserv, we would be hard pressed to mandate the dismissed employees’ reinstatement given the
lapse of more than seven (7) years.
Respondent filed a Motion to Dismiss8 the petition on the ground that KFWU did not acquire any legal personality because its membership of mixed rank-
31 and-file and supervisory employees violated Article 245 of the Labor Code, and its failure to submit its books of account contravened the ruling of the
This length of time from the date the incident occurred to its Resolution coupled with the demonstrated litigiousness of the disputants: (1) with all sorts of Court in Progressive Development Corporation v. Secretary, Department of Labor and Employment. 9
allegations thrown by either party against the other; (2) the two separate filings of a notice of strike by the Union; (3) the Assumption Orders of the DOLE;
(4) our own finding of unfair labor practice by Digitel in targeting the union member-employees, abundantly show that the relationship between Digitel and
the union member-employees is strained. Indeed, such discordance between the parties can very well be a necessary consequence of the protracted and In an Order dated May 17, 2000, Med-Arbiter Bactin found KFWU’s legal personality defective and dismissed its petition for certification election, thus:
branched out litigation. We adhere to the oft-quoted doctrine that separation pay may avail in lieu of reinstatement if reinstatement is no longer practical
or in the best interest of the parties.32
We scrutinize the facts and evidences presented by the parties and arrived at a decision that at least two (2) members of [KFWU], namely: Dany I.
Fernandez and Jesus R. Quinto, Jr. are supervisory employees, having a number of personnel under them. Being supervisory employees, they are
Under the doctrine of strained relations, the payment of separation pay is considered an acceptable alternative to reinstatement when the latter option is prohibited under Article 245 of the Labor Code, as amended, to join the union of the rank and file employees. Dany I. Fernandez and Jesus R. Quinto, Jr.,
no longer desirable or viable. On one hand, such payment liberates the employee from what could be a highly oppressive work environment. On the other Chief Engineers of the Maintenance and Manufacturing Department, respectively, act as foremen to the line engineers, mechanics and other non-skilled
hand, it releases the employer from the grossly unpalatable obligation of maintaining in its employ a worker it could no longer trust.33 workers and responsible [for] the preparation and organization of maintenance shop fabrication and schedules, inventory and control of materials and
supplies and tasked to implement training plans on line engineers and evaluate the performance of their subordinates. The above-stated actual functions
of Dany I. Fernandez and Jesus R. Quinto, Jr. are clear manifestation that they are supervisory employees.
Finally, an illegally dismissed employee should be awarded moral and exemplary damages as their dismissal was tainted with unfair labor
practice.34 Depending on the factual milieu, jurisprudence has awarded varying amounts as moral and exemplary damages to illegally dismissed
employees when the dismissal is attended by bad faith or fraud; or constitutes an act oppressive to labor; or is done in a manner contrary to good morals, xxxx
good customs or public policy; or if the dismissal is effected in a wanton, oppressive or malevolent manner. 351âwphi1
Since petitioner’s members are mixture of rank and file and supervisory employees, petitioner union, at this point [in] time, has not attained
In Nueva Ecija I Electric Cooperative, Inc. (NEECO I) Employees Association v. National Labor Relations Commission, we intoned: the status of a legitimate labor organization. Petitioner should first exclude the supervisory employees from it membership before it can attain
the status of a legitimate labor organization. The above judgment is supported by the decision of the Supreme Court in the Toyota Case10 wherein the
High Tribunal ruled:
Unfair labor practices violate the constitutional rights of workers and employees to self-organization, are inimical to the legitimate interests of both labor
and management, including their right to bargain collectively and otherwise deal with each other in an atmosphere of freedom and mutual respect; and
disrupt industrial peace and hinder the promotion of healthy and stable labor-management relations. As the conscience of the government, it is the "As respondent union’s membership list contains the names of at least twenty seven (27) supervisory employees in Level Five Positions, the union could
Court’s sworn duty to ensure that none trifles with labor rights. 36 not prior to purging itself of its supervisory employee members, attain the status of a legitimate labor organization. Not being one, it cannot possess the
requisite personality to file a petition for certification election." (Underscoring omitted.)
xxxx
We awarded moral damages in the amount of P 10,000.00 and likewise awarded P 5,000.00 as exemplary damages for each dismissed employee.
Furthermore, the commingling of rank and file and supervisory employees in one (1) bargaining unit cannot be cured in the exclusion-inclusion
In the recent case of Purefoods Corporation v. Nagkakaisang Samahang Manggagawa ng Purefoods Rank-and-File,37 we awarded the aggregate proceedings [at] the pre-election conference. The above ruling is supported by the Decision of the Supreme Court in Dunlop Slazenger (Phils.), Inc. vs.
amount of P 500,000.00 as moral and exemplary damages to the illegally dismissed union member-employees which exact number was undetermined. Honorable Secretary of Labor and Employment, et al., G.R. No. 131248 dated December 11, 199811 x x x.

xxxx
In the case at hand, with the Union’s manifestation that only 13 employees remain as respondents, as most had already accepted separation pay, and
consistent with our finding that Digitel committed an unfair labor practice in violation of the employees’ constitutional right to self-organization, we deem it WHEREFORE, premises considered, the petition for certification election is hereby dismissed for lack of requisite legal status of petitioner to file this
proper to award each of the illegally dismissed union member-employees the amount of P 10,000.00 and P 5,000.00 as moral and exemplary damages, instant petition.SO ORDERED.12 (Emphasis supplied)
respectively.
Labor Relations Set IV * Takata v BLR to Silang v Coa * Page 18 of 29

On the basis of the aforecited decision, respondent filed with DOLE Regional Office No. IV a Petition for Cancellation of Charter/Union Registration of The key to the closure that petitioner seeks could have been Republic Act (R.A.) No. 9481. 25 Sections 8 and 9 thereof provide:
KFWU,13 the final outcome of which, unfortunately, cannot be ascertained from the records.
Section 8. Article 245 of the Labor Code is hereby amended to read as follows:
Meanwhile, KFWU appealed14 to the DOLE which issued a Decision on August 18, 2000, the dispositive portion of which reads:
"Art. 245. Ineligibility of Managerial Employees to Join any Labor Organization; Right of Supervisory Employees. - Managerial employees are not eligible
WHEREFORE, the appeal is GRANTED. The Order dated 17 May 2000 of the Med-Arbiter is REVERSED and SET ASIDE. Accordingly, let the entire to join, assist or form any labor organization. Supervisory employees shall not be eligible for membership in the collective bargaining unit of the rank-and-
records of the case be remanded to the office of origin for the immediate conduct of certification election, subject to the usual pre-election conference, file employees but may join, assist or form separate collective bargaining units and/or legitimate labor organizations of their own. The rank and file union
among the rank-and-file employees of Kawashima Textile Manufacturing Philippines, Inc. with the following choices: and the supervisors' union operating within the same establishment may join the same federation or national union."

1. Kawashima Free Workers Union-PTGWO Local Chapter No. 803; and Section 9. A new provision, Article 245-A is inserted into the Labor Code to read as follows:
2. No union.
"Art. 245-A. Effect of Inclusion as Members of Employees Outside the Bargaining Unit. - The inclusion as union members of employees outside the
Pursuant to Rule XI, Section 11.1 of the New Implementing Rules, the employer is hereby directed to submit to the office of origin the certified list of bargaining unit shall not be a ground for the cancellation of the registration of the union. Said employees are automatically deemed removed
current employees in the bargaining unit for the last three months prior to the issuance of this decision.SO DECIDED. 15 from the list of membership of said union." (Emphasis supplied)

The DOLE held that Med-Arbiter Bactin's reliance on the decisions of the Court in Toyota Motor Philippines Corporation v. Toyota Motor Philippines Moreover, under Section 4, a pending petition for cancellation of registration
Corporation Labor Union16 and Dunlop Slazenger, Inc. v. Secretary of Labor and Employment 17 was misplaced, for while Article 245 declares supervisory
employees ineligible for membership in a labor organization for rank-and-file employees, the provision did not state the effect of such prohibited
will not hinder a legitimate labor organization from initiating a certification election, viz:
membership on the legitimacy of the labor organization and its right to file for certification election. Neither was such mixed membership a ground for
cancellation of its registration. Section 11, Paragraph II, Rule XI of Department Order No. 9 "provides for the dismissal of a petition for certification
election based on lack of legal personality of a labor organization only on the following grounds: (1) [KFWU] is not listed by the Regional Office or the Sec. 4. A new provision is hereby inserted into the Labor Code as Article 238-A to read as follows:
Bureau of Labor Relations in its registry of legitimate labor organizations; or (2) [KFWU's] legal personality has been revoked or canceled with
finality."18 The DOLE noted that neither ground existed; on the contrary, KFWU's legal personality was well-established, for it held a certificate of creation
"Art. 238-A. Effect of a Petition for Cancellation of Registration. - A petition for cancellation of union registration shall not suspend the proceedings
and had been listed in the registry of legitimate labor organizations.
for certification election nor shall it prevent the filing of a petition for certification election.

As to the failure of KFWU to file its books of account, the DOLE held that such omission was not a ground for revocation of union registration or dismissal
of petition for certification election, for under Section 1, Rule VI of Department Order No. 9, a local or chapter like KFWU was no longer required to file its In case of cancellation, nothing herein shall restrict the right of the union to seek just and equitable remedies in the appropriate courts." (Emphasis
books of account.19 supplied)

Respondent filed a Motion for Reconsideration20 but the DOLE denied the same in its September 28, 2000 Resolution. 21 Furthermore, under Section 12 of R.A. No. 9481, employers have no personality to interfere with or thwart a petition for certification election filed by a
legitimate labor organization, to wit:

However, on appeal by respondent, the CA rendered the December 13, 2002 Decision assailed herein, reversing the August 18, 2000 DOLE Decision,
thus: Sec. 12. A new provision, Article 258-A is hereby inserted into the Labor Code to read as follows:

Since respondent union clearly consists of both rank and file and supervisory employees, it cannot qualify as a legitimate labor organization "Art. 258-A. Employer as Bystander. - In all cases, whether the petition for certification election is filed by an employer or a legitimate labor
imbued with the requisite personality to file a petition for certification election. This infirmity in union membership cannot be corrected in the organization, the employer shall not be considered a party thereto with a concomitant right to oppose a petition for certification election. The
inclusion-exclusion proceedings during the pre-election conference. employer's participation in such proceedings shall be limited to: (1) being notified or informed of petitions of such nature; and (2) submitting
the list of employees during the pre-election conference should the Med-Arbiter act favorably on the petition." (Emphasis supplied)

Finally, contrary to the pronouncement of public respondent, the application of the doctrine enunciated in Toyota Motor Philippines Corporation vs. Toyota
However, R.A. No. 9481 took effect only on June 14, 2007;26 hence, it applies only to labor representation cases filed on or after said date. 27 As the
Motor Philippines Corporation Labor Union was not construed in a way that effectively denies the fundamental right of respondent union to organize and
seek bargaining representation x x x. petition for certification election subject matter of the present petition was filed by KFWU on January 24, 2000, 28 R.A. No. 9481 cannot apply to it. There
may have been curative labor legislations29 that were given retrospective effect,30 but not the aforecited provisions of R.A. No. 9481, for otherwise,
substantive rights and interests already vested would be impaired in the process. 31
For ignoring jurisprudential precepts on the matter, the Court finds that the Undersecretary of Labor, acting under the authority of the Secretary of Labor,
acted with grave abuse of discretion amounting to lack or excess of jurisdiction.
Instead, the law and rules in force at the time of the filing by KFWU of the petition for certification election on January 24, 2000 are R.A. No.
6715,32 amending Book V of Presidential Decree (P.D.) No. 442 (Labor Code), 33as amended, and the Rules and Regulations Implementing R.A. No.
WHEREFORE, premises considered, the Petition is hereby GRANTED. The Decision dated 18 August 2000 of the Undersecretary of Labor, acting under 6715,34 as amended by Department Order No. 9, series of 1997.35
the authority of the Secretary, is hereby REVERSED and SET ASIDE. The Order dated 17 May 2000 of the Med-Arbiter dismissing the petition for
certification election filed by Kawashima Free Workers Union-PTGWO Local Chapter No. 803 is REINSTATED.SO ORDERED.22 (Emphasis supplied)
It is within the parameters of R.A. No. 6715 and the Implementing Rules that the Court will now resolve the two issues raised by petitioner.

KFWU filed a Motion for Reconsideration23 but the CA denied it.


If there is one constant precept in our labor laws – be it Commonwealth Act No. 213 (1936),36 R.A. No. 875 (1953),37 P.D. No. 442 (1974), Executive
Order (E.O.) No. 111 (1986)38 or R.A. No. 6715 (1989) - it is that only a legitimate labor organization may exercise the right to be certified as the exclusive
The Republic of the Philippines (petitioner) filed the present petition to seek closure on two issues: representative of all the employees in an appropriate collective bargaining unit for purposes of collective bargaining. 39 What has varied over the years has
been the degree of enforcement of this precept, as reflected in the shifting scope of administrative and judicial scrutiny of the composition of a labor
organization before it is allowed to exercise the right of representation.
First, whether a mixed membership of rank-and-file and supervisory employees in a union is a ground for the dismissal of a petition for certification
election in view of the amendment brought about by D.O. 9, series of 1997, which deleted the phraseology in the old rule that "[t]he appropriate
bargaining unit of the rank-and-file employee shall not include the supervisory employees and/or security guards;" and One area of contention has been the composition of the membership of a labor organization, specifically whether there is a mingling of supervisory and
rank-and-file employees and how such questioned mingling affects its legitimacy.
Second, whether the legitimacy of a duly registered labor organization can be collaterally attacked in a petition for a certification election through a motion
to dismiss filed by an employer such as Kawashima Textile Manufacturing Phils., Inc. 24 It was in R.A. No. 875, under Section 3, that such questioned mingling was first prohibited,40 to wit:

The petition is imbued with merit.


Labor Relations Set IV * Takata v BLR to Silang v Coa * Page 19 of 29

Sec. 3. Employees’ right to self-organization. – Employees shall have the right to self-organization and to form, join or assist labor organizations of their The petition, when filed by a legitimate labor organization, shall contain, among others:
own choosing for the purpose of collective bargaining through representatives of their own choosing and to engage in concerted activities for the purpose
of collective bargaining and other mutual aid or protection. Individuals employed as supervisors shall not be eligible for membership in a labor
xxxx
organization of employees under their supervision but may form separate organizations of their own. (Emphasis supplied)

(c) description of the bargaining unit which shall be the employer unit unless circumstances otherwise require; and provided further, that the appropriate
Nothing in R.A. No. 875, however, tells of how the questioned mingling can affect the legitimacy of the labor organization. Under Section 15, the only
bargaining unit of the rank-and-file employees shall not include supervisory employees and/or security guards. (Emphasis supplied)
instance when a labor organization loses its legitimacy is when it violates its duty to bargain collectively; but there is no word on whether such mingling
would also result in loss of legitimacy. Thus, when the issue of whether the membership of two supervisory employees impairs the legitimacy of a rank-
and-file labor organization came before the Court En Banc in Lopez v. Chronicle Publication Employees Association, 41 the majority pronounced: By that provision, any questioned mingling will prevent an otherwise legitimate and duly registered labor organization from exercising its right to file a
petition for certification election.
It may be observed that nothing is said of the effect of such ineligibility upon the union itself or on the status of the other qualified members thereof should
such prohibition be disregarded. Considering that the law is specific where it intends to divest a legitimate labor union of any of the rights and privileges Thus, when the issue of the effect of mingling was brought to the fore in Toyota,48 the Court, citing Article 245 of the Labor Code, as amended by R.A.
granted to it by law, the absence of any provision on the effect of the disqualification of one of its organizers upon the legality of the union, may be No. 6715, held:
construed to confine the effect of such ineligibility only upon the membership of the supervisor. In other words, the invalidity of membership of one of the
organizers does not make the union illegal, where the requirements of the law for the organization thereof are, nevertheless, satisfied and
met.42 (Emphasis supplied) Clearly, based on this provision, a labor organization composed of both rank-and-file and supervisory employees is no labor organization at all. It cannot,
for any guise or purpose, be a legitimate labor organization. Not being one,an organization which carries a mixture of rank-and-file and supervisory
employees cannot possess any of the rights of a legitimate labor organization, including the right to file a petition for certification election for
Then the Labor Code was enacted in 1974 without reproducing Sec. 3 of R.A. No. 875. The provision in the Labor Code closest to Sec. 3 is Article the purpose of collective bargaining. It becomes necessary, therefore, anterior to the granting of an order allowing a certification election, to inquire
290,43 which is deafeningly silent on the prohibition against supervisory employees mingling with rank-and-file employees in one labor organization. Even into the composition of any labor organization whenever the status of the labor organization is challenged on the basis of Article 245 of the Labor Code.
the Omnibus Rules Implementing Book V of the Labor Code44 (Omnibus Rules) merely provides in Section 11, Rule II, thus:
xxxx
Sec. 11. Supervisory unions and unions of security guards to cease operation. – All existing supervisory unions and unions of security guards shall, upon
the effectivity of the Code, cease to operate as such and their registration certificates shall be deemed automatically cancelled. However, existing
collective agreements with such unions, the life of which extends beyond the date of effectivity of the Code shall be respected until their expiry date In the case at bar, as respondent union's membership list contains the names of at least twenty-seven (27) supervisory employees in Level Five
insofar as the economic benefits granted therein are concerned. positions, the union could not, prior to purging itself of its supervisory employee members, attain the status of a legitimate labor organization. Not being
one, it cannot possess the requisite personality to file a petition for certification election. 49 (Emphasis supplied)

Members of supervisory unions who do not fall within the definition of managerial employees shall become eligible to join or assist the rank and file
organization. The determination of who are managerial employees and who are not shall be the subject of negotiation between representatives of In Dunlop,50 in which the labor organization that filed a petition for certification election was one for supervisory employees, but in which the membership
included rank-and-file employees, the Court reiterated that such labor organization had no legal right to file a certification election to represent a
supervisory union and the employer. If no agreement s reached between the parties, either or both of them ma bring the issue to the nearest Regional
Office for determination. (Emphasis supplied) bargaining unit composed of supervisors for as long as it counted rank-and-file employees among its members.51

It should be emphasized that the petitions for certification election involved in Toyota and Dunlop were filed on November 26, 1992 and September 15,
The obvious repeal of the last clause of Sec. 3, R.A. No. 875 prompted the Court to declare in Bulletin v. Sanchez45 that supervisory employees who do
not fall under the category of managerial employees may join or assist in the formation of a labor organization for rank-and-file employees, but they may 1995, respectively; hence, the 1989 Rules was applied in both cases.
not form their own labor organization.
But then, on June 21, 1997, the 1989 Amended Omnibus Rules was further amended by Department Order No. 9, series of 1997 (1997 Amended
While amending certain provisions of Book V of the Labor Code, E.O. No. 111 and its implementing rules 46continued to recognize the right of supervisory Omnibus Rules). Specifically, the requirement under Sec. 2(c) of the 1989 Amended Omnibus Rules - that the petition for certification election indicate
employees, who do not fall under the category of managerial employees, to join a rank-and-file labor organization.47 that the bargaining unit of rank-and-file employees has not been mingled with supervisory employees - was removed. Instead, what the 1997 Amended
Omnibus Rules requires is a plain description of the bargaining unit, thus:

Effective 1989, R.A. No. 6715 restored the prohibition against the questioned mingling in one labor organization, viz:
Rule XI
Certification Elections
Sec. 18. Article 245 of the same Code, as amended, is hereby further amended to read as follows xxxx
Sec. 4. Forms and contents of petition. - The petition shall be in writing and under oath and shall contain, among others, the following: x x x (c) The
description of the bargaining unit.52
"Art. 245. Ineligibility of managerial employees to join any labor organization; right of supervisory employees. Managerial employees are not
eligible to join, assist or form any labor organization. Supervisory employees shall not be eligible for membership in a labor organization of the rank-and-
file employees but may join, assist or form separate labor organizations of their own." (Emphasis supplied) In Pagpalain Haulers, Inc. v. Trajano,53 the Court had occasion to uphold the validity of the 1997 Amended Omnibus Rules, although the specific
provision involved therein was only Sec. 1, Rule VI, to wit:
Unfortunately, just like R.A. No. 875, R.A. No. 6715 omitted specifying the exact effect any violation of the prohibition would bring about on the legitimacy
of a labor organization. Sec. 1. Chartering and creation of a local/chapter.- A duly registered federation or national union may directly create a local/chapter by submitting to the
Regional Office or to the Bureau two (2) copies of the following: a) a charter certificate issued by the federation or national union indicating the creation or
establishment of the local/chapter; (b) the names of the local/chapter’s officers, their addresses, and the principal office of the local/chapter; and (c) the
It was the Rules and Regulations Implementing R.A. No. 6715 (1989 Amended Omnibus Rules) which supplied the deficiency by introducing the following
local/ chapter’s constitution and by-laws; provided that where the local/chapter’s constitution and by-laws is the same as that of the federation or national
amendment to Rule II (Registration of Unions):
union, this fact shall be indicated accordingly.

Sec. 1. Who may join unions. – x x x Supervisory employees and security guards shall not be eligible for membership in a labor organization of the rank-
All the foregoing supporting requirements shall be certified under oath by the Secretary or the Treasurer of the local/chapter and attested to by its
and-file employees but may join, assist or form separate labor organizations of their own; Provided, that those supervisory employees who are included in
President.
an existing rank-and-file bargaining unit, upon the effectivity of Republic Act No. 6715, shall remain in that unit x x x. (Emphasis supplied)

which does not require that, for its creation and registration, a local or chapter submit a list of its members.
and Rule V (Representation Cases and Internal-Union Conflicts) of the Omnibus Rules, viz:

Then came Tagaytay Highlands Int’l. Golf Club, Inc. v. Tagaytay Highlands Employees Union-PGTWO54 in which the core issue was whether mingling
Sec. 1. Where to file. – A petition for certification election may be filed with the Regional Office which has jurisdiction over the principal office of the
affects the legitimacy of a labor organization and its right to file a petition for certification election. This time, given the altered legal milieu, the Court
employer. The petition shall be in writing and under oath.
abandoned the view in Toyota and Dunlop and reverted to its pronouncement in Lopez that while there is a prohibition against the mingling of supervisory
and rank-and-file employees in one labor organization, the Labor Code does not provide for the effects thereof. 55 Thus, the Court held that after a labor
Sec. 2. Who may file. – Any legitimate labor organization or the employer, when requested to bargain collectively, may file the petition. organization has been registered, it may exercise all the rights and privileges of a legitimate labor organization. Any mingling between supervisory and
Labor Relations Set IV * Takata v BLR to Silang v Coa * Page 20 of 29

rank-and-file employees in its membership cannot affect its legitimacy for that is not among the grounds for cancellation of its registration, unless such In its Comment8 and Position Paper,9 petitioner HCCS consistently noted that it is a parochial school with a total of 156 employees as of June 28, 2002,
mingling was brought about by misrepresentation, false statement or fraud under Article 239 of the Labor Code.56 broken down as follows: ninety-eight (98) teaching personnel, twenty-five (25) non-teaching academic employees, and thirty-three (33) non-teaching non-
academic workers. It averred that of the employees who signed to support the petition, fourteen (14) already resigned and six (6) signed twice. Petitioner
raised that members of private respondent do not belong to the same class; it is not only a mixture of managerial, supervisory, and rank-and-file
In San Miguel Corp. (Mandaue Packaging Products Plants) v. Mandaue Packing Products Plants-San Miguel Packaging Products-San Miguel Corp.
employees – as three (3) are vice-principals, one (1) is a department head/supervisor, and eleven (11) are coordinators – but also a combination of
Monthlies Rank-and-File Union-FFW,57 the Court explained that since the 1997 Amended Omnibus Rules does not require a local or chapter to provide a
teaching and non-teaching personnel – as twenty-seven (27) are non-teaching personnel. It insisted that, for not being in accord with Article 245 10 of the
list of its members, it would be improper for the DOLE to deny recognition to said local or chapter on account of any question pertaining to its individual
Labor Code, private respondent is an illegitimate labor organization lacking in personality to file a petition for certification election, as held in Toyota Motor
members.58
Philippines Corporation v. Toyota Motor Philippines Corporation Labor Union;11 and an inappropriate bargaining unit for want of community or mutuality of
interest, as ruled in Dunlop Slazenger (Phils.), Inc. v. Secretary of Labor and Employment12 and De La Salle University Medical Center and College of
More to the point is Air Philippines Corporation v. Bureau of Labor Relations, 59 which involved a petition for cancellation of union registration filed by the Medicine v. Laguesma.13
employer in 1999 against a rank-and-file labor organization on the ground of mixed membership:60 the Court therein reiterated its ruling in Tagaytay
Highlands that the inclusion in a union of disqualified employees is not among the grounds for cancellation, unless such inclusion is due to
Private respondent, however, countered that petitioner failed to substantiate its claim that some of the employees included in the petition for certification
misrepresentation, false statement or fraud under the circumstances enumerated in Sections (a) and (c) of Article 239 of the Labor Code.61lavvphil
election holds managerial and supervisory positions.14 Assuming it to be true, it argued that Section 11 (II), 15 Rule XI of DOLE Department Order (D.O.)
No. 9, Series of 1997, provided for specific instances in which a petition filed by a legitimate organization shall be dismissed by the Med-Arbiter and that
All said, while the latest issuance is R.A. No. 9481, the 1997 Amended Omnibus Rules, as interpreted by the Court in Tagaytay Highlands, San Miguel "mixture of employees" is not one of those enumerated. Private respondent pointed out that questions pertaining to qualifications of employees may be
and Air Philippines, had already set the tone for it. Toyota and Dunlop no longer hold sway in the present altered state of the law and the rules. threshed out in the inclusion-exclusion proceedings prior to the conduct of the certification election, pursuant to Section 2, 16 Rule XII of D.O. No. 9. Lastly,
similar to the ruling in In Re: Globe Machine and Stamping Company, 17 it contended that the will of petitioner’s employees should be respected as they
had manifested their desire to be represented by only one bargaining unit. To back up the formation of a single employer unit, private respondent
Consequently, the Court reverses the ruling of the CA and reinstates that of the DOLE granting the petition for certification election of KFWU. asserted that even if the teachers may receive additional pay for an advisory class and for holding additional loads, petitioner’s academic and non-
academic personnel have similar working conditions. It cited Laguna College v. Court of Industrial Relations, 18 as well as the case of a union in West
Now to the second issue of whether an employer like respondent may collaterally attack the legitimacy of a labor organization by filing a motion to dismiss Negros College in Bacolod City, which allegedly represented both academic and non-academic employees.
the latter’s petition for certification election.
On August 10, 2002, Med-Arbiter Agatha Ann L. Daquigan denied the petition for certification election on the ground that the unit which private
Except when it is requested to bargain collectively,62 an employer is a mere bystander to any petition for certification election; such proceeding is non- respondent sought to represent is inappropriate. She resolved:
adversarial and merely investigative, for the purpose thereof is to determine which organization will represent the employees in their collective bargaining
with the employer.63 The choice of their representative is the exclusive concern of the employees; the employer cannot have any partisan interest therein;
A certification election proceeding directly involves two (2) issues namely: (a) the proper composition and constituency of the bargaining unit; and (b) the
it cannot interfere with, much less oppose, the process by filing a motion to dismiss or an appeal from it; 64 not even a mere allegation that some validity of majority representation claims. It is therefore incumbent upon the Med-Arbiter to rule on the appropriateness of the bargaining unit once its
employees participating in a petition for certification election are actually managerial employees will lend an employer legal personality to block the composition and constituency is questioned.
certification election.65 The employer's onlyright in the proceeding is to be notified or informed thereof. 66

Section 1 (q), Rule I, Book V of the Omnibus Rules defines a "bargaining unit" as a group of employees sharing mutual interests within a given employer
The amendments to the Labor Code and its implementing rules have buttressed that policy even more. unit comprised of all or less than all of the entire body of employees in the employer unit or any specific occupational or geographical grouping within
such employer unit. This definition has provided the "community or mutuality of interest" test as the standard in determining the constituency of a
WHEREFORE, the petition is GRANTED. The December 13, 2002 Decision and October 7, 2003 Resolution of the Court of Appeals and the May 17, collective bargaining unit. This is so because the basic test of an asserted bargaining unit’s acceptability is whether or not it is fundamentally the
2000 Order of Med-Arbiter Anastacio L. Bactin are REVERSED and SETASIDE, while the August 18, 2000 Decision and September 28, 2000 Resolution combination which will best assure to all employees the exercise of their collective bargaining rights. The application of this test may either result in the
of the Department of Labor and Employment are REINSTATED.No costs.SO ORDERED. formation of an employer unit or in the fragmentation of an employer unit.
Republic of the Philippines
SUPREME COURT
In the case at bar, the employees of petitioner, may, as already suggested, quite easily be categorized into (2) general classes: one, the teaching staff;
Manila and two, the non-teaching-staff. Not much reflection is needed to perceive that the community or mutuality of interest is wanting between the teaching
EN BANC and the non-teaching staff. It would seem obvious that the teaching staff would find very little in common with the non-teaching staff as regards
G.R. No. 179146 July 23, 2013
responsibilities and function, working conditions, compensation rates, social life and interests, skills and intellectual pursuits, etc. These are plain and
HOLY CHILD CATHOLIC SCHOOL, Petitioner,
patent realities which cannot be ignored. These dictate the separation of these two categories of employees for purposes of collective bargaining.
vs. (University of the Philippines vs. Ferrer-Calleja, 211 SCRA 451)19
HON. PATRICIA STO. TOMAS, in her official capacity as Secretary of the Department of Labor and Employment, and PINAG-ISANG TINIG AT
LAKAS NG ANAKPAWIS – HOLY CHILD CATHOLIC SCHOOL TEACHERS AND EMPLOYEES LABOR UNION (HCCS-TELU-
PIGLAS), Respondents. Private respondent appealed before the SOLE, who, on December 27, 2002, ruled against the dismissal of the petition and directed the conduct of two
DECISION separate certification elections for the teaching and the non-teaching personnel, thus:
PERALTA, J.:
We agree with the Med-Arbiter that there are differences in the nature of work, hours and conditions of work and salary determination between the
Assailed in this petition for review on certiorari under Rule 45 of the Rules of Civil Procedure are the April 18, 2007 Decision1 and July 31, 2007
teaching and non-teaching personnel of petitioner. These differences were pointed out by petitioner in its position paper. We do not, however, agree with
Resolution2 of the Court of Appeals in CA-G.R. SP No. 76175, which affirmed the December 27, 2002 Decision3 and February 13, 2003 Resolution4 of the
the Med-Arbiter that these differences are substantial enough to warrant the dismissal of the petition. First, as pointed out by private respondent,
Secretary of the Department of Labor and Employment (SOLE) that set aside the August 10, 2002 Decision5 of the Med-Arbiter denying private
"inappropriateness of the bargaining unit sought to be represented is not a ground for the dismissal of the petition." In fact, in the cited case of University
respondent’s petition for certification election.
of the Philippines v. Ferrer-Calleja, supra, the Supreme Court did not order the dismissal of the petition but ordered the conduct of a certification election,
limiting the same among the non-academic personnel of the University of the Philippines.
The factual antecedents are as follows:
It will be recalled that in the U.P. case, there were two contending unions, the Organization of Non-Academic Personnel of U.P. (ONAPUP) and All U.P.
On May 31, 2002, a petition for certification election was filed by private respondent Pinag-Isang Tinig at Lakas ng Anakpawis – Holy Child Catholic Workers Union composed of both academic and nonacademic personnel of U.P. ONAPUP sought the conduct of certification election among the rank-
School Teachers and Employees Labor Union (HCCS-TELUPIGLAS), alleging that: PIGLAS is a legitimate labor organization duly registered with the and-file non-academic personnel only while the all U.P. Workers Union sought the conduct of certification election among all of U.P.’s rank-and-file
Department of Labor and Employment (DOLE) representing HCCS-TELU-PIGLAS; HCCS is a private educational institution duly registered and employees covering academic and nonacademic personnel. While the Supreme Court ordered a separate bargaining unit for the U.P. academic
operating under Philippine laws; there are approximately one hundred twenty (120) teachers and employees comprising the proposed appropriate personnel, the Court, however, did not order them to organize a separate labor organization among themselves. The All U.P. Workers Union was not
bargaining unit; and HCCS is unorganized, there is no collective bargaining agreement or a duly certified bargaining agent or a labor organization certified directed to divest itself of its academic personnel members and in fact, we take administrative notice that the All U.P. Workers Union continue to exist with
as the sole and exclusive bargaining agent of the proposed bargaining unit within one year prior to the filing of the petition.6 Among the documents a combined membership of U.P. academic and non-academic personnel although separate bargaining agreements is sought for the two bargaining units.
attached to the petition were the certificate of affiliation with Pinag-Isang Tinig at Lakas ng Anakpawis Kristiyanong Alyansa ng Makabayang Obrero Corollary, private respondent can continue to exist as a legitimate labor organization with the combined teaching and non-teaching personnel in its
(PIGLAS-KAMAO) issued by the Bureau of Labor Relations (BLR), charter certificate issued by PIGLASKAMAO, and certificate of registration of HCCS- membership and representing both classes of employees in separate bargaining negotiations and agreements.
TELU as a legitimate labor organization issued by the DOLE.7
Labor Relations Set IV * Takata v BLR to Silang v Coa * Page 21 of 29

WHEREFORE, the Decision of the Med-Arbiter dated 10 August 2002 is hereby REVERSED and SET ASIDE. In lieu thereof, a new order is hereby On the other hand, aside from reiterating its previous submissions, private respondent cites Sections 9 and 12 38 of Republic Act (R.A.) No. 9481 to
issued directing the conduct of two certification elections, one among the non-teaching personnel of Holy Child Catholic School, and the other, among the buttress its contention that petitioner has no standing to oppose the petition for certification election. On the basis of the statutory provisions, it reasons
teaching personnel of the same school, subject to the usual pre-election conferences and inclusion-exclusion proceedings, with the following choices: that an employer is not a party-in-interest in a certification election; thus, petitioner does not have the requisite right to protect even by way of restraining
order or injunction.
A. Certification Election Among Petitioner’s Teaching Personnel:
1. Holy Child Catholic School Teachers and Employees Labor Union; and First off, We cannot agree with private respondent’s invocation of R.A. No. 9481. Said law took effect only on June 14, 2007; hence, its applicability is
2. No Union. limited to labor representation cases filed on or after said date. 39Instead, the law and rules in force at the time private respondent filed its petition for
B. Certification Election Among Petitioner’s Non-Teaching Personnel: certification election on May 31, 2002 are R.A. No. 6715, which amended Book V of Presidential Decree (P.D.) No. 442 (the Labor Code), as amended,
1. Holy Child Catholic School Teachers and Employees Labor Union; and and the Rules and Regulations Implementing R.A. No. 6715, as amended by D.O. No. 9, which was dated May 1, 1997 but took effect on June 21,
2. No Union. 1997.40

Petitioner is hereby directed to submit to the Regional Office of origin within ten (10) days from receipt of this Decision, a certified separate list of its
However, note must be taken that even without the express provision of Section 12 of RA No. 9481, the "Bystander Rule" is already well entrenched in
teaching and non-teaching personnel or when necessary a separate copy of their payroll for the last three (3) months prior to the issuance of this
this jurisdiction. It has been consistently held in a number of cases that a certification election is the sole concern of the workers, except when the
Decision.20
employer itself has to file the petition pursuant to Article 259 of the Labor Code, as amended, but even after such filing its role in the certification process
ceases and becomes merely a bystander.41 The employer clearly lacks the personality to dispute the election and has no right to interfere at all
Petitioner filed a motion for reconsideration21 which, per Resolution dated February 13, 2003, was denied. Consequently, petitioner filed before the CA a therein.42 This is so since any uncalled-for concern on the part of the employer may give rise to the suspicion that it is batting for a company
Petition for Certiorari with Prayer for Temporary Restraining Order and Preliminary Injunction. 22 The CA resolved to defer action on the prayer for TRO union.43 Indeed, the demand of the law and policy for an employer to take a strict, hands-off stance in certification elections is based on the rationale that
pending the filing of private respondent’s Comment.23 Later, private respondent and petitioner filed their Comment 24 and Reply,25respectively. the employees’ bargaining representative should be chosen free from any extraneous influence of the management; that, to be effective, the bargaining
representative must owe its loyalty to the employees alone and to no other.44
On July 23, 2003, petitioner filed a motion for immediate issuance of a TRO, alleging that Hon. Helen F. Dacanay of the Industrial Relations Division of
the DOLE was set to implement the SOLE Decision when it received a summons and was directed to submit a certified list of teaching and non-teaching Now, going back to petitioner’s contention, the issue of whether a petition for certification election is dismissible on the ground that the labor
personnel for the last three months prior to the issuance of the assailed Decision. 26 Acting thereon, on August 5, 2003, the CA issued the TRO and organization’s membership allegedly consists of supervisory and rank-and-file employees is actually not a novel one. In the 2008 case of Republic v.
ordered private respondent to show cause why the writ of preliminary injunction should not be granted. 27Subsequently, a Manifestation and Motion28 was Kawashima Textile Mfg., Philippines, Inc.,45 wherein the employer-company moved to dismiss the petition for certification election on the ground inter alia
filed by private respondent, stating that it repleads by reference the arguments raised in its Comment and that it prays for the immediate lifting of the TRO that the union membership is a mixture of rank-and-file and supervisory employees, this Court had conscientiously discussed the applicability of Toyota
and the denial of the preliminary injunction. The CA, however, denied the manifestation and motion on November 21, 2003 29 and, upon motion of and Dunlop in the context of R.A. No. 6715 and D.O. No. 9, viz.:
petitioner,30 granted the preliminary injunction on April 21, 2005.31 Thereafter, both parties filed their respective Memorandum.32
It was in R.A. No. 875, under Section 3, that such questioned mingling was first prohibited, to wit:
On April 18, 2007, the CA eventually dismissed the petition. As to the purported commingling of managerial, supervisory, and rank-and-file employees in
private respondent’s membership, it held that the Toyota ruling is inapplicable because the vice-principals, department head, and coordinators are neither
Sec. 3. Employees' right to self-organization. - Employees shall have the right to self-organization and to form, join or assist labor organizations of their
supervisory nor managerial employees. It reasoned:
own choosing for the purpose of collective bargaining through representatives of their own choosing and to engage in concerted activities for the purpose
of collective bargaining and other mutual aid or protection. Individuals employed as supervisors shall not be eligible for membership in a labor
x x x While it may be true that they wield power over other subordinate employees of the petitioner, it must be stressed, however, that their functions are organization of employees under their supervision but may form separate organizations of their own. (Emphasis supplied)
not confined with policy-determining such as hiring, firing, and disciplining of employees, salaries, teaching/working hours, other monetary and non-
monetary benefits, and other terms and conditions of employment. Further, while they may formulate policies or guidelines, nonetheless, such is merely
Nothing in R.A. No. 875, however, tells of how the questioned mingling can affect the legitimacy of the labor organization. Under Section 15, the only
recommendatory in nature, and still subject to review and evaluation by the higher executives, i.e., the principals or executive officers of the petitioner. It
instance when a labor organization loses its legitimacy is when it violates its duty to bargain collectively; but there is no word on whether such mingling
cannot also be denied that in institutions like the petitioner, company policies have already been pre-formulated by the higher executives and all that the
would also result in loss of legitimacy. Thus, when the issue of whether the membership of two supervisory employees impairs the legitimacy of a rank-
mentioned employees have to do is carry out these company policies and standards. Such being the case, it is crystal clear that there is no improper
and-file labor organization came before the Court En Banc in Lopez v. Chronicle Publication Employees Association, the majority pronounced:
commingling of members in the private respondent union as to preclude its petition for certification of (sic) election. 33

It may be observed that nothing is said of the effect of such ineligibility upon the union itself or on the status of the other qualified members thereof should
Anent the alleged mixture of teaching and non-teaching personnel, the CA agreed with petitioner that the nature of the former’s work does not coincide
such prohibition be disregarded. Considering that the law is specific where it intends to divest a legitimate labor union of any of the rights and privileges
with that of the latter. Nevertheless, it ruled that the SOLE did not commit grave abuse of discretion in not dismissing the petition for certification election,
granted to it by law, the absence of any provision on the effect of the disqualification of one of its organizers upon the legality of the union, may be
since it directed the conduct of two separate certification elections based on Our ruling in University of the Philippines v. Ferrer-Calleja.34
construed to confine the effect of such ineligibility only upon the membership of the supervisor. In other words, the invalidity of membership of one of the
organizers does not make the union illegal, where the requirements of the law for the organization thereof are, nevertheless, satisfied and met. (Emphasis
A motion for reconsideration35 was filed by petitioner, but the CA denied the same;36 hence, this petition assigning the alleged errors as follows: supplied)

I. Then the Labor Code was enacted in 1974 without reproducing Sec. 3 of R.A. No. 875. The provision in the Labor Code closest to Sec. 3 is Article 290,
THE HONORABLE COURT OF APPEALS ERRED IN HOLDING THAT THE RULING IN THE CASE OF TOYOTA MOTOR PHILIPPINES which is deafeningly silent on the prohibition against supervisory employees mingling with rank-and-file employees in one labor organization. Even the
CORPORATION VS. TOYOTA MOTOR PHILIPPINES CORPORATION LABOR UNION (268 SCRA 573) DOES NOT APPLY IN THE CASE AT BAR Omnibus Rules Implementing Book V of the Labor Code (Omnibus Rules) merely provides in Section 11, Rule II, thus:
DESPITE THE [COMMINGLING] OF BOTH SUPERVISORY OR MANAGERIAL AND RANK-AND-FILE EMPLOYEES IN THE RESPONDENT UNION;
II
Sec. 11. Supervisory unions and unions of security guards to cease operation. - All existing supervisory unions and unions of security guards shall, upon
THE HONORABLE COURT OF APPEALS ERRED IN ITS CONFLICTING RULING ALLOWING THE CONDUCT OF CERTIFICATION ELECTION BY
the effectivity of the Code, cease to operate as such and their registration certificates shall be deemed automatically cancelled. However, existing
UPHOLDING THAT THE RESPONDENT UNION REPRESENTED A BARGAINING UNIT DESPITE ITS OWN FINDINGS THAT THERE IS NO
collective agreements with such unions, the life of which extends beyond the date of effectivity of the Code shall be respected until their expiry date
MUTUALITY OF INTEREST BETWEEN THE MEMBERS OF RESPONDENT UNION APPLYING THE TEST LAID DOWN IN THE CASE OF
insofar as the economic benefits granted therein are concerned.
UNIVERSITY OF THE PHILIPPINES VS. FERRER-CALLEJA (211 SCRA 451).37

We deny. Members of supervisory unions who do not fall within the definition of managerial employees shall become eligible to join or assist the rank and file
organization. The determination of who are managerial employees and who are not shall be the subject of negotiation between representatives of
Petitioner claims that the CA contradicted the very definition of managerial and supervisory employees under existing law and jurisprudence when it did supervisory union and the employer. If no agreement s reached between the parties, either or both of them may bring the issue to the nearest Regional
not classify the vice-principals, department head, and coordinators as managerial or supervisory employees merely because the policies and guidelines Office for determination. (Emphasis supplied)
they formulate are still subject to the review and evaluation of the principal or executive officers of petitioner. It points out that the duties of the vice-
principals, department head, and coordinators include the evaluation and assessment of the effectiveness and capability of the teachers under them; that
such evaluation and assessment is independently made without the participation of the higher Administration of petitioner; that the fact that their The obvious repeal of the last clause of Sec. 3, R.A. No. 875 prompted the Court to declare in Bulletin v. Sanchez that supervisory employees who do not
recommendation undergoes the approval of the higher Administration does not take away the independent nature of their judgment; and that it would be fall under the category of managerial employees may join or assist in the formation of a labor organization for rank-and-file employees, but they may not
form their own labor organization.
difficult for the vice-principals, department head, and coordinators to objectively assess and evaluate the performances of teachers under them if they
would be allowed to be members of the same labor union.
Labor Relations Set IV * Takata v BLR to Silang v Coa * Page 22 of 29

While amending certain provisions of Book V of the Labor Code, E.O. No. 111 and its implementing rules continued to recognize the right of supervisory that the bargaining unit of rank-and-file employees has not been mingled with supervisory employees - was removed. Instead, what the 1997 Amended
employees, who do not fall under the category of managerial employees, to join a rank- and-file labor organization. Omnibus Rules requires is a plain description of the bargaining unit, thus:

Effective 1989, R.A. No. 6715 restored the prohibition against the questioned mingling in one labor organization, viz.: Rule XI
Certification Elections
xxxx
Sec. 18. Article 245 of the same Code, as amended, is hereby further amended to read as follows:
Sec. 4. Forms and contents of petition. - The petition shall be in writing and under oath and shall contain, among others, the following: x x x (c) The
description of the bargaining unit."
Art. 245. Ineligibility of managerial employees to join any labor organization; right of supervisory employees. Managerial employees are not eligible to join,
assist or form any labor organization. Supervisory employees shall not be eligible for membership in a labor organization of the rank-and-file employees
In Pagpalain Haulers, Inc. v. Trajano, the Court had occasion to uphold the validity of the 1997 Amended Omnibus Rules, although the specific provision
but may join, assist or form separate labor organizations of their own (Emphasis supplied)
involved therein was only Sec. 1, Rule VI, to wit:

Unfortunately, just like R.A. No. 875, R.A. No. 6715 omitted specifying the exact effect any violation of the prohibition would bring about on the legitimacy
Sec. 1. Chartering and creation of a local/chapter.- A duly registered federation or national union may directly create a local/chapter by submitting to the
of a labor organization.
Regional Office or to the Bureau two (2) copies of the following: a) a charter certificate issued by the federation or national union indicating the creation or
establishment of the local/chapter; (b) the names of the local/chapter's officers, their addresses, and the principal office of the local/chapter; and (c) the
It was the Rules and Regulations Implementing R.A. No. 6715 (1989 Amended Omnibus Rules) which supplied the deficiency by introducing the following local/ chapter's constitution and by-laws; provided that where the local/chapter's constitution and by-laws is the same as that of the federation or national
amendment to Rule II (Registration of Unions): union, this fact shall be indicated accordingly.

Sec. 1. Who may join unions. - x x x Supervisory employees and security guards shall not be eligible for membership in a labor organization of the rank- All the foregoing supporting requirements shall be certified under oath by the Secretary or the Treasurer of the local/chapter and attested to by its
and-file employees but may join, assist or form separate labor organizations of their own; Provided, that those supervisory employees who are included in President.
an existing rank-and-file bargaining unit, upon the effectivity of Republic Act No. 6715, shall remain in that unit x x x. (Emphasis supplied)
which does not require that, for its creation and registration, a local or chapter submit a list of its members.
and Rule V (Representation Cases and Internal-Union Conflicts) of the Omnibus Rules, viz.;
Then came Tagaytay Highlands Int'l. Golf Club, Inc. v. Tagaytay Highlands Employees Union-PTGWO in which the core issue was whether mingling
Sec. 1. Where to file. - A petition for certification election may be filed with the Regional Office which has jurisdiction over the principal office of the affects the legitimacy of a labor organization and its right to file a petition for certification election. This time, given the altered legal milieu, the Court
employer. The petition shall be in writing and under oath. abandoned the view in Toyota and Dunlop and reverted to its pronouncement in Lopez that while there is a prohibition against the mingling of supervisory
and rank-and-file employees in one labor organization, the Labor Code does not provide for the effects thereof. Thus, the Court held that after a labor
organization has been registered, it may exercise all the rights and privileges of a legitimate labor organization. Any mingling between supervisory and
Sec. 2. Who may file. - Any legitimate labor organization or the employer, when requested to bargain collectively, may file the petition.
rank-and-file employees in its membership cannot affect its legitimacy for that is not among the grounds for cancellation of its registration, unless such
mingling was brought about by misrepresentation, false statement or fraud under Article 239 of the Labor Code.
The petition, when filed by a legitimate labor organization, shall contain, among others:
In San Miguel Corp. (Mandaue Packaging Products Plants) v. Mandaue Packing Products Plants-San Miguel Packaging Products-San Miguel Corp.
xxxx Monthlies Rank-and-File Union-FFW, the Court explained that since the 1997 Amended Omnibus Rules does not require a local or chapter to provide a
list of its members, it would be improper for the DOLE to deny recognition to said local or chapter on account of any question pertaining to its individual
members.
(c) description of the bargaining unit which shall be the employer unit unless circumstances otherwise require; and provided further, that the appropriate
bargaining unit of the rank-and-file employees shall not include supervisory employees and/or security guards. (Emphasis supplied)
More to the point is Air Philippines Corporation v. Bureau of Labor Relations, which involved a petition for cancellation of union registration filed by the
employer in 1999 against a rank-and-file labor organization on the ground of mixed membership: the Court therein reiterated its ruling in Tagaytay
By that provision, any questioned mingling will prevent an otherwise legitimate and duly registered labor organization from exercising its right to file a Highlands that the inclusion in a union of disqualified employees is not among the grounds for cancellation, unless such inclusion is due to
petition for certification election. misrepresentation, false statement or fraud under the circumstances enumerated in Sections (a) and (c) of Article 239 of the Labor Code.

Thus, when the issue of the effect of mingling was brought to the fore in Toyota, the Court, citing Article 245 of the Labor Code, as amended by R.A. No. All said, while the latest issuance is R.A. No. 9481, the 1997 Amended Omnibus Rules, as interpreted by the Court in Tagaytay Highlands, San Miguel
6715, held: and Air Philippines, had already set the tone for it. Toyota and Dunlop no longer hold sway in the present altered state of the law and the rules.46

Clearly, based on this provision, a labor organization composed of both rank-and-file and supervisory employees is no labor organization at all. It cannot, When a similar issue confronted this Court close to three years later, the above ruling was substantially quoted in Samahang Manggagawa sa Charter
for any guise or purpose, be a legitimate labor organization. Not being one, an organization which carries a mixture of rank-and-file and supervisory Chemical Solidarity of Unions in the Philippines for Empowerment and Reforms (SMCC-Super) v. Charter Chemical and Coating Corporation.47 In
employees cannot possess any of the rights of a legitimate labor organization, including the right to file a petition for certification election for the purpose unequivocal terms, We reiterated that the alleged inclusionof supervisory employees in a labor organization seeking to represent the bargaining unit of
of collective bargaining. It becomes necessary, therefore, anterior to the granting of an order allowing a certification election, to inquire into the rank-and-file employees does not divest it of its status as a legitimate labor organization. 48
composition of any labor organization whenever the status of the labor organization is challenged on the basis of Article 245 of the Labor Code.

xxxx Indeed, Toyota and Dunlop no longer hold true under the law and rules governing the instant case. The petitions for certification election involved in
In the case at bar, as respondent union's membership list contains the names of at least twenty-seven (27) supervisory employees in Level Five Toyota and Dunlop were filed on November 26, 1992 and September 15, 1995, respectively; hence, the 1989 Rules and Regulations Implementing R.A.
positions, the union could not, prior to purging itself of its supervisory employee members, attain the status of a legitimate labor organization. Not being No. 6715 (1989 Amended Omnibus Rules) was applied. In contrast, D.O. No. 9 is applicable in the petition for certification election of private respondent
one, it cannot possess the requisite personality to file a petition for certification election. (Emphasis supplied) as it was filed on May 31, 2002.

In Dunlop, in which the labor organization that filed a petition for certification election was one for supervisory employees, but in which the membership Following the doctrine laid down in Kawashima and SMCC-Super, it must be stressed that petitioner cannot collaterally attack the legitimacy of private
included rank-and-file employees, the Court reiterated that such labor organization had no legal right to file a certification election to represent a respondent by praying for the dismissal of the petition for certification election:
bargaining unit composed of supervisors for as long as it counted rank-and-file employees among its members.
Except when it is requested to bargain collectively, an employer is a mere bystander to any petition for certification election; such proceeding is non-
It should be emphasized that the petitions for certification election involved in Toyota and Dunlop were filed on November 26, 1992 and September 15, adversarial and merely investigative, for the purpose thereof is to determine which organization will represent the employees in their collective bargaining
1995, respectively; hence, the 1989 Rules was applied in both cases. with the employer. The choice of their representative is the exclusive concern of the employees; the employer cannot have any partisan interest therein; it
cannot interfere with, much less oppose, the process by filing a motion to dismiss or an appeal from it; not even a mere allegation that some employees
participating in a petition for certification election are actually managerial employees will lend an employer legal personality to block the certification
But then, on June 21, 1997, the 1989 Amended Omnibus Rules was further amended by Department Order No. 9, series of 1997 (1997 Amended election. The employer's only right in the proceeding is to be notified or informed thereof.
Omnibus Rules). Specifically, the requirement under Sec. 2(c) of the 1989 Amended Omnibus Rules - that the petition for certification election indicate
Labor Relations Set IV * Takata v BLR to Silang v Coa * Page 23 of 29

The amendments to the Labor Code and its implementing rules have buttressed that policy even more. 49 In the same manner, the teaching and non-teaching personnel of petitioner school must form separate bargaining units.1âwphi1 Thus, the order for the
conduct of two separate certification elections, one involving teaching personnel and the other involving non-teaching personnel. It should be stressed
that in the subject petition, private respondent union sought the conduct of a certification election among all the rank-and-file personnel of petitioner
Further, the determination of whether union membership comprises managerial and/or supervisory employees is a factual issue that is best left for
school. Since the decision of the Supreme Court in the U.P. case prohibits us from commingling teaching and non-teaching personnel in one bargaining
resolution in the inclusion-exclusion proceedings, which has not yet happened in this case so still premature to pass upon. We could only emphasize the
unit, they have to be separated into two separate bargaining units with two separate certification elections to determine whether the employees in the
rule that factual findings of labor officials, who are deemed to have acquired expertise in matters within their jurisdiction, are generally accorded not only
respective bargaining units desired to be represented by private respondent. In the U.P. case, only one certification election among the non-academic
with respect but even finality by the courts when supported by substantial evidence. 50 Also, the jurisdiction of this Court in cases brought before it from the
personnel was ordered, because ONAPUP sought to represent that bargaining unit only. No petition for certification election among the academic
CA via Rule 45 is generally limited to reviewing errors of law or jurisdiction. The findings of fact of the CA are conclusive and binding. Except in certain
personnel was instituted by All U.P. Workers Union in the said case; thus, no certification election pertaining to its intended bargaining unit was ordered
recognized instances,51We do not entertain factual issues as it is not Our function to analyze or weigh evidence all over again; the evaluation of facts is
by the Court.58
best left to the lower courts and administrative agencies/quasi-judicial bodies which are better equipped for the task.52

Indeed, the purpose of a certification election is precisely to ascertain the majority of the employees’ choice of an appropriate bargaining unit – to be or
Turning now to the second and last issue, petitioner argues that, in view of the improper mixture of teaching and non-teaching personnel in private
not to be represented by a labor organization and, if in the affirmative case, by which one. 59
respondent due to the absence of mutuality of interest among its members, the petition for certification election should have been dismissed on the
ground that private respondent is not qualified to file such petition for its failure to qualify as a legitimate labor organization, the basic qualification of which
is the representation of an appropriate bargaining unit. At this point, it is not amiss to stress once more that, as a rule, only questions of law may be raised in a Rule 45 petition. In Montoya v. Transmed Manila
Corporation,60 the Court discussed the particular parameters of a Rule 45 appeal from the CA’s Rule 65 decision on a labor case, as follows:
We disagree.
x x x In a Rule 45 review, we consider the correctness of the assailed CA decision, in contrast with the review for jurisdictional error that we undertake
under Rule 65. Furthermore, Rule 45 limits us to the review of questions of law raised against the assailed CA decision. In ruling for legal correctness, we
The concepts of a union and of a legitimate labor organization are different from, but related to, the concept of a bargaining unit:
have to view the CA decision in the same context that the petition for certiorari it ruled upon was presented to it; we have to examine the CA decision
from the prism of whether it correctly determined the presence or absence of grave abuse of discretion in the NLRC decision before it, not on the basis of
Article 212(g) of the Labor Code defines a labor organization as "any union or association of employees which exists in whole or in part for the purpose of whether the NLRC decision on the merits of the case was correct. In other words, we have to be keenly aware that the CA undertook a Rule 65 review,
collective bargaining or of dealing with employers concerning terms and conditions of employment." Upon compliance with all the documentary not a review on appeal, of the NLRC decision challenged before it. This is the approach that should be basic in a Rule 45 review of a CA ruling in a labor
requirements, the Regional Office or Bureau shall issue in favor of the applicant labor organization a certificate indicating that it is included in the roster of case. In question form, the question to ask is: Did the CA correctly determine whether the NLRC committed grave abuse of discretion in ruling on the
legitimate labor organizations. Any applicant labor organization shall acquire legal personality and shall be entitled to the rights and privileges granted by case?61
law to legitimate labor organizations upon issuance of the certificate of registration. 53
Our review is, therefore, limited to the determination of whether the CA correctly resolved the presence or absence of grave abuse of discretion in the
In case of alleged inclusion of disqualified employees in a union, the proper procedure for an employer like petitioner is to directly file a petition for decision of the SOLE, not on the basis of whether the latter's decision on the merits of the case was strictly correct. Whether the CA committed grave
cancellation of the union’s certificate of registration due to misrepresentation, false statement or fraud under the circumstances enumerated in Article 239 abuse of discretion is not what is ruled upon but whether it correctly determined the existence or want of grave abuse of discretion on the part of the
of the Labor Code, as amended.54 To reiterate, private respondent, having been validly issued a certificate of registration, should be considered as having SOLE.
acquired juridical personality which may not be attacked collaterally.
WHEREFORE, the pet1t1on is DENIED. The April 18, 2007 Decision and July 31, 2007, Resolution of the Court of Appeals in CA-G.R. SP No. 76175,
On the other hand, a bargaining unit has been defined as a "group of employees of a given employer, comprised of all or less than all of the entire body which affirmed the December 27, 2002 Decision of the Secretary of the Department of Labor and Employment that set aside theAugust 10, 2002 Decision
of employees, which the collective interests of all the employees, consistent with equity to the employer, indicated to be best suited to serve reciprocal of the Med-Arbiter denying private respondent's petition for certification election are hereby AFFIRMED.SO ORDERED
rights and duties of the parties under the collective bargaining provisions of the law."55 In determining the proper collective bargaining unit and what unit
would be appropriate to be the collective bargaining agency, the Court, in the seminal case of Democratic Labor Association v. Cebu Stevedoring
Republic of the Philippines
Company, Inc.,56 mentioned several factors that should be considered, to wit: (1) will of employees (Globe Doctrine); (2) affinity and unity of employees'
SUPREME COURT
interest, such as substantial similarity of work and duties, or similarity of compensation and working conditions; (3) prior collective bargaining history; and
Manila
(4) employment status, such as temporary, seasonal and probationary employees. We stressed, however, that the test of the grouping is community or
FIRST DIVISION
mutuality of interest, because "the basic test of an asserted bargaining unit's acceptability is whether or not it is fundamentally the combination which will
G.R. No. 157086 February 18, 2013
best assure to all employees the exercise of their collective bargaining rights." 57
LEPANTO CONSOLIDATED MINING COMPANY, Petitioner,
vs.
As the SOLE correctly observed, petitioner failed to comprehend the full import of Our ruling in U.P. It suffices to quote with approval the apt disposition of THE LEPANTO CAPATAZ UNION, Respondent.
the SOLE when she denied petitioner’s motion for reconsideration: DECISION
BERSAMIN, J.:
Capatazes are not rank-and-file employees because they perform supervisory functions for the management; hence, they may form their own union that
Petitioner likewise claimed that we erred in interpreting the decision of the Supreme Court in U.P. v. Ferrer-Calleja, supra. According to petitioner, the
is separate and distinct from the labor organization of rank-and-file employees.
Supreme Court stated that the non-academic rank-andfile employees of the University of the Philippines shall constitute a bargaining unit to the exclusion
of the academic employees of the institution. Hence, petitioner argues, it sought the creation of separate bargaining units, namely: (1) petitioner’s
teaching personnel to the exclusion of non-teaching personnel; and (2) petitioner’s non-teaching personnel to the exclusion of teaching personnel. The Case-

Petitioner appears to have confused the concepts of membership in a bargaining unit and membership in a union. In emphasizing the phrase "to the Lepanto Consolidated Mining Company (Lepanto) assails the Resolution promulgated on December 18, 2002, 1whereby the Court of Appeals (CA)
exclusion of academic employees" stated in U.P. v. Ferrer-Calleja, petitioner believed that the petitioning union could not admit academic employees of dismissed its petition for certiorari on the ground of its failure to first file a motion for reconsideration against the decision rendered by the Secretary of the
the university to its membership. But such was not the intention of the Supreme Court. Department of Labor and Employment (DOLE); and the resolution promulgated on January 31, 2003,2 whereby the CA denied Lepanto's motion for
reconsideration.
A bargaining unit is a group of employees sought to be represented by a petitioning union. Such employees need not be members of a union seeking the
conduct of a certification election. A union certified as an exclusive bargaining agent represents not only its members but also other employees who are Antecedents
not union members. As pointed out in our assailed Decision, there were two contending unions in the U.P. case, namely, the Organization of Non-
Academic Personnel of U.P. (ONAPUP) and the All U.P. Worker’s Union composed of both U.P. academic and non-academic personnel. ONAPUP
As a domestic corporation authorized to engage in large-scale mining, Lepanto operated several mining claims in Mankayan, Benguet. On May 27, 1998,
sought the conduct of a certification election among the rank-and-file non-academic personnel only, while the All U.P. Workers Union intended to cover all
respondent Lepanto Capataz Union (Union), a labor organization duly registered with DOLE, filed a petition for consent election with the Industrial
U.P. rank-and-file employees, involving both academic and non-academic personnel.
Relations Division of the Cordillera Regional Office (CAR) of DOLE, thereby proposing to represent 139 capatazes of Lepanto.3

The Supreme Court ordered the "non-academic rank-and-file employees of U.P. to constitute a bargaining unit to the exclusion of the academic
In due course, Lepanto opposed the petition, 4 contending that the Union was in reality seeking a certification election, not a consent election, and would
employees of the institution", but did not order them to organize a separate labor organization. In the U.P. case, the Supreme Court did not dismiss the
be thereby competing with the Lepanto Employees Union (LEU), the current collective bargaining agent. Lepanto pointed out that the capatazes were
petition and affirmed the order for the conduct of a certification election among the non-academic personnel of U.P., without prejudice to the right of the
already members of LEU, the exclusive representative of all rank-and-file employees of its Mine Division.
academic personnel to constitute a separate bargaining unit for themselves and for the All U.P. Workers Union to institute a petition for certification
election.
Labor Relations Set IV * Takata v BLR to Silang v Coa * Page 24 of 29

On May 2, 2000, Med-Arbiter Michaela A. Lontoc of DOLE-CAR issued a ruling to the effect that the capatazescould form a separate bargaining unit due On the day of the certification election, however, Lepanto presented an opposition/protest. 11 Hence, on February 8, 2001, a hearing was held on
to their not being rank-and-file employees,5 viz: Lepanto’s opposition/protest. Although the parties were required in that hearing to submit their respective position papers, Lepanto later opted not to
submit its position paper,12 and contended that the issues identified during the hearing did not pose any legal issue to be addressed in a position paper. 13
xxxx
On April 26, 2001, Med-Arbiter Florence Marie A. Gacad-Ulep of DOLE-CAR rendered a decision certifying the Union as the sole and exclusive
bargaining agent of all capatazes of Lepanto.14
We agree with petitioner that its members perform a function totally different from the rank-and-file employees. The word capataz is defined in Webster’s
Third International Dictionary, 1986 as "a boss", "foreman" and "an overseer". The employer did not dispute during the hearing that the capatazes
indeed take charge of the implementation of the job orders by supervising and instructing the miners, mackers and other rank-and-file workers On May 18, 2001, Lepanto appealed the decision of Med-Arbiter Gacad-Ulep to the DOLE Secretary.
under them, assess and evaluate their performance, make regular reports and recommends (sic) new systems and procedure of work, as well
as guidelines for the discipline of employees. As testified to by petitioner’s president, the capatazes are neither rank-and-file nor supervisory
By her Resolution dated September 17, 2002,15 DOLE Secretary Patricia A. Sto. Tomas affirmed the decision dated April 26, 2001, holding and disposing
and, more or less, fall in the middle of their rank. In this respect, we can see that indeed the capatazes differ from the rank-and-file and can by
thus:
themselves constitute a separate bargaining unit.

Appellant accused Med-Arbiter Ulep of grave abuse of discretion amounting to lack of jurisdiction based on her failure to resolve appellant’s motion to
While it is claimed by the employer that historically, the capatazes have been considered among the rank-and-file and that it is only now that they seek a
modify order to submit position papers and on rendering judgment on the basis only of appellee’s position paper.
separate bargaining unit such history of affiliation with the rank-and-file association of LEU cannot totally prevent the capatazes from disaffiliating and
organizing themselves separately. The constitutional right of every worker to self-organization essentially gives him the freedom to join or not to join an
organization of his own choosing. We deny.

The fact that petitioner seeks to represent a separate bargaining unit from the rank-and-file employees represented by the LEU renders the contract bar Section 5, Rule XXV of Department Order No. 9, otherwise known as the New Rules Implementing Book V of the Labor Code, states that "in all
rule inapplicable. While the collective bargaining agreement existing between the LEU and the employer covering the latter’s rank-andfile employee proceedings at all levels, incidental motions shall not be given due course, but shall remain as part of the records for whatever they may be worth when
covers likewise the capatazes, it was testified to and undisputed by the employer that the capatazes did not anymore participate in the renegotiation and the case is decided on the merits".
ratification of the new CBA upon expiration of their old one on 16 November 1998. Their nonparticipation was apparently due to their formation of the new
bargaining unit. Thus, while the instant petition was filed on 27 May 1998, prior to the freedom period, in the interest of justice and in consonance with the
constitutional right of workers to self-organization, the petition can be deemed to have been filed at the time the 60-day freedom period set in. After all, Further, the motion to modify order to submit position papers filed by appellant is without merit. Appellant claimed that the issues over which Med-Arbiter
the petition was still pending and unresolved during this period. Ulep directed the submission of position papers were: (1) failure to challenge properly; (2) failure (especially of LEU) to participate actively in the
proceedings before the decision calling for the conduct of certification election; and (3) validity of earlier arguments. According to appellant, the first issue
was for appellee LCU to reply to in its position paper, the second issue was for the LEU and the third issue for appellant company to explain in their
WHEREFORE, the petition is hereby granted and a certification election among the capataz employees of the Lepanto Consolidated Mining Company is respective position paper. It was the position of appellant company that unless the parties filed their position paper on each of their respective issues, the
hereby ordered conducted, subject to the usual preelection and inclusion/exclusion proceedings, with the following choices: other parties cannot discuss the issues they did not raise in the same position papers and have to await receipt of the others’ position paper for their
appropriate reply.
1.Lepanto Capataz Union; and
2.No Union. Section 9, Rule XI of Department Order No. 9, which is applied with equal force in the disposition of protests on the conduct of election, states that "the
Med-Arbiter shall in the same hearing direct all concerned parties, including the employer, to simultaneously submit their respective position papers within
The employer is directed to submit to this office within ten (10) days from receipt hereof a copy of the certified list of its capataz employees and the payroll a non-extendible period of ten days". The issues as recorded in the minutes of 28 February 2001 hearing before the Med- Arbiter are clear. The parties,
covering the said bargaining unit for the last three (3) months prior to the issuance hereof.SO DECIDED. 6 including appellant company were required to submit their respective positions on whether there was proper challenge of the voters, whether LEU failed
to participate in the proceedings, if so, whether it should be allowed to participate at this belated stage and whether the arguments raised during the pre-
election conferences and in the protests are valid. The parties, including appellant company were apprised of these issues and they agreed thereto. The
Lepanto appealed to the DOLE Secretary.7
minutes of the hearing even contained the statement that "no order will issue" and that "the parties are informed accordingly". If there is any matter that
had to be clarified, appellant should have clarified the same during the said hearing and refused to file its position paper simultaneously with LCU and
On July 12, 2000, then DOLE Undersecretary Rosalinda Dimapilis- Baldoz (Baldoz), acting by authority of the DOLE Secretary, affirmed the ruling of LEU. It appears that appellant did not do so and acquiesced to the filing of its position paper within fifteen days from the date of said hearing.
Med-Arbiter Lontoc,8 pertinently stating as follows:
Neither is there merit in appellant’s contention that the Med- Arbiter resolved the protest based solely on appellee LCU’s position paper. Not only did the
xxxx Med-Arbiter discuss the demerits of appellant’s motion to modify order to submit position papers but likewise the demerits of its protest. We do not,
however, agree with the Med-Arbiter that the protest should be dismissed due to appellant’s failure to challenge the individual voters during the election.
We take note of the minutes of the pre-election conference on 10 November 2000, thus:
The bargaining unit sought to be represented by the appellee are the capataz employees of the appellant. There is no other labor organization of
capatazes within the employer unit except herein appellant. Thus, appellant is an unorganized establishment in so far as the bargaining unit of capatazes
is concerned. In accordance with the last paragraph of Section 11, Rule XI, Department Order No. 9 which provides that "in a petition filed by a legitimate "It was also agreed upon (by union and management’s legal officer) that all those listed will be allowed to vote during the certification election subject to
labor organization involving an unorganized establishment, the Med-Arbiter shall, pursuant to Article 257 of the Code, automatically order the conduct of challenge by management on ground that none of them belongs to the bargaining unit". (Underscoring supplied)
certification election after determining that the petition has complied with all requirements under Section 1, 2 and 4 of the same rules and that none of the
grounds for dismissal thereof exists", the order for the conduct of a certification election is proper.
It is therefore, not correct to say that there was no proper challenge made by appellant company. The challenge was already manifested during the pre-
election conference, specifying that all listed voters were being challenged because they do not belong to the bargaining unit of capatazes. Likewise, the
Finally, as to the issue of whether the Med-Arbiter exhibited ignorance of the law when she directed the conduct of a certification election when appellee formal protest filed by appellant company on the day of the election showed its protest to the conduct of the election on the grounds that (1) none of the
prays for the conduct of a consent election, let it be stressed that appellee seeks to be recognized as the sole and exclusive bargaining representative of names submitted and included (with pay bracket 8 and 9) to vote qualifies as capataz under the five-point characterization made in 02 May 2000 decision
all capataz employees of appellant. There are two modes by which this can be achieved, one is by voluntary recognition and two, by consent or calling for the conduct of certification election; (2) the characterization made in the 02 May 2000 decision pertains to shift bosses who constitutes another
certification election. Voluntary recognition under Rule X, Department Order No. 9 is a mode whereby the employer voluntarily recognizes the union as union, the Lepanto Local Staff Union; and (3) the names listed in the voters’ list are members of another union, the Lepanto Employees Union. This
the bargaining representative of all the members in the bargaining unit sought to be represented. Consent and certification election under Rules XI and constitutes proper challenge to the eligibility of all the voters named in the list which includes all those who cast their votes. The election officer should
XII of Department Order No. 9 is a mode whereby the members of the bargaining unit decide whether they want a bargaining representative and if so, have not canvassed the ballots and allowed the Med-Arbiter to first determine their eligibility.
who they want it to be. The difference between a consent election and a certification election is that the conduct of a consent election is agreed upon by
the parties to the petition while the conduct of a certification election is ordered by the Med-Arbiter. In this case, the appellant withdrew its consent and
Notwithstanding the premature canvass of the votes, we note that appellant company failed to support its grounds for challenge with sufficient evidence
opposed the conduct of the election. Therefore, the petition necessarily becomes one of a petition for certification election and the Med-Arbiter was
for us to determine the validity of its claim. No job description of the challenged voters was submitted by appellant from which we can verify whether the
correct in granting the same.9
said voters are indeed disqualified from the alleged five-point characterization made in the 02 May 2000 decision, either before the Med-Arbiter or on
appeal. Neither was the job description of the shift bosses whom appellant company claims pertain to the alleged five-point characterization submitted for
xxxx our perusal. The challenge must perforce fail for lack of evidence.

In the ensuing certification election held on November 28, 2000, the Union garnered 109 of the 111 total valid votes cast. 10
Labor Relations Set IV * Takata v BLR to Silang v Coa * Page 25 of 29

As to the alleged membership of appellee LCU’s member with another union LEU, the issue has been resolved in the 02 May 200[0] decision of Med- And, secondly, the ruling in National Federation of Labor v. Laguesma reiterates St. Martin’s Funeral Home v. National Labor Relations
Arbiter Lontoc which we affirmed on 12 July 2000. Commission,23 where the Court has pronounced that the special civil action of certiorariis the appropriate remedy from the decision of the National Labor
Relations Commission (NLRC) in view of the lack of any appellate remedy provided by the Labor Code to a party aggrieved by the decision of the NLRC.
Accordingly, any decision, resolution or ruling of the DOLE Secretary from which the Labor Code affords no remedy to the aggrieved party may be
WHEREFORE, the appeal is hereby DENIED for lack of merit and the decision of the Med-Arbiter dated 26 April 2001, certifying Lepanto Capataz Union
reviewed through a petition for certiorari initiated only in the CA in deference to the principle of the hierarchy of courts.
as the sole and exclusive bargaining agent of all capataz workers of Lepanto Consolidated Mining Company, is AFFIRMED.SO RESOLVED.16

Yet, it is also significant to note that National Federation of Labor v. Laguesma also reaffirmed the dictum issued in St. Martin’s Funeral Homes v.
Ruling of the CA
National Labor Relations Commission to the effect that "the remedy of the aggrieved party is to timely file a motion for reconsideration as a precondition
for any further or subsequent remedy, and then seasonably avail of the special civil action of certiorari under Rule 65 x x x."24
Still dissatisfied with the result, but without first filing a motion for reconsideration, Lepanto challenged in the CA the foregoing decision of the DOLE
Secretary through a petition for certiorari.
Indeed, the Court has consistently stressed the importance of the seasonable filing of a motion for reconsideration prior to filing the certiorari petition.
In SMC Quarry 2 Workers Union-February Six Movement (FSM) Local Chapter No. 1564 v. Titan Megabags Industrial Corporation 25 and Manila Pearl
On December 18, 2002, the CA dismissed Lepanto’s petition for certiorari, stating in its first assailed resolution: Corporation v. Manila Pearl Independent Workers Union,26 the Court has even warned that a failure to file the motion for reconsideration would be fatal to
the cause of the petitioner.27 Due to its extraordinary nature as a remedy, certiorari is to be availed of only when there is no appeal, or any plain, speedy
or adequate remedy in the ordinary course of law.28 There is no question that a motion for reconsideration timely filed by Lepanto was an adequate
Considering that the petitioner failed to file a prior motion for reconsideration of the Decision of the public respondent before instituting the present petition remedy in the ordinary course of law in view of the possibility of the Secretary of Justice reconsidering her disposition of the matter, thereby according the
as mandated by Section 1 of Rule 65 of the 1997 Rules of Civil Procedure, as amended, the instant "Petition for Certiorari Under Rule 65 with Prayer for
relief Lepanto was seeking.1âwphi1
Temporary Restraining Order and Injunction" is hereby DISMISSED.

Under the circumstances, Lepanto’s failure to timely file a motion for reconsideration prior to filing its petition forcertiorari in the CA rendered the
Well-settled is the rule that the "filing of a petition for certiorari under Rule 65 without first moving for reconsideration of the assailed resolution generally
September 17, 2002 resolution of the DOLE Secretary beyond challenge.
warrants the petition’s outright dismissal. As we consistently held in numerous cases, a motion for reconsideration by a concerned party is indispensable
for it affords the NLRC an opportunity to rectify errors or mistakes it might have committed before resort to the courts can be had.
II.
It is settled that certiorari will lie only if there is no appeal or any other plain, speedy and adequate remedy in the ordinary course of law against acts of
public respondents. Here, the plain and adequate remedy expressly provided by law was a motion for reconsideration of the impugned resolution, based Capatazes are not rank-and-file employees; hence, they could form their own union
on palpable or patent errors, to be made under oath and filed within ten (10) days from receipt of the questioned resolution of the NLRC, a procedure
which is jurisdictional. Further, it should be stressed that without a motion for reconsideration seasonably filed within the ten-day reglementary period, the
Anent the second issue, we note that Med-Arbiter Lontoc found in her Decision issued on May 2, 2000 that thecapatazes were performing functions
questioned order, resolution or decision of NLRC, becomes final and executory after ten (10) calendar days from receipt thereof." (Association of Trade
totally different from those performed by the rank-and-file employees, and that the capatazes were "supervising and instructing the miners, mackers and
Unions (ATU), Rodolfo Monteclaro and Edgar Juesan vs. Hon. Commissioners Oscar N. Abella, Musib N. Buat, Leon Gonzaga, Jr., Algon
other rank-and-file workers under them, assess[ing] and evaluat[ing] their performance, mak[ing] regular reports and recommend[ing] new systems and
Engineering Construction Corp., Alex Gonzales and Editha Yap. 323 SCRA 50).SO ORDERED.17
procedure of work, as well as guidelines for the discipline of employees."29 Hence, Med-Arbiter Lontoc concluded, the capatazes "differ[ed] from the rank-
and-file and [could] by themselves constitute a separate bargaining unit."30
Lepanto moved to reconsider the dismissal, but the CA denied its motion for reconsideration through the second assailed resolution.18
Issues
Agreeing with Med-Arbiter Lontoc’s findings, then DOLE Undersecretary Baldoz, acting by authority of the DOLE Secretary, observed in the resolution
Hence, this appeal by Lepanto based on the following errors, namely:
dated July 12, 2000, thus:31
I
THE COURT OF APPEALS ERRED IN SUMMARILY DISMISSING THE PETITION FOR CERTIORARI ON THE GROUND THAT NO PRIOR MOTION
FOR RECONSIDERATION WAS FILED. THE DECISION OF THE SECRETARY BEING FINAL AND EXECUTORY, A MOTION FOR The bargaining unit sought to be represented by the appellee are the capataz employees of the appellant. There is no other labor organization of
RECONSIDERATION WAS NOT AN AVAILABLE REMEDY FOR PETITIONER. capatazes within the employer unit except herein appellant. Thus, appellant is an unorganized establishment in so far as the bargaining unit of capatazes
II is concerned. In accordance with the last paragraph of Section 11, Rule XI, Department Order No. 9 which provides that "in a petition filed by a legitimate
ON THE MERITS, THE SECRETARY OF LABOR ACTED WITHOUT OR IN EXCESS OF JURISDICTION, [O]R WITH GRAVE ABUSE OF labor organization involving an unorganized establishment, the Med-Arbiter shall, pursuant to Article 257 of the Code, automatically order the conduct of
DISCRETION AMOUNTING TO LACK OR EXCESS OF JURISDICTION IN ISSUNG THE DECISION DATED SEPTEMBER 17, 2002, WHEN SHE certification election after determining that the petition has complied with all requirements under Section 1, 2 and 4 of the same rules and that none of the
DELIBERATELY IGNORED THE FACTS AND RULED IN FAVOR OF THE RESPONDENT UNION, DESPITE HER OWN FINDING THAT THERE HAD grounds for dismissal thereof exists", the order for the conduct of a certification election is proper. 32
BEEN A PREMATURE CANVASS OF VOTES. 19
We cannot undo the affirmance by the DOLE Secretary of the correct findings of her subordinates in the DOLE, an office that was undeniably possessed
Lepanto argues that a motion for reconsideration was not an available remedy due to the decision of the DOLE Secretary being already classified as final of the requisite expertise on the matter in issue. In dealing with the matter, her subordinates in the DOLE fairly and objectively resolved whether the Union
and executory under Section 15, Rule XI, Book V of Omnibus Rules Implementing the Labor Code, as amended by Department Order No. 9, series of could lawfully seek to be the exclusive representative of the bargaining unit of capatazes in the company. Their factual findings, being supported by
1997;20 that the Union’s petition for consent election was really a certification election; that the Union failed to give a definite description of the bargaining substantial evidence, are hereby accorded great respect and finality. Such findings cannot be made the subject of our judicial review by petition under
unit sought to be represented; and that the capatazes should be considered as rank-and-file employees. Rule 45 of the Rules of Court, because:

The issues to be resolved are, firstly, whether a motion for reconsideration was a pre-requisite in the filing of its petition for certiorari; and, secondly, x x x [T]he office of a petition for review on certiorari under Rule 45 of the Rules of Court requires that it shall raise only questions of law. The factual
whether the capatazes could form their own union independently of the rank-and-file employees. findings by quasi-judicial agencies, such as the Department of Labor and Employment, when supported by substantial evidence, are entitled to great
Ruling respect in view of their expertise in their respective field. Judicial review of labor cases does not go far as to evaluate the sufficiency of evidence on which
The petition for review has no merit. the labor official’s findings rest. It is not our function to assess and evaluate all over again the evidence, testimonial and documentary, adduced by the
I. parties to an appeal, particularly where the findings of both the trial court (here, the DOLE Secretary) and the appellate court on the matter coincide, as in
The filing of the motion for reconsideration is a pre-requisite to the filing of a petition for certiorari to assail the decision of the DOLE Secretary this case at bar. The Rule limits that function of the Court to review or revision of errors of law and not to a second analysis of the evidence. Here,
petitioners would have us re-calibrate all over again the factual basis and the probative value of the pieces of evidence submitted by the Company to the
DOLE, contrary to the provisions of Rule 45. Thus, absent any showing of whimsical or capricious exercise of judgment, and unless lack of any basis for
We hold to be untenable and not well taken Lepanto’s submissions that: (1) a motion for reconsideration was not an available remedy from the decision of
the conclusions made by the appellate court may be amply demonstrated, we may not disturb such factual findings.33
the DOLE Secretary because of Section 15, Rule XI, Book V of the Omnibus Rules Implementing the Labor Code, as amended; and (2) the ruling
in National Federation of Labor v. Laguesma21 (recognizing the remedy of certiorari against the decision of the DOLE Secretary to be filed initially in the
CA) actually affirms its position that an immediate recourse to the CA on certiorari is proper even without the prior filing of a motion for reconsideration. In any event, we affirm that capatazes or foremen are not rank-andfile employees because they are an extension of the management, and as such they
may influence the rank-and-file workers under them to engage in slowdowns or similar activities detrimental to the policies, interests or business
objectives of the employers.34
To start with, the requirement of the timely filing of a motion for reconsideration as a precondition to the filing of a petition for certiorari accords with the
principle of exhausting administrative remedies as a means to afford every opportunity to the respondent agency to resolve the matter and correct itself if
need be.22 WHEREFORE, the Court DENIES the petition for review for lack of merit, and AFFIRMS the resolutions the Court of Appeals promulgated on December
18, 2002 and January 31, 2003.Petitioner to pay the costs of suit.SO ORDERED
Labor Relations Set IV * Takata v BLR to Silang v Coa * Page 26 of 29

Republic of the Philippines On 23 November 2007, another Memorandum was sent to the officers of NAMADACWAD requiring them to explain within 72-hours why they should not
SUPREME COURT be held liable for the actions committed by Cagula.10
Manila
EN BANC
Finding prima facie case against them, GM Gamboa filed formal charges against the officers and members of NAMADACWAD as follow:
G.R. No. 194192 June 16, 2015
DAVAO CITY WATER DISTRICT REPRESENTED BY ITS GENERAL MANAGER, RODORA N. GAMBOA,Petitioner,
vs. 1. For DCWD Administrative Case No. 34-2007 against the officials of NAMADACWAD for violation of Existing Civil Service Law and Rules of
RODRIGO L. ARANJUEZ, GREGORIO S. CAGULA, CELESTINO A. BONDOC, DANILO L. BUHAY, PEDRO E. ALCALA, JOSEPH A. VALDEZ, TITO Serious Nature defined under Section 46 [12], Book V of Executive Order No. 292, 11 in relation to Rule IV, Section 52 B [4] of the Civil Service
V. SABANGAN, MARCELINO B. ANINO, JUANITO C. PANSACALA, JOEMARIE B. ALBA, ANTERO M. YMAS, ROLANDO L. LARGO, RENEBOY Resolution No. 99193612 dated August 31, 1999 and Civil Service Resolution No. 02131613 dated October 11, 2002 and MC No. 33 dated
U. ESTEBAN, MANUEL B. LIBANG, ROMEORICO A. LLANOS, ARTHUR C. BACHILLER, SOCRATES V. CORCUERA, ALEJANDRO C. PICHON, October 21, 1994.14
GRACIANO A. MONCADA, ROLANDO K. ESCORIAL, NOEL A. DAGALE, EMILIO S. MOLINA, SHERWIN S. SOLAMO, FULGENCIO I. DYGUAZO,
GUALBERTO S. PAGATPAT, JOSEPH B. ARTAJO, FELIXBERTO Q. OBENZA, FLORANTE A. FERRAREN, ELSA A. ELORDE, CARLOS P.
MORRE, JAMES AQUILINO M. COLOMA, JOAQUIN 0. CADORNA, JR., LORNA M. MAXINO, ROMULO A. REYES, NOEL G. LEGASPI, ELEANOR 2. For DCWD Administrative Case Nos. 11-2007 to 33-2007 and 35-2007 to 44-2007 involving the individual members of NAMADACWAD for
R. LAMOSTE, WELMER E. CRASCO, DELIO T. OLAER, VICENTE R. MASUCOL, IRENEO A. CUBAL, EDWIN A. DELA PENA, JIMMY A. TROCIO, violation of Existing Civil Service Law and Rules of Serious Nature defined under Section 46 [12], Book V of Executive Order No. 292,15 in
WILFREDO L. TORREON, ALEJANDRITO M. ALO, RAUL S. SAGA, JOSELITO P. RICONALLA, TRISEBAL Q. AGUILAR, ARMAN N. LORENZO, relation to Rule IV, Section 52 B [4] of the Civil Service Resolution No. 991936 dated August 31, 1999 and Civil Service Resolution No. 021316
SR. and PEDRO C. GUNTING, Respondents. dated October 11, 2002.
RESOLUTION
PEREZ, J.: After giving those concerned the opportunity to explain through several hearings and submission of additional evidence, the Hearing
This is a Petition for Review on Certiorari1 of the Decision2 of the Twenty Third Division of the Court of Appeals in CA-G.R. SP No. 02793- MIN dated 7 Committee, through the authority given by DCWD to hear the administrative charges, filed on 14 March 2008 its Consolidated Resolution and
October 2010, affirming the 14 January 2009 Resolution No. 09-0047 rendered by the Civil Service Commission (CSC). Recommendation finding the officers and members of the NAMADACWAD guilty as charged with penalties ranging from suspension to
The Facts dismissal from service with all accessory penalties under the CSC Law and Rules. 16

Petitioner Davao City Water District(DCWD) is a government-owned and controlled corporation in Davao City represented by its General Manager Engr. On 19 March 2008, GM Gamboa issued several Orders 17 adopting the recommendation submitted by the Hearing Committee but modifying some of the
Rodora N. Gamboa (GM Gamboa). The private respondents, namely, Rodrigo L. Aranjuez, Gregorio S. Cagula, Celestino A. Bondoc, Danilo L.Buhay, corresponding penalties in view of mitigating circumstances such as first infractionand substantial justice. However, three officials namely Rodrigo L.
Pedro E. Alcala, Joseph A. Valdez, Tito V. Sabangan,Marcelino B. Anino, Juanito C. Pansacala, Joemarie B. Alba, Antero M. Ymas, Rolando L. Largo, Aranjuez, Cagula and Celestino A. Bondoc were penalized with dismissal from the service for the reason that the infraction was the second administrative
Reneboy U. Esteban, Manuel B. Libang, Romeorico A. Llanos, Arthur C. Bachiller, Socrates V. Corcuera, Alejandro C. Pichon, Graciano A . Moncada, offense of serious nature.18
Rolando K. Escorial, Noel A. Dagale, Emilio S. Molina, Sherwin S. Solamo, Fulgencio I. Dyguazo, Gualberto S. Pagatpat, Joseph B. Artajo, Felixberto Q.
Obenza, Florante A. Ferraren, Elsa A. Elorde, Carlos P. Morre, James Aquilino M. Coloma, Joaquin O. Cadorna, Jr., Lorna M. Maxino, Romulo A. Reyes,
Noel G. Legaspi, Eleanor R. Lamoste, WelmerE. Crasco, Delio T. Olaer, Vicente R. Masucol, Ireneo A. Cubal, Edwin A. dela Peña, Jimmy A. Trocio, Aggrieved, Aranjuez, et al., filed an Urgent Motion for Reconsideration19 with Prayer to Suspend the Immediate Execution of the Orders dated 19 March
Wilfredo L. Torreon, Alejandrito M.Alo, Raul S. Saga, Joselito P. Riconalla, Trisebal Q. Aguilar, Arman N. Lorenzo, Sr. and Pedro C. Gunting (Aranjuez, et 2008. The Motion for Reconsideration was thereafter submitted for resolution after the Hearing Committee waived the filing of a Comment. On 17 April
al.) are officers and members of Nagkahiusang Mamumuo sa Davao City Water District (NAMADACWAD). They were charged with several 2008, the Motion was denied by DCWD.
administrative cases due to acts committed during the anniversary celebration of DCWD such as wearing of t-shirts with inscriptions and posting of bond
papers outside the designated places. The inscriptions and postings bore employees’ grievances. On 2 May 2008, Aranjuez, et al., filed an appeal before the CSC bringing up, among other issues, the violation of their constitutional rights to assemble
and petition for redress of grievances.20
The records show that as early as 16 May 2007, the members and officers of NAMADACWAD have been staging pickets in front of the DCWD Office
during their lunch breaks to air their grievances about the non-payment of their Collective Negotiation Agreement (CNA) incentives and their opposition to In its Comment, DCWD defended the Orders on the basis of Section 6 of CSC Resolution No. 02131621 which provides that the concerted activity like the
DCWD’s privatization and proposed One Hundred Million Peso Loan. participation of the officers and employees during the fun run wearing t-shirts with inscriptions was prohibited because it was done during office hours.
Moreover, the act of Cagula in posting papers with grievances outside the designated areas was a clear violation of MC No. 33 in relation to 8 February
On 31 October 2007, GM Gamboa issued an Office Memorandum addressed to all department managers concerning the different activities that would 1996 Office Memorandum. It was submitted that due to Cagula’s membership in the Board of Directors of NAMADACWAD, the other officers were
take place during DCWD’s then upcoming anniversary celebration. The Memorandum reads: solidarily responsible for his actions.22

Please be informed that the opening activities of our 34th anniversary this coming 09 November 2007 are the motorcade and the fun run. The assembly CSC Resolution
area will be at the Victoria Plaza Mall parking, in front of Cynthia’s Lechon Hauz, 6:00 o’clock in the morning.
On 14 January 2009, CSC issued a Resolution23 partly granting the consolidated appeal and held that the collective act of respondents in wearing t-shirts
In view of this, everybody is expected to be there except only those who are assigned as a skeletal force. All carpool vehicles are also enjoined to with grievance inscriptions during office hours was not within the ambit of the definition of prohibited mass action punishable under CSC Resolution
proceed at the said area. The participants are free to wear any sports attire. Further, you are advised to sign in the attendance sheet provided by the 021316 since there was no intent to cause work stoppage. However, though not prohibited under the Resolution, the act was considered as an offense
HRD.3 punishable under "Violation of Reasonable Office Rules and Regulations." CSC further ruled that Cagula’s act of posting of grievances outside the
designated areas was a clear violation of MC No. 33. By reason of Cagula’s position, the other officers of NAMADACWAD were considered as having
agreed and conspired to commit the said act and as such are as liable as Cagula.
On 8 November 2007, the officers and members of NAMADACWAD held an Emergency General Assembly and they agreed to wear NAMADACWAD t-
shirts with inscriptions stating, "CNA Incentive Ihatag Na, Dir. Braganza Pahawa Na!" on the day of the anniversary. 4
On the other hand, and contrary to the assertions of DCWD, the violations committed by the private respondents are not serious in nature due to the lack
of any abusive, vulgar, defamatory or libelous language. The dispositive portion reads:
Came the anniversary, officers and members sported t-shirts with inscriptions "CNA Incentive Ihatag Na, Dir. Braganza Pahawa Na!" at the beginning of
the Fun Run at VictoriaPlaza at around 6:30 in the morning and continued to wear the same inside the premises of the DCWD office during the office
hours. Also, one of the members of the Board of Directors of NAMADACWAD Gregorio S. Cagula (Cagula), with the help of some of its members, WHEREFORE, the Consolidated Appeal filed by Rodrigo L. Aranjuez, et al. is PARTLY GRANTED. The Orders dated March 19, 2008 issued by the
attached similar inscriptions and posters of employees’ grievances to a post in the motor pool area, an area not among the officially designated General Manager Rodora N. Gamboa finding appellants guilty of Violation of Existing Civil Service Law and Rules of Serious Nature (Section 46 [12]
places5 for posting of grievances as prescribed by DCWD’s Office Memorandum 6 dated 8 February 1996 and pursuant to CSC Memorandum Circular No. Book V of Executive Order No. 292, in relation to Rule IV, Section 52 B [4] of the CSC Resolution No. 991936 dated August 31, 1999 and CSC
33,7 Series of 1994 (MC No. 33).8 Resolution No. 021316 dated October 11, 2002 and CSC MC No. 33 dated October 21, 1994), are hereby MODIFIED. Accordingly, appellants are hereby
found liable for Violation of Reasonable Office Rules and Regulations and are meted the following penalties, to wit:

As a consequence of their actions, GM Gamboa sent a Memorandum dated 14 November 2007 addressed to the officers and members of
NAMADACWAD, requiring them to explain the reasons for the attire they wore during the anniversary celebration. Through a collective letter dated 19 1. As to members Danilo Buhay, Pedro E. Alcala, Joseph A. Valdez, Tito V. Sabangan, Marcelino B. Anino, Juanito C. Pansacala, Joemarie B.
November 2007, the officers and members explained that the Memorandum only required the employees to wear any sports attire, though theirs were Alba, Antero M. Ymas, Rolando L. Largo, Reneboy U. Esteban, Manuel B. Libang, Romeorico A. Llanos, Arthur C. Bachiller, Socrates V.
with additional inscriptions containing grievances. They countered that the inscriptions were but manifestations of their constitutional rights of free speech Corcuera, Alejandro C. Pichon, Graciano A. Moncada, Rolando Escorial, Noel A. Dagale, Emilio S. Molina, Sherwin S. Solano, Danilo L. Buhay
and freedom of expression.9 and Fulgencio I. Dyguazo, the penalty of reprimand;

2. As to officers Gualberta S. Pagatpat, Joseph A. Artalo, Felixberto Q. Obenza, Florante A. Ferraren, Elsa A. Ilorde, Carlos P. Morre, James
Aquilino M. Coloma, Joacquin O. Cadorna, Jr., Lorna M. Maximo, Romulo A. Reyes, Noel G.Legazpi, Eleanor R. Lamoste, Welmer E. Crasco,
Labor Relations Set IV * Takata v BLR to Silang v Coa * Page 27 of 29

Delio T. Olaer, Vicente R. Masucol, Ireneo Cubal, Rodrigo L. Aranjuez, Gregorio S. Cagula and Celestino A. Bondoc, the penalty of reprimand Prefatorily, DCWD contends that the appeal of Aranjuez, et al., should have been dismissed by the CSC for non-compliance with Section 46 of CSC
and strong warning that a repetition of the same shall be dealt with severely. Resolution No. 991936, particularly their failure to file a notice of appeal, their failure to show proof of payment of the appeal fee and the petition’s invalid
verification and certification of non-forum shopping.
3. As to members Edwin A. dela Peña, Jummy A. Trocio, Wilfredo A. Torreon, Alejandrito M. Alo, Raul S. Saga, Joselito P. Riconalla, Trisebal
Q. Aguilar,Arman L. Lorenzo, Sr. and Pedro C. Gunting, they are likewise found guilty of the offense of Violation of Reasonable Office Rules We are not persuaded.
and Regulations but are not meted a penalty considering that they are casual employees whose renewal of appointments were held in
abeyance.24
Though the appeal before the CSC lacked a notice of appeal as required by CSC Resolution No. 991936 or the Uniform Rules on Administrative Cases in
the Civil Service (URACCS),27 the Consolidated Memorandum filed by the private respondents was enough to be considered as a sufficient compliance
Aggrieved, DCWD filed a Petition for Review under Rules 43 before the Court of Appeals alleging procedural and substantive infirmities of the CSC with the rules. The Memorandum delineates the errors asserted against DCWD and the discussions supporting their arguments. We find merit in the
Resolution. sufficiency of the Memorandum rather than strict compliance in view of the constitutional right of every employee to security of tenure. A more relevant
consideration of public interest is accorded whenever the merits of a case collide with rigid application of the rules. 28
The Court of Appeals’ Decision
Further, we find that the Civil Service Commission, the agency directly concerned, the ruling of which was upheld by the Court of Appeals on review,
correctly exercised jurisdiction over respondent’s appeal from the decision of petitioner DCWD, thereby ruling against, if sub silentio, the argument of
In its decision, the Court of Appeals affirmed in toto25 the resolution of CSC.
petitioner that the appeal should be dismissed for lack of proof of payment of appeal. The Civil Service Commission and the Court of Appeals considered
the procedural issue raised by petitioner as a surmountable bar to the resolution of the main issue of respondents’ constitutional right to free
The appellate court disagreed with the contention of DCWD that there was a violation of any provision of Resolution No. 021316 in this wise: expression29 as amplified with specificity by their guaranteed right as workers to peaceful concerted activity and their entitlement to security of
tenure.30 The decisions of the Civil Service Commission and the Court of Appeals are squarely supported by Adalim v. Taniñas 31 stating that:
As correctly observed by the Civil Service Commission, the act of respondents in sporting a t-shirt with the inscription "CNA INCENTIVE IHATAG NA,
DIRECTOR BRAGANZA,PAHAWA NA!" during the fun run and even inside the office premises hardly qualifies as a prohibited concerted mass action In a number of cases, we upheld the CSC’s decision relaxing its procedural rules to render substantial justice. The Revised Rules on Administrative
under CSC Resolution No. 021316. Cases in the Civil Service themselves provide that administrative investigations shall be conducted without strict recourse to the technical rules of
procedure and evidence applicable to judicial proceedings. The case before the CSC involves the security of tenure of public employees protected by the
Constitution. Public interest requires a resolution of the merits of the appeal instead of dismissing the same based on a rigid application of the CSC Rules
xxxx of Procedure. Accordingly, both the CSC and the CA properly allowed respondent employees’ appeal despite procedural lapses to resolve the issue on
the merits.
To say the least, Section 5 of Resolution No. 01316 provides a specific guideline as to what constitutes a prohibited concerted activity. A prohibited
concerted activity must be one undertaken by government employees, by themselves or through their association, with the intent of effecting work In Republic of the Philippines v. Court of Appeals, 32 this Court pronounced that technical rules of procedure are not ends in themselves but primarily
stoppage or service disruption, in order to realize their demands or force concessions. In the case at hand, we can readily observe that respondent’s devised and designed to help in the proper and expedient dispensation of justice. In appropriate cases, therefore, the rules may have to be so construed
participation in the fun run, as well as their behavior inside the premises of DCWD office during the regular working hours of that day indicate a complete
liberally as to meet and advance the cause of substantial justice. While it is desirable that the rules of procedure are faithfully and even meticulously
absence of any intention on their part to effect a work stoppage or disturbance. In fact, as attested by both parties, all the respondents participated with observed, courts should not be so strict about procedural lapses that do not really impair the proper administration of justice. If the rules are intended to
the planned activities and festivities on that day.26 ensure the orderly conduct of litigation, it is because of the higher objective they seek which is the protection of substantive rights of the
parties.33 Substantial justice, in other words must prevail. In Paler,34 We said:
The appellate court was likewise in agreement with the CSC which considered as simple violation of office rules the posting of banners outside the
designated posting areas by Cagula. Also like the CSC, it ruled that such offense is not punishable with the penalty of dismissal. When substantial justice dictates it, procedural rules may be relaxed in order to arrive at a just disposition of a case. The purpose behind limiting the
period of appeal is to avoid unreasonable delay in the administration of justice and to put an end to controversies. A one-day delay as in this case, does
The DCWD is now before us still with its basic arguments, though rephrased: not justify denial of the appeal where there is absolutely no indication of intent to delay as in this case, does not justify denial of the appeal where there is
absolutely no indication of intent to delay justice on the part of Paler and the pleading is meritorious on its face.

We rule in favor of the allowance of respondents’ appeal because:


I.
The court a quo failed to rule on the issue whether or not the respondents’ Consolidated Appeal filed before the CSC was sufficient in form and Law and jurisprudence grant to courts the prerogative to relax compliance with procedural rules of even the most mandatory character, mindful of the duty
substance. to reconcile both the need to put an end to litigation speedily and the parties’ right to an opportunity to be heard. 35 (Emphasis supplied)
II.
The court a quo erred in ruling that the concerted mass action on November 9, 2007 was not prohibited under Resolution No. 021316. Quoting again the case of Republic v. Court of Appeals,36 we pointed out that this Court can temper rigid rules in favor of substantial justice. We find that
III. pronouncement apt and fit to this case. Thereby we are not detained by the omissions of the respondents in their resort to the CSC, and we thus proceed
The court a quo erred in ruling that Resolution No. 021316 and MC No. 33 are considered "reasonable office rules and regulations" within the purview of
to the merits of the petitioners’ submissions.
Section 52 C [3] of the Uniform Rules on Administrative Cases.
IV.
The court a quo erred in ruling that respondents’ act of posting white bond papers with union-related inscriptions on their t-shirts while inside the office Lastly, on the form, we find no merit in the contention that Aranjuez was not authorized to sign on behalf of the other petitioners. Pursuant to Union
premises does not constitute serious violation of Civil Service Rules but only a violation of Reasonable Office Rules and Regulations, despite the fact that Resolution No. 015-200837 attached as Annex A to the Appellants’ 015-2008 Consolidated Memorandum dated 26 March 2008, the officers and members
the said Memorandum Circular No. 33 is a CSC-issued Memorandum and not DCWD-issued Rules. of NAMDACWAD gave Aranjuez a general authority to represent the organization in all legal matters to be filed for whatever purpose it may serve. From
V. the general and broad grant of authority, Aranjuez possessed the specific authority to sign in behalf of his principal the verification and certification
The court a quo erred in ruling that MC No. 33 was not violated by respondent Gregorio S. Cagula and the rest of the officials of NAMADACWAD who against non-forum shopping required of the petition.
were charged in DCWD Administrative case No. 34-2007.
VI.
To the kernel, then.
The court a quo erred in not taking into consideration that respondents Aranjuez, Cagula and Bondoc were second-time offenders who were previously
charged and penalized for violation of MC No. 33, thereby justifying their dismissal from the service.
VII. DCWD primarily contends that CSC and the Court of Appeals erred in ruling that the concerted mass action on 9 November 2007 is not prohibited under
The court a quo erred when it failed to rule on the issue of whether the decisions of a government agency, acting as Disciplining Authority, in disciplinary Resolution No. 021316. We disagree. DCWD relies on Resolution No. 021316, which states:
cases are immediately executory upon receipt thereof.
Section 6. Permissible Concerted Mass Action.– A concerted activity or mass action done outside of government office hours shall not be deemed a
The Court's Ruling prohibited concerted activity or mass action within the contemplation of this omnibus rules provided the same shall not occasion or result in the disruption
of work or service.38
The Court finds no merit in the petition.
Labor Relations Set IV * Takata v BLR to Silang v Coa * Page 28 of 29

DCWD argues that since the concerted or mass action was done within government office hours, such act was not permissible, therefore prohibited. 4. No poster, placard, streamer or other similar materials containing abusive, vulgar, defamatory or libelous language shall be allowed.
Otherwise stated, a concerted activity done within the regular government office hours is automatically a violation of Section 6 of the Resolution.
Pursuant to this mandate, the former General Manager of DCWD issued an office memorandum designating the bulletin board at the motor pool area
Notably, however, a prohibited concerted mass action is defined not in Sec. 6 of Resolution No. 021316 but in Sec. 5 thereof. Thus: below the Office of the Purchasing Division and the side of the office building beside the guard house where the bundy clock is located as the designated
areas for posting of grievances.44 Clearly, the DCWD Office Memorandum hews close and faithfully to MC No. 33. It is a reasonable rule issued by the
heads of the agencies in order to regulate posting of grievances of the employees.
Section 5. Definition of Prohibited Concerted Mass Action. - As used in this Omnibus Rules, the phrase ‘‘prohibited concerted activity or mass action’’
shall be understood to refer to any collective activity undertaken by government employees, by themselves or through their employees organizations, with
the intent of effecting work stoppage or service disruption in order to realize their demands of force concession, economic or otherwise, from their It is correct to conclude that those who enter government service are subjected to a different degree of limitation on their freedom to speak their mind;
respective agencies or the government. It shall include mass leaves, walkouts, pickets and acts of similar nature.39(Emphasis ours). however, it is not tantamount to the relinquishment of their constitutional right of expression otherwise enjoyed by citizens just by reason of their
employment.45 Unarguably, a citizen who accepts public employment "must accept certain limitations on his or her freedom." But there are some rights
and freedoms so fundamental to liberty that they cannot be bargained away in a contract for public employment. It is the Court’s responsibility to ensure
The operative phrases are "any collective activity" and "work stoppage or service disruption." Without the intent at work stoppage or service disruption,
that citizens are not deprived of these fundamental rights by virtue of working for the government.46
the concerted activity is not prohibited. The time and place of the activity are not determinative of the prohibition. Whether done within government hours,
a concerted activity is allowed if it is without any intent at work stoppage.
The GSIS case pronounced:
We cannot isolate the provision of Section 6 of the Resolution from definition of prohibited activity in Section 5 thereof. It is erroneous to interpret the
provisions in such a way that an act not within the circumstances as defined under Section 5 can still be regarded as prohibited if done within government Government workers, whatever their ranks, have as much right as any person in the land to voice out their protests against what they believe to be a
hours. To subscribe to the argument of DCWD would in effect expand the definition provided by Resolution No. 021316 on what constitutes a prohibited violation of their rights and interests. Civil Service does not deprive them of their freedom of expression. It would be unfair to hold that by joining the
mass action. government service, the members thereof have renounced or waived this basic liberty. This freedom can be reasonably regulated only but can never be
taken away.47
It is clear that the collective activity of joining the fun run in t-shirts with inscriptions on CNA incentives was not to effect work stoppage or disrupt the
service. As pointed out by the respondents, they followed the advice of GM Gamboa "to be there" at the fun run. Respondents joined, and did not disrupt In simple paraphrase we say, regulation of the freedom of expression is not removal of the constitutional right.
the fun run. They were in sports attire that they were allowed, nay required, to wear. Else, government employees would be deprived of their
constitutional right to freedom of expression. 40 This, then, being the fact, we have to rule against the findings of both the CSC and Court of Appeals that
Apparently, DCWD, not satisfied by the CSC ruling that a violation of the memorandum is punishable with reprimand, argues that what occurred was a
the wearing of t-shirts with grievance inscriptions constitutes as a violation of Reasonable Office Rules and Regulations.
serious violation implying that a higher penalty is warranted.

First off and as correctly pointed out by the charged officials and members in their 19 November 2007 Reply Letter to DCWD, they did not violate the 31
Under Section 52 (C) (3), Rule IV of Resolution No. 991936,48 violation of reasonable office rules and regulations is punishable with reprimand on the first
October 2007 Office Memorandum issued by GM Gamboa relating to the proper attire to be worn during the fun run. The Office Memorandum was clear
offense and suspension ranging from one to thirty days for the second offense.
in its order that the participants are free to wear any sports attire during the event. To reiterate, the t-shirts they wore fall within the description of "any
sports attire" that the Memorandum allowed to be worn.
In Re: Failure of Various Employees to Register their Time of Arrival and/or Departure from Office in the Chronolog Machine, the charged court
employees were penalized for violation of reasonable office rules and regulations due to their violation of Supreme Court Administrative Circular No. 36-
More importantly we need to refer to GSIS v. Villaviza (GSIS case).41 It was there ruled that the acts of GSIS employees wearing similarly colored shirts
2001 requiring all employees to register their daily attendance, in the Chronolog Time Recorder Machine (CTRM) and in the logbook of their respective
while attending a public hearing inside the GSIS Office, with clenching of fists and orating against the then President Winston Garcia, were not
offices. Following Resolution No. 991936 that violation of reasonable rules and regulations is a light offense, the Court penalized its erring employees with
constitutive of a prohibited activity but were only an exercise of their constitutional freedom of expression. 42 We repeat:
the penalty of reprimand.49

In this case, CSC found that the acts of respondents in going to the GSIS-IU office wearing red shirts to witness a public hearing do not amount to a
Thus, in line with the civil service rules and jurisprudence, we conclude that a violation of an office memorandum, which was issued as an internal rule to
concerted activity or mass action proscribed above. CSC even added that their actuations can be deemed an exercise of their constitutional right to
regulate the area for posting of grievances inside the office premise, is only a light offense punishable by reprimand.
freedom of expression. The CA found no cogent reason to deviate therefrom.

Rules and regulations are issued to attain harmony, smooth operation, maximize efficiency and productivity, with the ultimate objective of realizing the
As defined in Section 5 of CSC Resolution No. 02-1316 which serves to regulate the political rights of those in the government service, the concerted
functions of particular offices and agencies of the government.50
activity or mass action proscribed must be coupled with the "intent of effecting work stoppage or service disruption in order to realize their demands of
force concession. "Wearing similarly colored shirts, attending a public hearing at the GSIS-IU office, bringing with them recording gadgets, clenching their
fists, some even badmouthing the guards and PGM Garcia, are acts not constitutive of an (i) intent to effect work stoppage or service disruption and (ii) On the submissions that the decisions of a government agency, acting as Disciplining Authority, are immediately executory upon receipt thereof, we need
for the purpose of realizing their demands or force concession. merely cite Section 37 of the Resolution No. 991936 which clearly provides that:

Precisely, the limitations or qualifications found in Section 5 of CSC Resolution No. 02-1316 are there to temper and focus the application of such Section 37. Finality of Decisions. — A decision rendered by heads of agencies whereby a penalty of suspension for not more than thirty (30) days or a
prohibition. Not all collective activity or mass undertaking of government employees is prohibited. Otherwise, we would be totally depriving our brothers fine in an amount not exceeding thirty (30) days' salary is imposed, shall be final and executory. However, if the penalty imposed is suspension exceeding
and sisters in the government service of their constitutional right to freedom of expression. 43 thirty (30) days, or fine in an amount exceeding thirty (30) days salary, the same shall be final and executory after the lapse of the reglementary period for
filing a motion for reconsideration or an appeal and no such pleading has been filed. 51
DCWD also found that Cagula and the rest of the officials violated MC No. 33 in relation to 8 February 1996 Office Memorandum. DCWD also argues that
a violation of this circular constitutes as a serious violation of CSC Rules as the circular is a CSC-issued Memorandum and not just a mere issuance of As distinguished by the law, if the imposed suspension exceeds thirty days or the fine imposed is in an amount over thirty-day salary, the decision will
DCWD. only attain finality after the lapse of the reglementary period in the absence of any motion for reconsideration or appeal. Penalties within the 30-day
threshold are immediately executory penalties.
CSC issued MC No. 33 in recognition of the rights of the government employees to air their grievances balanced by the delivery of services to the public
which should not be prejudiced. MC No. 33 sets down rules governing the posting of posters and other similar materials within the premises of In this case, the members and officials, except the casual employees who were not meted with penalty as the renewal of their employment was held in
government agencies as follows: abeyance, were sanctioned with penalties ranging from suspension of work from one (1) month and one (1) day to dismissal from service. 52 Evidently, the
finality and execution of the judgment did not take place after the lapse of the reglementary period because as previously discussed, the members and
officials were able to file their consolidated appeal in lieu of notice of appeal.
1. All head of agencies are hereby directed to provide specific spaces within their respective premises, preferably near the bundy clock, at the
canteen or places normally frequented by employees, where employees’ unions/associations could post their posters.
As clear as the provision on the finality of decisions is Section 42 of Resolution No. 991936 on the effect of motions for reconsideration.1âwphi1 Thus:
2. x x x.
Section 42. Effect of Filing. — The filing of a motion for reconsideration within the reglementary period of fifteen (15) days shall stay the execution of the
decision sought to be reconsidered.53 (Emphasis ours)
3. The hanging of posters and streamers shall only be allowed in the designated areas.
Labor Relations Set IV * Takata v BLR to Silang v Coa * Page 29 of 29

The first and fundamental duty of the Court is to apply the law. If the law is clear and free from any doubt or ambiguity as the quoted provision, there is no
room for construction or interpretation. The letter must be taken to mean exactly what it says and the court has no choice but to see to it that its mandate
is obeyed.54

The ponente appreciates the concurrence of Justice Marvic M.V.F. Leonen. No need was seen, though, to add to the ruling that the present facts limited.

WHEREFORE, We DENY the petition for review on certiorari. Nonetheless, the decision of the CSC which was affirmed in toto by the CA is MODIFIED.
The finding of administrative liability of and the penalty of reprimand against the NAMADACWAD members namely Danilo L. Buhay, Pedro E. Alcala,
Joseph A. Valdez, Tito V. Sabangan, Marcelino B. Anino, Juanito C. Pansacala, Joemarie B. Alba, Antero M. Ymas, Rolando L. Largo, Reneboy U.
Esteban, Manuel B. Libang, Romeorico A. Llanos, Arthur C. Bachiller, Socrates V. Corcuera, Alejandro C. Pichon, Graciano A. Moncada, Rolando K.
Escorial, Noel A. Dagale, Emilio S. Molina, Sherwin S. Solamo, and Fulgencio I. Dyguazo are hereby REVERSED and SET ASIDE.

The finding of liability against the casual employees namely Edwin A. dela Peña, Jummy A. Trocio, Wilfredo L. Torreon, Alejandrito M. Alo, Raul S. Saga,
Joselito P. Riconalla, Trisebal Q. Aguilar, Arman N. Lorenzo, Sr. and Pedro C. Gunting is REVERSED and SET ASIDE.

As to officers Gualberto S. Pagatpat, Joseph B. Artajo, Felixberto Q. Obenza, Florante A. Ferraren, Elsa A. Elorde, Carlos P. Morre, James Aquilino M.
Coloma, Joaquin O. Cadorna, Jr., Lorna M. Maxino, Romulo A. Reyes, Noel G. Legaspi, Eleanor R.Lamoste, Welmer E. Crasco, Delio T. Olaer, Vicente
R. Masucol, Ireneo Cubal, Rodrigo L. Aranjuez, Gregorio S. Cagula and Celestino A. Bondoc, the penalty of reprimand and strong warning that a
repetition of the same shall be dealt with severely is hereby AFFIRMED.

SO ORDERED.JOSE PORTUGAL PEREZ


Associate Justice

You might also like